mckinsey casebook

149
Das Consulting Case-Boook Initiiert durch das WHU 2000 Career-Management-Team

Upload: ferry-triwahyudi

Post on 26-Dec-2015

196 views

Category:

Documents


16 download

TRANSCRIPT

Page 1: Mckinsey CaseBook

Das Consulting Case-Boook

Initiiert durch das WHU 2000 Career-Management-Team

Page 2: Mckinsey CaseBook

1

Die meisten Beratungsunternehmen benutzen „Cases“ im Rekrutierungs-Prozeß. Die Interviewer erhoffen sich davon Informationen über die Fähigkeit der Kandidaten, ihre eigenen Gedanken zu strukturieren, sinnvolle Fragen zu stellen und zu plausiblen Lösungen zu gelangen. Dabei ist die Vorgehensweise mindestens so wichtig wie die Antwort selbst.

Dieses WHU-Consulting-Case-Buch soll Euch bei der Vorbereitung auf solche Case-Interviews helfen. Ihr findet eine kurze Einführung in die Beratungsbranche (aus einer US-amerikanischen Perspektive); grundlegende Konzepte werden im Ansatz wiederholt und einige allgemeine Interview-Ratschläge sind aufgeführt. Den Kern bilden jedoch rund 75 beispielhafte Cases, die am besten in Probe-Interviews zu zweit geübt werden können.

Grundlage für dieses Buch waren die von den Consulting-Clubs der J.L. Kellogg Graduate School of Mangement und der University of Michigan herausgegebenen Case-Sammlungen. Aufbereitet und zusammengestellt wurde es vom WHU 2000 Team Career-Management.

Wir wünschen Euch viel Spaß bei der Vorbereitung und viel Erfolg in den Interviews!

Wolfram Gerlof Carolin Torner Bernd Trautwein Robert Vollrath Malte Wulfetange

Page 3: Mckinsey CaseBook

2

Inhaltsverzeichnis

1 OVERVIEW OF THE CONSULTING INDUSTRY ............................................................ 5

1.1 WHAT IS CONSULTING? .............................................................................................................. 5

2 CONSULTING ARTICLES .................................................................................................. 11

2.1 THE MANAGEMENT CONSULTING INDUSTRY ............................................................................ 11 2.2 REFLECTIONS ON FIRST-YEAR RECRUITING .............................................................................. 15

2.3 IS CONSULTING THE RIGHT FIELD FOR YOU? ............................................................................ 23

3 OVERVIEW OF CASES ....................................................................................................... 29

4 DIFFERENT TYPES OF CASES ........................................................................................ 32

5 FRAMEWORKS FOR CASES ............................................................................................ 33

5.1 GENERAL MODELS ................................................................................................................... 33 5.2 ADDITIONAL MODELS ............................................................................................................... 35

6 DOS AND DON’TS ................................................................................................................ 46

7 HOW TO PREPARE FOR THE CONSULTING INTERVIEWS? .................................. 48

8 GENERAL INTERVIEW QUESTIONS ............................................................................. 49

9 SAMPLE CASES ................................................................................................................... 50

9.1 CHINA PRODUCTS DIVISION ...................................................................................................... 50 9.2 HEALTHCARE COMPANY........................................................................................................... 52 9.3 ELECTRONIC JOINT VENTURE ................................................................................................... 53

9.4 TELEVISION CABLE COMPANY .................................................................................................. 54 9.5 MAGAZINE SUNDAY SUPPLEMENT ............................................................................................ 56 9.6 AMERICAN EXPRESS CHARGE CARD ......................................................................................... 57 9.7 TELEVISION CABLE COMPANY .................................................................................................. 58 9.8 CREDIT CARD DIVISION OF BANK ............................................................................................. 60 9.9 MOVIE RENTAL BUSINESS ........................................................................................................ 61 9.10 AUTO SERVICE STORES ............................................................................................................ 63 9.11 SPORTS FRANCHISE .................................................................................................................. 64

9.12 DURABLE GOODS DISTRIBUTION CASE ..................................................................................... 65 9.13 BUSINESS FORMS CASE ............................................................................................................ 67 9.14 “HIGH-END” POTS & PANS COMPANY CASE ............................................................................ 68 9.15 PAPER PRODUCTS MANUFACTURER CASE ................................................................................ 69 9.16 PIANOS ..................................................................................................................................... 71 9.17 COKE VS. RC VALUE CHAIN ..................................................................................................... 72 9.18 FERTILIZER ............................................................................................................................... 74 9.19 AIRPLANE MANUFACTURER ...................................................................................................... 75 9.20 MYSTERIOUS AUDIOCASSETTE MARKET................................................................................... 75

9.21 WINDMILL ................................................................................................................................ 76 9.22 BANK OF LUKE ......................................................................................................................... 77 9.23 CANDY COMPANY .................................................................................................................... 79 9.24 SKYSCRAPER ............................................................................................................................ 80 9.25 CONSULTING FIRM (I) ............................................................................................................... 81 9.26 COSMETIC COMPANY IN EUROPE .............................................................................................. 82 9.27 SEMICONDUCTORS .................................................................................................................... 84 9.28 AIRLINE INDUSTRY ................................................................................................................... 85

9.29 OIL TANKER ............................................................................................................................. 86 9.30 FERTILIZER ............................................................................................................................... 86

Page 4: Mckinsey CaseBook

3

9.31 RETAIL ADVERTISING PRICING .................................................................................................. 87 9.32 AUTOMOBILE INDUSTRY ........................................................................................................... 87 9.33 SCIENTIFIC INDUSTRY ............................................................................................................... 89 9.34 ALUMINUM INDUSTRY .............................................................................................................. 91 9.35 MEAT PACKING INDUSTRY ....................................................................................................... 92 9.36 PIANO TUNERS.......................................................................................................................... 93 9.37 CONSULTING FIRM STRATEGY .................................................................................................. 95 9.38 CORN FEED COMPANY .............................................................................................................. 97

9.39 SELECTIVE BINDING CASE ........................................................................................................ 99 9.40 VIDEO GAMES ........................................................................................................................ 102 9.41 STEAM BOILER HOSES ............................................................................................................ 105 9.42 MERGER CANDIDATE IN CHEMICAL INDUSTRY ....................................................................... 106 9.43 MACHINE-LOADING CASE....................................................................................................... 108 9.44 OIL REFINING INDUSTRY ......................................................................................................... 110 9.45 AGRICULTURAL EQUIPMENT MANUFACTURER ....................................................................... 111 9.46 INSURANCE COMPANY ............................................................................................................ 113

9.47 CONSULTING FIRM (2) ............................................................................................................ 113 9.48 POTS & PANS (2) .................................................................................................................... 114 9.49 DIAPERS .................................................................................................................................. 114 9.50 CABLE TELEVISION COMPANY (2) .......................................................................................... 115 9.51 CHILLED BEVERAGES ............................................................................................................. 116 9.52 DISTILLED SPIRITS .................................................................................................................. 117 9.53 CHEWING GUM MARKET ........................................................................................................ 118 9.54 FRENCH PIZZA MARKET ......................................................................................................... 118

9.55 GOLFBALL MARKET ENTRY .................................................................................................... 119 9.56 OVERSEAS CONSTRUCTION ..................................................................................................... 119 9.57 PACKAGING MATERIAL MANUFACTURER ............................................................................... 120 9.58 AIRLINE EXPANSION ............................................................................................................... 120 9.59 HEALTH CARE COSTS ............................................................................................................. 121 9.60 LOCAL BANKING DEMAND ..................................................................................................... 122 9.61 FROZEN DESSERTS .................................................................................................................. 123 9.62 DIRECT MAIL RETAILER ......................................................................................................... 123 9.63 CHEMICAL SWEETENER MANUFACTURER ............................................................................... 124

9.64 TELECOMMUNICATIONS DIVERSIFICATION .............................................................................. 125 9.65 ALUMINIUM CAN MANUFACTURER ........................................................................................ 126 9.66 FILM PROCESSING ................................................................................................................... 127 9.67 CONCRETE MANUFACTURER .................................................................................................. 127 9.68 SHIPPING CONTAINER MANUFACTURER .................................................................................. 128 9.69 HEALTHCARE COMPANY GROWTH ......................................................................................... 129 9.70 REGIONAL GROCERY STORE CHAIN ........................................................................................ 130 9.71 MAGAZINE DISTRIBUTION ....................................................................................................... 131

9.72 KNITTING MACHINE DEMAND ................................................................................................ 132 9.73 CEMENT MANUFACTURER CAPACITY ADDITION .................................................................... 132 9.74 SNACK FOOD COMPANY ......................................................................................................... 134 9.75 BEVERAGE COMPANY COST STRUCTURE................................................................................ 135 9.76 PERMANENT LIGHT BULBS ..................................................................................................... 136 9.77 SUPER REGIONAL BANK ......................................................................................................... 136 9.78 CIGAR BAR ............................................................................................................................. 138 9.79 NEW MAGAZINE ..................................................................................................................... 139

9.80 CASTOR MANUFACTURER ....................................................................................................... 140 9.81 LOGGING COMPANY ............................................................................................................... 141 9.82 INFORMATION SERVICES COMPANY ........................................................................................ 142 9.83 PIPELINE COMPANY ................................................................................................................ 144 9.84 AUTO MANUFACTURER .......................................................................................................... 146 9.85 DELI MEAT PRODUCER ........................................................................................................... 147

Page 5: Mckinsey CaseBook

4

Page 6: Mckinsey CaseBook

5

1 Overview of the Consulting Industry

1.1 What is Consulting?

By the University of Chicago Graduate School of Business Management Consulting

Group:1

In this section, we will provide an overview of the profession, the types of consulting, projects, and how consulting firms are structured.

Understanding each firm's approach to consulting services is extremely important to landing a job – that is why corporate presentations can be so valuable, provided that you come with specific questions you would like answered.

1.1.1 Why Do Companies Hire Consultants?

There are several reasons that firms hire consultants:

1. To obtain an objective viewpoint regarding a given business problem or

issue. Consultants are relatively unaffected by a company's politics or the way in which business was conducted in the past, so the consulting firm delivers what is perceived as an objective analysis. This perspective can be important for motivating employees to change.

2. To utilize the specific expertise of the consulting firm. For example, the consulting firm may offer an industry authority to which the client would like access. Additionally, the consulting firm may have done similar projects in the past for comparable companies.

3. To obtain information about where the company stands in an industry.

Consulting firms often develop benchmark data on the performance of industry average and best-in-class companies in order to provide expert advice regarding performance improvements.

4. To provide resources to address a specific problem. Often, clients simply do not have enough time or resources to devise solutions to certain problems. Consulting firms can avoid the day-to-day distractions that the clients' managers cannot. Further, consultants may offer labor power to coordinate and execute an implementation.

1.1.2 Consulting Project Types

Generally, consulting firms classify their services into of three categories: Strategy, Business Process Reengineering (or simply "Reengineering") and Specific Services. These categories are not mutually exclusive and the distinctions can easily blur. In effect, there are as many different types of consulting projects as there are business

1 This article was obtained from the 1997-1998 Resource Guide prepared by the Management

Consulting Group of the University of Chicago. Please note that portions of the article that were only applicable to the Graduate School of Business (at the University of Chicago) have not been included for the convenience of the reader.

Page 7: Mckinsey CaseBook

6

problems. We will try to explain each of these types in some detail, but keep in mind that it is impossible to describe the full spectrum of consulting services in this Guide.

Regardless of project type, client involvement is extremely important to the eventual success of any project. Firms follow very different approaches to involving client personnel. For example, some firms require a certain amount of full-time client resources dedicated to the project. Others require only sporadic assistance for portions of the project, such as financial analysis or engineering problems. In extreme cases, client personnel become an integral part of the consulting project.

1.1.2.1 Strategy Consulting

Strategy is the most difficult type of project to explain, because it means different things to different firms. Generally, a strategy project involves a "life cycle crossroads" for the client. For example, determining if the client should expand its product line or focus on existing products, or deciding what services the company should provide ten years from now are examples of strategic projects. A strategy consulting engagement will typically involve the highest levels of the client's organization, since responsibility for the direction of the company lies there.

Most consulting firms will perform a "Five Forces"-style or value chain analysis (both from Michael Porter's book Competitive Strategy) to evaluate all strategic options available to a firm and determine a suggested or potential course of action. This would include a detailed financial projection of the different scenarios. After recommending a given strategy, the project would either conclude or lead to an implementation phase.

Implementation is a major issue among consultants today. Consultants who permit the client to implement a solution believe that success will be realized when the client is forced to take ownership of the solution. On the other hand, other consultants argue that, because their firm was instrumental in developing the solution, they ought to assist the client in implementing the solution. There is a definite trend in the consulting industry toward having consultants assist in implementation. In fact, more often than ever, consultants are being judged by clients on their ability to implement change.

1.1.2.2 Business Process Reengineering

The term reengineering has been popular since Hammer and Champy's book Reengineering the Corporation became a best seller. There is nothing mystical about the term - it simply means taking an objective look at the way in which a business is run. For example, through benchmarking against similar companies, a firm may decide that it takes too long to fill customers' orders. A consultant would then analyze the individual steps of the order fulfillment process and determine ways to cut time, increase quality, enhance customer satisfaction, etc. A revised process is determined and then proposed to the client. Reengineering engagements more often include an implementation phase in a project than do strategy engagements. Some recent literature suggests that reengineering is losing favor and that certain firms are distancing themselves from the term, if not the practice.

Page 8: Mckinsey CaseBook

7

1.1.2.3 Specific Services

Another component of the consulting industry concerns itself with specific tasks and expertise needed by clients. Although the various issues relevant to this type of consulting are innumerable, a few specific areas are currently prominent:

1.1.2.4 Technology and Systems Consulting

Systems consulting is chiefly concerned with giving clients advice about the ideal configuration of their information systems, the introduction of client-server computing, and software and hardware purchases.

1.1.2.5 Human Resources Consulting

Human resources (HR) consultants help firms make compensation decisions and offer insights on benefit packages, pension funding, workplace diversity, and employee development. Executive compensation is a hot topic in HR consulting.

1.1.2.6 Litigation Consulting

These consultants work with law firms to plan case strategies, provide economic analysis, and develop courtroom tactics and/or evidentiary presentations.

1.1.2.7 Financial Consulting

Finance consultants provide guidance to corporations and money managers in the areas of securities pricing, economic forecasts, and strategies for creating shareholder value.

1.1.2.8 Other Industry-Specific Services

Many niche firms fall into this category. For example, in the healthcare consulting field, consultants are often asked to justify the need to build a new hospital (a "feasibility study"). The financial backers of the new hospital would rely on the consultant's findings before proceeding with construction.

1.1.3 Trends

Growth - Major consulting firms have been boasting double-digit rates of growth.

Overseas Expansion - Much of the growth in the consulting industry has been international, with firms competing to build a client base in various countries.

Range of Services - Many firms have moved toward offering a broader range of services (e.g., strategy through implementation). Mergers and expansions are fueling this trend.

Decreasing Growth of Strategy Consulting – After the restructuring, downsizing, and reengineering phase of the 80's and early 90's, strategic projects have developed around continued growth and expansion overseas.

Page 9: Mckinsey CaseBook

8

1.1.4 The Structure of Consulting Firms

Most firms will have very few job classifications. Titles vary by firm, but the responsibilities are generally similar. The following are the basic classifications and job descriptions:

1.1.4.1 Business Analyst I Analyst

These positions are not held by MBAs, but rather by the most capable individuals right out of top undergraduate programs. The business analyst position is typically held for 1-3 years between undergraduate and graduate school. The analyst's responsibilities range from research and data gathering to functioning on a level equal to post-MBA consultants.

1.1.4.2 Associate / Senior Consultant

Entry-level for MBAs. The associate is usually given the role of information gatherer. This will typically involve research, obtaining information from clients via interviews and/or financial data, and analyzing the information to draw conclusions. These conclusions must usually be presented to the rest of the project team in the format of a presentation. In projects where there is a client team, the associate may manage a subgroup of client team members. Associates are often asked to present part of the project team's findings to the client because associates are typically most familiar with the data collected. The associate is typically given very broad directions and is expected to be creative and thorough in collecting relevant information.

1.1.4.3 Senior Associate / Engagement Manager

The senior associate or manager classification implies day-to-day supervisory responsibility on engagements. The senior associate will manage client team members (if applicable), consultants, and business analysts on the project. At the senior associate level, the project budget becomes a concern.

1.1.4.4 Principal /Associate Partner / Senior Manager

Those at the principal level are required either to manage several projects simultaneously or one large project full-time. Client relationships are critical at all job classification levels, but particularly in this case because the principal typically has the most frequent contact with upper level management. Frequent contact helps to ensure additional projects in the future. The principal is responsible for setting the direction for a project, with approval from the managing partner on the engagement. In many cases, the principal also begins to take on administrative duties within the firm.

1.1.4.5 Partner/ Director/ Vice President

Partners are responsible both for negotiating engagements and for reviewing the work generated by those engagements. The ultimate responsibility for a project's success falls on the partner's shoulders. With several projects to oversee at once in addition to

Page 10: Mckinsey CaseBook

9

their marketing duties, partners are often the hardest-working consultants in the office. A partner's travel schedule is generally more hectic than that of the more junior consultants. As the partner juggles several projects at one time, the partner may only periodically visit each client site. The partner attends important meetings with senior client managers.

1.1.5 Compensation

The median salary for full-time consulting positions was $85,000 in the most recent recruiting year. For internships, consulting firms usually pay the monthly equivalent of their full-time salaries. Keep in mind, however, that first-year total compensation is usually much higher. For example, most firms offer a signing bonus of $10,000 to $25,000. Also, some firms will pay for the second year of business school (and recently, one firm offered to pay for both years of business school as part of their full-time offers to summer interns). A new employee is sometimes eligible for a performance bonus after the first year. In short, compensation is outstanding compared to what most of us were being paid before business school.

1.1.6 Lifestyle

So far, consulting looks like the ideal job: immediate responsibility, opportunity to make a difference, excellent pay, etc. For some people it is. These people are known as partners. Since only about one percent of consultants go on to become partners, what happens to the other 99 percent? There are two main reasons for the attrition. First, since consultants are in such high profile positions, they typically receive job offers from clients and frequent contacts from corporate recruiters. Second, being in a position of responsibility usually translates into long hours in the office and frequent travel. This does not leave much time for a personal life. In addition, if a spouse or children are in the picture, it may not be possible to “have it all.” The greatest amount of attrition occurs around the three- to four-year mark, when consultants have gained enough experience to be offered positions involving a better balance of work and personal life at the same or higher compensation.

Given the high investment made by consulting firms in developing personnel, reducing attrition can save a lot of money. Lately, firms have implemented programs designed to lessen the burden on consultants and, theoretically, prevent valued employees from wanting to look elsewhere. To reduce the out-of-town burden, many firms have institutionalized Fridays in the office or allow consultants to work from home on Fridays. This limits their being away from family and friends only three nights per week. Other firms have a more office-intensive style that involves going to the client site only when necessary. To address concerns about raising a family, some firms have recently instituted part-time programs. Most of these programs only require three days of work per week.

If you have lifestyle concerns, the best time to ask these questions of firms is during the recruiting receptions. These receptions are extremely low-risk, so do not be shy about asking tough questions concerning the amount and frequency of travel and other lifestyle concerns. Contrary to popular belief, it is very hard to be rejected

Page 11: Mckinsey CaseBook

10

because of one's performance at a reception. It is also possible to ask lifestyle questions of recent alumni or second-years that interned at the firm in question.

Page 12: Mckinsey CaseBook

11

2 Consulting Articles

2.1 The Management Consulting Industry

By David J. Collis

David J. Collis is an assistant professor of business administration at the Harvard

Business School and faculty adviser to the Management Consulting Club. Professor

Collis has extensive experience with consulting firms.

Approximately 100,000 people worldwide work full-time in the management consulting industry,2 generating about $25 billion in annual revenues.3 Just over half of these consultants come from the United States; another quarter come from Europe. While management consulting fluctuates with the business cycle, the industry has nevertheless grown more than twice as fast as GNP for the last decade. Such a large, dynamic industry is by no means homogeneous, however. This article describes how the management consulting industry can be segmented and identifies some important trends in the industry with attendant implications for those intending to pursue a career in management consulting.

2.1.1 Segmentation

The first, and probably most significant, way to segment the consulting industry is to compare large and small firms. Most consulting firms are small, often one-person, operations: half of all consulting firms generate less than $500,000 in annual billings, and one-third employ fewer than four people. If the typical consulting firm is small, however, the typical consultant works for a large firm: the fifty largest consulting firms in the United States account for approximately three-quarters of domestic revenue, while an estimated three-quarters of all consultants work in firms employing more than 100 professionals. This skewed size distribution reflects the low barriers to entry to this industry: anyone can hang out a shingle bearing the title "Consultant," and many former executives do just that. Corporate policies of early retirement, downsizing, and outsourcing have created both the supply and the demand for independent consultants, and, many of these small shops exist, often serving a single client. Whether these firms are attractive starting points for new consultants is debatable, since they lack both the breadth of clients and depth of support of the big firms. However, veteran consultants often end their careers in their own consulting firms.

A second and often overlooked distinction in consulting is between in-house and external consultants. While the industry definition, strictly speaking, covers only outside consultants, many large corporations have their own internal consulting arms, usually affiliated with a planning department. Internal consultants perform essentially the same functions as external consultants, while enjoying a more direct career path into line management. Contrary to the prevailing belief, only I in 1000 consultants

2 Economist survey, February 13, 1988, p. 7. 3 Data estimates are from Consultants News, June 1992, and various earlier issues.

Page 13: Mckinsey CaseBook

12

makes a direct transfer to a top executive position in a client organization and only then after many years in the consulting firm. Transfers at junior levels are common, but most of the 20% annual rate of turnover among consultants is not due to consultants being hired by clients. For those who are not convinced that consulting is a lifetime career but who want a variety of experiences and exposure to senior management problems at an early stage in their careers, albeit in a more limited number of settings-there are five times as many external consultants as internal consultants.4 Internal consulting can be an attractive option.

The third important basis for segmentation in the consulting industry is degree of specialization. While all firms provide a variety of services, most consulting firms generate the majority of their revenue from one type of work. The two main dimensions of specialization are function-for example, logistics, organization design, management information systems (MIS), or management education - and customer group or industry, for example, health care, financial services, or government.

Of these specializations, the largest is MIS consulting, which is, for the most part, the domain of the accounting firms: six of the ten largest consulting firms in the world are the consulting arms of the big six accounting firms. In the United States alone, these firms generate over $3 billion in billings annually. The second largest specialization is compensation and benefits consulting. Four of the top twelve U.S. consulting firms fall into this category, with annual billings exceeding $1.5 billion. In third place as a specialization is management/strategy consulting. This area includes the generalist management consultants, such as McKinsey, Booz Allen, and their newer first cousins, the strategy consultants, such as BCG and Bain. Although these firms are ostensibly full-line consultants, clients often find their cost structure uneconomic for consulting on functional activities such as logistics. Instead, the generalist consultants concentrate on higher value-added consulting for senior management. The other functional or industry specialists in consulting tend to be small, reflecting their origins as one-person shops run by an executive with a particular skill or industry knowledge. Understanding how each consulting firm specializes, and ensuring that it matches your interests, is therefore a vital first step in considering which firms to approach for a position.

The consulting industry no longer draws a distinction between formulation and implementation. All firms in all categories of consulting recognize that their role must involve effecting change in the client organization, and differences between them are now of degree, not of substance. Some specialists in "change management" exist, but even they would like to be involved in developing the direction of change. Similarly, although some firms may be known for a particular technique tool, such as BCG and the experience curve in the 1970s, these reputations are usually more a reflection of marketing than of a fundamentally different approach to consulting. When the publicity for a firm surrounds a particular solution to a general problem, like time-based competition for strategy, it is usually not representative of a profound difference in the type of work the firm undertakes.

4 Journal of Management Consulting, Vol. 3. Summer 1984.

Page 14: Mckinsey CaseBook

13

A last distinction among consulting firms is their degree of internationalization. Most firms now have offices or affiliations outside their home country, but the extent of non-domestic business varies substantially. Among the world's top twenty consultants, for example, one firm, Hewitt Associates, does only 7% of its work outside the United States, compared with McKinsey's 60% and the U.K. firm did 94%. However, even those firms with extensive overseas networks tend to have independent offices, so they can be responsive to local needs. This implies that working for an international consulting firm will not necessarily allow you to work overseas. You usually have to ask explicitly for an overseas assignment and often have to recruit with the overseas office in addition to the domestic office.

2.1.2 Trends

Management consulting will continue to grow, more cyclically than in the past, but still faster than GNP. The rationale for hiring consultants-to access the specific expertise needed to quickly solve a current problem-will remain and will, if anything, cover more tasks in the future as firms reconsider the costs of all their internal functions. In fact, predicting which corporate activities will be outsourced could give you a head start in identifying the next growth specialty in consulting.

The industry will also continue to move to an hourglass shape: increasingly, there is a bimodal distribution of firms into the large (annual billings in excess of $100 million) and the small (less than $5 million in annual billings). This structure results both from the ease of entry for newcomers, and from the competitive advantages of larger firms-reputation, diversified client base, and broader geographic base. While a few firms are able to break through the mid-size plateau to become recognized large players - or are acquired by other firms looking for broader scope-many bump along unsteadily at $ 10 million to $20 million in billings before falling back as the initial momentum subsides. For the potential consultant, this suggests that the key question to be answered before committing to the attractions of fast promotion at a newer rapidly growing consulting firm is, "Does it have the capability to break through to the first tier?" This question is particularly important if you anticipate a lifetime career in consulting: more than half of the consulting firms currently operating did not exist fifteen years ago, and only I% of consulting firms are more than fifty years old.

Other industry trends are the acquisitions by outsiders of consulting firms and the move toward broad scope consulting firms, under which single ownership provides a variety of consulting specialties. At least half of the top twenty firms have made recent acquisitions-three of which propelled the acquiring parties into the top twenty-and there have been more than fifty substantial acquisitions since the mid-1980s. The rationale for these acquisitions lies in the economies of scope that a broad line competitor can exploit, particularly in marketing. It has been estimated that only a third of a consultant's business comes from repeat clients, while new business, increasingly, is won in competitive bids against comparable consulting firms. As a result, about 20% of a consulting firm's costs lie in acquiring clients. If an accounting firm can leverage its audit relationship into MIS consulting, or if a strategy consulting report can recommend hiring the sister benefits consulting firm for the follow-on organization study, this expense could be substantially reduced.

Page 15: Mckinsey CaseBook

14

Unfortunately, the success of broad scope consulting firms and of outside ownership remains doubtful. Saatchi & Saatchi is the most obvious example of the failure of an outsider to build a broad scope consulting business, but even those companies like Mercer, which are still operating successfully, have yet to demonstrate the value of broad scope. Citibank tried and exited consulting, and the accounting firms are still struggling to establish a relationship with their consulting arms that peaceably compensates consultants more than auditors, maintains a wall between consulting and auditing, and yet leverages the audit relationship into consulting work.

The issue really relates to the clients' decision-making process. The purchaser of the audit - the CFO or controller - is, for example, usually not the purchaser of strategy consulting; nor is the ease of buying a range of consulting services from a single source of much value to a client when compared to the ability to choose the best specialist for a given type of work. As a result, most of the broad scope firms are essentially umbrella-holding companies for a set of independent specialists having little interaction with one another. One reason is the difficulty inherent in merging cultures. There have been, for example, few acquisitions of one strategy consulting firm by another, partly because the problems resulting from merging cultures cause the firms' major asset-people-to leave. In considering a career in consulting, I would therefore suggest that the ultimate ownership of the firm you might work for is not of great importance. Although it is true that these acquisitions are occurring, their impact on your daily activities, particularly in the junior positions, will be relatively limited. Working at Braxton (now owned by Deloitte & Touche), for example, would not be dissimilar to working at Monitor or at Braxton when each was independent.

There is, however one industry trend that will affect you: globalization. To serve the increasing global needs of clients effectively, any large consulting firm today needs a global network of offices. This network can be created by establishing alliances with overseas affiliates but is now more often achieved by setting up foreign offices. This is an expensive process that has been one reason why medium-sized consulting firms have willingly sold to outsiders prepared to make the necessary investments. Most foreign offices are currently operated with a great deal of autonomy. However, to match the globalization of clients in the future, consulting firms are themselves likely to further integrate their worldwide operations.

As consulting firms increase their geographic scope, global competition is likely to increase. Today, U.S. firms dominate the U.S. market, European firms the European market, and Asia is undeveloped both as a market for and a source of consultants. Twenty years from now there will be far more interaction among geographic markets and, I suspect, a larger Asian presence. As future management consultants, you must be prepared to learn a foreign language and to travel overseas, both to serve your clients effectively and to learn from best practice in other countries. Although this prescription is true for all executives, it is doubly true for management consultants, who must be leaders in the development of skills if they are to continue to provide value to clients.

The final trend with implications for management consultants is the continuing pressure that increased rivalry places on consulting firms to truly provide value to clients. In practical terms this will mean that consultants will have to be less

Page 16: Mckinsey CaseBook

15

formulaic than in the past. The rapid diffusion of information and techniques within the industry prevents anyone from monopolizing a concept for any length of time and means that clients are often familiar with the new frameworks themselves. The value provided by consulting firms will have to come from their ability to apply concepts and to customize them for particular client needs, not simply from their possession of a particular technology.

To meet these sort of demands, it is likely that the employee profile of consulting firms will alter somewhat. No longer will a 24-year-old MBA be able to add value simply by applying a concept the client has not seen before. Instead, consulting firms will work more closely with client management in defining and analyzing problems and in formulating and implementing solutions. This will require a more experienced consultant, more capable of understanding the manager's role, and more versed in people skills than the functional expert of the past. These senior consultants will be supported by junior "para-consultants" who can perform the mechanistic, repetitive tasks more cost effectively. Thus the pyramid structure inside consulting firms will change-on average in the large firms, one partner supports eleven consultants-as the number of very senior and very junior employee swells.

Finally, the good news is that to truly meet client demands for value for money, consulting will have to become an even more exciting, challenging, and ultimately rewarding career than ever before.

2.2 Reflections on First-Year Recruiting

By Phil Collins

Class of 1993, Harvard Business School

If investment banking was the career of choice in the 1980s, one could argue that management consulting has replaced it as the most sought-after business profession in the 1990s. As a result, obtaining summer positions in the consulting industry has become increasingly competitive.

2.2.1 Why consulting?

Perhaps the first and most important step in first-year recruiting is deciding what type of summer position is right for you, based on your interests and long-term career plans. Your time and energy will be limited, and effective recruiting will by necessity require significant focus. Don't let the herd set your priorities: make sure you understand and can explain clearly why you are interested in consulting for the summer. Otherwise, you may end up with a great summer position for all the wrong reasons-a choice you may regret in the long run as you begin planning for your full-time career. Once you decide that pursuing a job in the consulting field is a productive way for you to spend your recruiting effort, your focus will shift to the most critical step: getting an offer. As competitiveness for summer positions in consulting has intensified, it has become increasingly important for prospective candidates to expend considerable effort in preparing for the recruiting process in order to ensure that their skills are appropriately highlighted and communicated,

Page 17: Mckinsey CaseBook

16

2.2.2 Knowing the firms

One of the most difficult aspects of preparing for a consulting job search is that there are many types of consulting firms, each with its own selection criteria. An effective job search will require you to assemble a considerable amount of information about each for the firms, which will be useful in two ways: this information will assist you in determining which firms you would be interested in working for, and it will be crucial in preparing for interviews. While preparing materials and attending recruiting briefings and career fairs will be helpful, you should also take advantage of any opportunity to learn about the firms from the consultants themselves, who will give you a more detailed and realistic understanding of the firms' focuses and values. Another valuable resource not to be overlooked is your classmates who worked at particular firms before business school or who went through summer consulting programs with firms. Generally, firms differ along a few important dimensions, and it is important to understand how each firm differentiates itself Consider the following issues:

• Type of work: Does the firm specialize along functional, industry or geographic lines? Are new consultants encouraged to be specialists or generalists? At what level of the client's organization does the firm work? Does it have a very strong practice in certain specialties, or does it attempt to be strong across a number of areas? What kinds of problems has the firm worked on before, and do these engagements sound interesting?

• Focus on implementation: After developing a set of recommendations, does the firm ctively participate in implementation?

• Practice development: Does the firm have a strong commitment to developing competencies in its practice and to disseminating its expertise throughout the firm?

• Focus on professional development: What kind of resources does the firm bring to bear on problems? What is the role of a new consultant on a project? What kind of training programs does the firm have, and how does it support the professional development of its consultants?

2.2.2.1 Getting an offer Once you have a good understanding of the various consulting firms, you should be able to identify those in which you have a sincere interest, and will therefore be ready to begin pursuing a summer position in earnest. Here are some tips:

1. Understand what consulting firms are looking for. While understanding the characteristics of each firm will be helpful, most firms are looking for the same kind of people: smart, creative problem solvers whose interpersonal skills will allow them to work well in a team environment. Given the nature of the work, consulting firms are also looking for people who are energetic and have an appetite for new challenges, traits often demonstrated by a record of past achievement. Given that most students at top business schools possess all of these requirements to some degree, successful candidates must communicate their unique strengths clearly and convincingly.

Page 18: Mckinsey CaseBook

17

2. Understand clearly why you want a job in consulting. There are no experience prerequisites for getting a summer job in consulting, other than knowing why you. want it and being able to clearly communicate your conviction. Consulting firms hire people from a wide variety of backgrounds, but a major career change may require some explanation.

3. Frame your skills and experience in terms of how you can add value to the firm and its clients. Provide concrete examples from your previous experience which demonstrate that you have been a creative problem solver, that you are successful working in teams, and that you have demonstrated leadership abilities.

4. Identify your weaknesses. Look carefully at your resume and identify your weak spots. What are the one or two questions that you hope they will not ask? They will, so prepare clear and convincing answers.

5. Be yourself and be honest. While it is important to be at your best in framing and communicating your skills, it is also important to be honest and to be yourself. Getting a job at a firm full of people with whom you would not get along, or where you would not enjoy the type of work being done, is not in the best interest of either you or the firm. A couple of bad interview experiences with a particular firm should indicate that this is not a place where you would be happy spending your summer-let alone your career. Be grateful that you figured this out early, and move on with enthusiasm to the next interview.

6. Save the best interview for last. There is a learning curve in this process, and you should schedule your interviews accordingly. Experience will allow you to become more relaxed, confident and convincing.

7. Ask insightful questions. Firms will inevitably ask you at the end of the interview if you have any questions. You should have. Good questions are firm-specific and thoughtful. They should be designed to demonstrate a strong understanding of the firm and to help you gain further insight into whether the firm is a top choice for you.

2.2.3 Preparing for the case interview

Nothing causes more anxiety in first-year students trying to land a summer job in consulting than the prospect of interview cases. For most firms, interviews and cases will make or break your candidacy. While they are clearly a crucial element in evaluating prospective employees, they are not nearly as frightening as one might expect. A good case interview is no more than a discussion about an interesting and challenging business problem, and provides an opportunity to showcase your knowledge and skills. Viewed in this way, the case interview can become considerably less daunting. Firms use cases to evaluate your analytic abilities and problem-solving skills. Keep in mind that they are not looking for a "correct" answer, but instead are trying to understand how you think and how you approach problems. This is an important distinction with implications for how you should respond to case situations. I suggest keeping the following points in mind:

Page 19: Mckinsey CaseBook

18

1. Stay calm. Listen carefully and take time to think clearly about the problem before formulating a response. This is especially important because it is difficult to recover from a hasty start.

2. Take notes. The last thing you should have to worry about is remembering case facts and numbers, so feel free to take notes during the case portion of the interview. This often helps you to concentrate on the problem-solving aspect of the case.

3. Ask questions. The questions you ask are often as important as your answers in helping the interviewer understand how you think and what issues you believe are important for further clarification and consideration. If you make assumptions, state them clearly.

4. Identify the most important issues in the case up front. Try to determine what is

critical, as opposed to what is merely interesting, and develop hypotheses to explain what is driving the important issues. If you are on the wrong track, the interviewer will often interrupt you and provide additional data.

5. Develop a framework for approaching the problem. Break the problem down into its constituent parts, and approach them in a logical way rather than generating random thoughts. Give the interviewer a road map of where you are going to take the discussion: the framework is a key to understanding how you think and approach problems and illustrates your ability to think about problems in a systematic way.

6. Be flexible. Adapt your analysis to the problem, and develop a framework that is appropriate. Don't try to force every problem to conform to a generic, prefabricated and inflexible analytical framework. Each problem is unique and will require a unique approach.

7. Think causally and logically. What are the underlying causes of the case situation, and what impact have they had? Develop a clear and logical chain of reasoning and understand the linkages between key elements of the problem.

8. Drive to action. Given an understanding of the key issues, causes, and linkages, what opportunities does the client have to take actions that will improve their performance?

Keep in mind that each firm approaches cases in a different way. Some cases are long and complex, encompassing a number of issues and presenting a lot of data, while others may be much shorter or less quantitative. You may be handed pages of data and asked for your impressions, or you may have a situation described to you in a qualitative way. Some firms may ask you to analyze an industry you have worked in, while other firms will deliberately ask you about industries with which you are unfamiliar. Some cases might require microeconomic analysis, while others may rely on knowledge of first-year marketing. Most often, the cases will require integration of knowledge of a number of subjects and functional areas.

The bottom line is that case interviews have been designed so that you cannot study for them, so don't bother. Review the major frameworks developed in first-year courses, brush up on your microeconomics, and then concentrate on ways of

Page 20: Mckinsey CaseBook

19

enhancing and demonstrating your problem solving skills. One good way to do this is to practice a few mock cases with another student.

While a consulting job search will require a great deal of time and effort, it can also be a challenging, rewarding, and even fun experience. You will meet a wide variety of intelligent and interesting people, you will be challenged to think on your feet to work through complex business problems, and you will gain a broader understanding of the different firms and of consulting as a career.

2.2.4 Second-Year Recruiting: Looking for the Long-Term

By Jim McManus:

Class of 1990, Harvard Business School

First-year interviewing for summer jobs in management consulting can be characterized as an exciting, head-spinning whirlwind of back-to-back meetings, whether you are successful or unsuccessful. For many, the process consists of five or so days crammed with as many as 20 interviews. The resounding line eloquently uttered by every recruiter that becomes particularly meaningful as the second-year process draws near is, "Don't be discouraged if things don't work out for the summer. We extend more offers for permanent positions, so your chances of receiving an offer second year increase."

Fortunately, the recruiters speak the truth. In addition, the interviewing process for second-year candidates is refreshingly slower paced and more manageable than the first-year cyclone. Overall, the process of pursuing a permanent position can be broken down into three broad components: investigating consulting firms, recruiting, and decision making.

2.2.5 Investigating Consulting Firms

We often under-utilize the vast career resources provided by our schools to assist us in identifying the right career "fit" after graduation. School career centers/placement offices offer a variety of tools, including career counseling services, interview skills and resume workshops, and company-specific literature that enable us to build a knowledge base on potential employers in a matter of days. Spend some time early in the process getting familiar with the workings of your career center. When you are planning your post-graduation career, to overlook the recruiting expertise and insights that your schools have amassed over the years clearly would be to forego one of the greatest benefit of business school.

Though the information available through formal school channels provides valuable background on particular consulting firms, the second-year recruit's most important resource, by far, are classmates and friends. Speak to as many people as possible about their summer job experiences; these discussions will provide you with the most pertinent, nitty-gritty details of what life is really like at Firm X or Y. If you are particularly attracted to a certain firm, seek the perspectives of many people who have experience at that firm. You will often find that one friend's views of a summer or pre-business school experience at a firm differ considerably from someone else's at the same firm. Given the variables of personality, office location, lifestyle preference,

Page 21: Mckinsey CaseBook

20

and the nature of the projects worked on, you are sure to pick up important new insights with each informal conversation with classmates.

The focus of your information search will also differ in the second year. When interviewing with consulting firms for summer positions, the primary goal of most first-years is simply to land a job at the firm of choice, with less emphasis placed on such "details" as location, lifestyle and culture. The summer experience is a relatively risk-free way to figure out whether or not consulting in general and a firm in particular will make sense for the long-term. Second- year recruiting is for the longer term. The "details" that you were willing to live without for an 8- to 10-week summer will become critical second year, which will help you to differentiate between the many opportunities you are likely to have.

You may find it helpful to evaluate consulting firms on three broad criteria: 1) the nature of the work they do; 2) the characteristics and cultures of the firms; and 3) the nature of the career opportunities offered. In terms of their consulting work, firms (and even different offices within the same firm) differ markedly in their degree of emphasis on the following dimensions: implementation vs. strategy; industry focus; revenue- generation or cost focus, and functional specializations. Issues such as the size of a typical case team and the role of the new consultant on a team should be considered. Though every firm will highlight the collaborative nature of its client relationships, there are significant differences in policies relating to the amount of time spent at clients' offices that will have a direct effect on the amount of travel and often the level of client impact you can expect.

In assessing the characteristics of each firm, one of the most critical attributes to consider is the culture of the firm, as reflected by its employees. This is also important when looking into various office alternatives within a firm. Can you see yourself working well with the people you have met before and during the recruiting process? It is important to be honest with yourself here! You will be spending a lot of time with these folks, both in the office and traveling, and an otherwise great project can quickly become a negative experience if you do not get along well with the other team members. Besides understanding the firm's personality and values, you should get a feel for other important attributes of each firm, including the availability of international opportunities and the number (and size) of offices, and weigh these factors against your particular preferences. In addition, you should consider the size and stability of the firm's client base and its vulnerability to a downturn. What is the firm's long-term strategy, and how well positioned is it to achieve that strategy?

As a prospective candidate, you can gain tremendous insight into a firm's commitment to its people and into the career opportunities available by evaluating the critical policies of training/skills development, performance evaluations, promotion, compensation, and assistance in outplacement. Although the high starting salaries in consulting are undoubtedly attractive, especially given our high debt levels, compensation should be just one of many criteria you use in deciding which firms to pursue.

Page 22: Mckinsey CaseBook

21

2.2.6 The Recruiting Process

Management consulting firms do not wait long after the beginning of the school year before kicking off their recruiting campaigns. Though the interview season does not officially begin until later in the school year, many firms invite students to information sessions and dinners throughout the fall to introduce prospective candidates to the firms' people and practices. These "informal" get-togethers give students the opportunity to evaluate firms before getting into the more time-intensive interview process. (As a practical matter, dress codes tend to be casual for on-campus presentations and professional for off-campus events.) If you are particularly attracted to a certain firm but have not been invited to attend their fall functions, take the initiative to call one of their recruiting coordinators to express an interest in attending. You certainly have nothing to lose!

Arrangements for interviews differ by school and by firm, so it is a good idea to understanding detail your school's policies at the start of the second year. Some schools have very strict schedules, while others are more relaxed. Depending on the firm, you may need to send a cover letter requesting an interview. However, many consulting firms have "open schedules" that allow you to arrange an on-campus interview directly through your school's career services office. Again, the best advice here is to know the policies of recruiters and of your school and to work within those policies.

Like the first-year process, the second-year interviewing normally consists of three or four rounds of interviews, with the final rounds taking place at the firms' offices. Unlike the first-year schedule, however, the second-year process takes place over a period of several weeks rather than several days. In general, recruiters use the first and second rounds to evaluate a candidate's problem-solving prowess and the final rounds to determine the personality "fit" between the candidate and the firm. Although a case should be expected in most interviews, the use of cases varies widely from firm to firm and even from interviewer to interviewer within the same firm. Since many consulting companies give strategy cases, it is a pretty good idea to review the various strategy frameworks before beginning the interview season. When going through case interviews, remember one important word of advice-relax!

2.2.7 The Decision-Making Process

Most people feel that the challenge in the recruiting process is actually landing a great job at the firm of first choice, but the real fun begins if you have the good fortune of having to decide between two or more firms of similar caliber. If you do receive offers from more than one firm, refer to the selection criteria you established at the outset of the entire recruiting process, taking into account differences between the work the firms do, the characteristics of the firms, and the long-term career opportunities. If other variables are relatively equal, the issue that should weigh most heavily in the decision should be the people with whom you will be working. Make sure that you have met and are comfortable with enough people at all tenure levels of the organization, paying particularly close attention to what the junior people are saying. Professor Collis's article provides more detailed advice about how to make choices between firms.

Page 23: Mckinsey CaseBook

22

One final word of advice: make sure that there is clear agreement between you and the firms from which you have offers as to their deadlines for accepting or rejecting offers. Receipt of an offer does not allow you to extend your job search indefinitely; after all, recruiters are under pressure to firm the size of the incoming class in a reasonable period of time. Furthermore, being considerate throughout the recruiting process can only enhance your image in the eyes of the recruiting firm, and such consideration is not likely to be forgotten by employees. These firms recognize that many of the brightest MBAs do want to have happy and fulfilled personal lives outside the office and do not want to sacrifice everything for their careers.

However, you should be aware that in almost any consulting firm, if the client phones your partner and says that he or she wants to see you in Timbuktu by 8:30 am the following morning, and if you happen to have plans that evening, you are going to find it very hard to avoid canceling your plans and going to Timbuktu!

My advice about lifestyle is, then, to make sure that you understand fully what kinds of lifestyles people in your prospective consulting firm really do lead and to be sure that you would find a similar lifestyle rewarding.

2.2.8 Career Paths

Career paths in consulting and industry differ considerably. In industry, a traditional career path might be to start fairly low down in the organization and to work one's way up the corporate ladder, step by step. Though increasingly, there are firms that have job rotation schemes and fast tracks to allow them to identify and promote their best people quickly, it is still true that, in industry generally, the commitment to the firm and time scale of your progress is still fairly long term.

In consulting, the time frame can often be shorter. Promotion decisions tend to be made in the two- to three-year time frame, and many companies employ so-called "up-or-out" policies. These policies require that you develop certain skills within a defined time frame in order to be allowed to continue with the organization. If you are unable to develop these skills, you may find that the opportunities presented to you by the firm diminish somewhat rapidly!

Although an up-or-out policy may at first sound rather brutal, in practice it is not always so. From the firm's point of view, such a policy makes a lot of sense, as it allows fresh ideas to be constantly brought into the firm by new recruits. The survival of a professional service firm is quite dependent on its ability to remain at the forefront of its field, and an up-or-out is one way in which a firm ensures its ability to regenerate itself. From the employee's point of view, the policy ensures that the environment will be dynamic and that the organization will provide a constant stream of new and challenging opportunities. In most firms that have such a policy, the policy is very sensitively administered, and employees very rarely get kicked out unexpectedly. Rather, frequent feedback allows employees to judge their own position within the firm accurately. Many firms have excellent out-placement services, usually administered unofficially through contacts with alumni, and moving out is not considered failure by any stretch of the imagination.

Page 24: Mckinsey CaseBook

23

In practice, the number of people who want to continue in consulting, who feel they are at the right firm, and who are not able to do so because of the up-or-out policy is actually quite small. Most people who leave consulting firms do so of their own will, either because they decide consulting is not for them or because another tremendously exciting opportunity presents itself to them.

As a potential employee, you should inquire about the consulting firm's promotion policies. This can often be a difficult subject to raise, but my experience is that most firms are very happy to explain how their promotion policies work and would much rather you understand these up front.

2.2.9 Remuneration

Until relatively recently, consulting firms were very much the leaders in MBA remuneration. While it is still true that on average most larger firms in industry pay less than most consulting firms, the number of exceptions to this particular rule is increasing every year. Small, high technology companies are increasingly recognizing the value that MBA students can add within their firms. Even some large traditional Midwest manufacturing corporations are responding to increasing competitive pressures by becoming more aggressive in recruiting bright young management talent. In this process, some industry salaries are rapidly approaching those offered by consulting firms. Certainly, even if your objective is to pay back your MBA debt in as few years as possible, you should not rule out a career in industry.

However, if you really do want to work in industry, and if remuneration is particularly important to you (and there is every reason that it should be, given that in the words of a classmate of mine, we are all "mini-LBOs" by the time we finish our MBAs), you will have to spend more time and effort searching out the best opportunities. Another classmate of mine, who was being recruited by a major manufacturing firm, found that when she asked about the possibility of a signing bonus, she received a blank look and the reply, "What's that?" It is by no means all industrial firms that are approaching remuneration parity with consulting firms! Moreover, consulting companies tend on the whole to have very well-oiled, effective recruiting machines, so getting a high-paying job tends to be less work for the recruit.

These, then, are some of the issues involved in making the consulting versus industry decision. It is a tough choice, and in each individual situation there will be many other personal factors involved, too. Whatever decision you come to, you can be assured of an exciting, challenging experience. Good luck, and have fun!

2.3 Is Consulting the Right Field for You?

By Tim Opler

Consulting is hot! Salaries are up. And more MBA students have entered the field within the last few years than any other area. These placement numbers have caught many business schools by surprise and, today, deans and administrators are scrambling to ensure that their MBA programs offer the right type of courses for prospective consultants. At the same time, many of you are giving management consulting a hard look. I understand why! Work in consulting is stimulating and the

Page 25: Mckinsey CaseBook

24

pay can be excellent. Salary offers at top MBA schools in 1996 for consultants averaged $80,000 per year, often with significant signing bonuses or tuition relief. As of December 1996, offers have been continuing to increase (several firms are offering packages well into six figure territory after a few investment banks upped their ante).

But is consulting really the right field for you? And, if so, how should you conduct your job search? A careful examination of your own skills, values and interests is an excellent idea, particularly given the wide range of available career options.

I recommend that you commit to an ongoing and serious process of introspection and skill inventorying before marching into your next job interview. The more convincingly and honestly you can answer questions about why you are across the table from the interviewer, the better you will do. To say nothing of long-term personal happiness. After all, making a difference in a career that you enjoy is an

important part of life. Doing a good job today in finding a career that matches your values and skill set is an investment that will pay off for many years to come. It's very

easy to see the time you're planning to spend exploring careers get taken up with other more immediate priorities. It's absolutely vital that you not let this happen.

2.3.1 The Options

There are basically two career options in consulting. Generalist or specialist. Not surprisingly, specialists apply specialized process and functional knowledge to real organizations with real problems. It's great work that offers clear value to many organizations. Without doubt, the hottest area in consulting today is informational technology. This is technical stuff that offers strong productivity improvements to countless businesses in areas like client/server, sales force automation, CICS/VBASIC/UNIX. And, its why the big IT consulting shops, like Andersen Consulting, will continue to experience meteoric growth. More people work for Andersen today than do for the top five generalist firms combined. Speaking of generalist, the other option available is to work for a firm which provides a wide variety of advice designed to make enterprises run faster, better, cheaper, meaner and more efficiently. Generalist firms include well-known names such as Bain & Company, BCG, Booz Alien & Hamilton, McKinsey, Mercer and Monitor plus a growing list of mid-sized consultancies and smaller boutiques. At the same time, some of the Big Six accounting firms have made tremendous inroads into the strategy consulting business. Coopers and Lybrand, for example, has a very high quality strategic consulting unit and is managing to attract some of the very brightest students from institutions like NYU and Wharton.

In all, over 300,000 people work full-time in the management consulting industry, generating more than $30 billion in annual revenues. Just over half of these consultants come from the United States; another quarter come from Europe. The most rapid growth is currently being seen in developing economies such as Brazil and Indonesia. There can be no doubt that this industry will continue to expand rapidly over the long-run although short-run retrenchments can and will happen.

Given the scope and size of this career opportunity, it is well-worth asking where you might fit into the industry. And, of course, whether you want to fit in. Let's start by asking what skills are in demand among consulting organizations.

Page 26: Mckinsey CaseBook

25

2.3.2 The Skills in Demand

Consulting firm interviews typically involve a combination of general background questions, a case question and questions about your past behavior (the much dreaded behavioral interview approach). There are, of course, many different approaches to interviewing and, for that matter, to being interviewed. But the bottom line is that firms are screening for skills that match their needs. It is vitally important that you make every effort to understand what these skills are before you step into the interview room.

Common skills on interviewer check lists include...

Skill #1: A Passion for Ideas. You can't eat what a consulting firm makes. You can't drive it. You can't smell it. The product is an idea, an insight, a suggestion, a way of thinking. Ultimately, consulting firms are nothing more than repositories of pure human capital. This means that their most important asset has to be the ability to generate relevant ideas through rigorous thinking and careful research. Hence the frenzy to hire the best and the brightest of America's business schools. This intellectual focus of consulting is clearly important in deciding whether you would do well in the field. A good consultant has to be a great thinker with a passion for ideas. You need to be the type that does well in school and likes it. You need to enjoy problem -diagnosis, problem-framing and problem-solving.

If you find yourself struggling with the academic-side of business school, getting stuck on cases, disliking writing, but excelling along other dimensions (e.g. human interaction or entrepreneurialism) you probably should not be a consultant. You may very well get a job offer anyway. Firms may hire you opportunistically, knowing that they you can generate more value for them than you are being paid. But advancing and leading may be a different matter altogether.

I would add that firms aren't nearly as pedigree-sensitive as some seem to think. Leaders of some of the most prominent firms in the consulting profession have made it with degrees from institutions far below the top-ranked schools. Pedigree can neither guarantee one success nor condemn one to failure. For that matter, the MBA degree itself need not be necessary. A number of firms are hiring persons with other degree backgrounds (e.g. law, engineering, public administration, medicine). McKinsey, in particular, has recently been aggressive in its pursuit of attorneys, PhDs and the like. I guess sheepskin is sheepskin... And, of course, many undergraduate students enter consulting, often in two or three year programs which are expected to be followed by a stint at business school.

Skill #2: A Passion for Client Service. With all of the money being thrown around by the consulting firms these days, it can be easy to get into the profession for the wrong reason. After all is said and done, consulting is a service profession and most firms screen carefully for commitment to others and ability to excel in meeting client needs. As a consultant you will always be working to help others. Your ability to serve clients will determine your success and the prospects of your employer. While intangible, a personal commitment to excel in meeting the needs of your clients is vital to enjoying the profession. In a recent letter published by Mitchell Madison Group, a seasoned ex-McKinsey consultant put it this way: "It is only through personal excellence that this profession becomes truly enjoyable. Those who

Page 27: Mckinsey CaseBook

26

demonstrate superior skills gain personal control early in their careers. These individuals are in such demand that, at any point in time, they have numerous options to choose from. They typically become engagement managers sooner, and tend to set the pace for their teams. Through their intellectual leadership they gain respect from the clients, the partners and their teammates. In a business world where institutional loyalty is rare, the individual needs to excel and generate his or her own marketability. The result is that the institution needs the individual, not the reverse. Over the years, I have observed that unfriendly clients become attentive when listening to people of excellence because their contribution is unique. Those who achieve excellence feel great about themselves and are more likely to find the consulting experience a path to fulfillment. The financial rewards become window dressing and the high of the experience becomes the drug of first choice."

Skill #3: A Passion for People. A consultant once told me that some of the most fulfilling relationships of his life were with clients. Not because he didn't cherish his spouse and family, but instead because he had built life-long, lasting partnerships with a number of clients through repeated contact and hard work. These relationships are what can make the long hours, stressful travel and corporate frustrations encountered by consultants worthwhile. Consultants who enjoy talking to people do well. It's a field where the gregarious do well with their teammates and their clients. This isn't to say that you must be the ultimate extrovert, but you do have to connect. However you accomplish this, whether it be by charm, humor, listening or hard-work, it's vital that you enjoy, understand and communicate with clients. Consulting firm interviewers are looking for people that they'd like to work with themselves. It's only human.

So, it's an odd admixture in demand at the consulting firms. Smart, likable people who are good at helping others. Not necessarily a natural combination of abilities you might say. The screening process, of course, can vary widely and many firms are looking for a unique traits. Other characteristics in demand including understanding of specific business issues, a tolerance for ambiguity, tolerance for absolutely abusive hours, superb IT skills, personal appearance, the ability to work quickly in spreadsheets, logical thinking skills, writing skills, willingness to travel and facility with languages.

2.3.3 Landing the Job You Want

Let's suppose for the moment that you've decided that you would like to pursue a position in consulting. Moreover, let's assume that you don't yet have offers from the three firms that you truly want to work for. What then is your next step? This all depends on where you are going to school. At some institutions, all the big firms show up and will talk to you. You pick them and they pick you. But this practice is the exception, not the rule. Most students who land positions in the consulting profession do so by scrambling, hustling and working hard in the job search process.

Anne Harris, Head of MBA Career Services, at the University of Virginia's Darden School argues that the most important aspects of conducting a consulting firm career search involve preparing the right resume, networking in the profession and getting "face time".

Page 28: Mckinsey CaseBook

27

The Resume: The resume is a necessary evil in your job search. Consulting firms are looking for organized resumes that convey the skills they are looking for, solid schooling, some relevant functional expertise (e.g. engineering or finance) and a track record of successful experience. The firms are typically tolerant of "career changers" but will be looking for you to provide a coherent story about why you are changing. It's not particularly important to worry about font choice and paragraph formatting. A good interviewer is looking for experience, enthusiasm and skill.

Networking: The key to landing a consulting position is to network. Not bad since a consultant has to be a natural networker. It really helps to have others batting for you and educating you about the profession. If you indicate in an interview that you already know someone at a firm your chances of landing a position will go up dramatically. There are lots of good ways to network. It's not as hard as many seem to think. Most people will be willing to help you if you give them the chance. Sometimes the firm you want is right on campus and provides an opportunity to get acquainted at a cocktail party or other so-called "cultivation event." More likely, you will need to strike out on your own. You should contact people at the firm you are interested in who come from the same school you attended or who you are linked to in some other way. The best way to get acquainted is over the telephone.

Making The Phone Call: The networking phone call is the single most valuable weapon in your job search arsenal. You can overcome the "intimidation factor" by practicing this technique with a colleague or your friendly career services director. You want to call a consultant and let them know that you are a student with a specific interest in their work or firm. It's important to be sincere, polite, friendly and very interested in the person you are calling on. If you are on the job market now, be direct and ask for help with your job search. "Can I send you my resume?" "What are you looking for?" You might ask a series of questions about the pros and cons of the firm, the work and the life. Or, alternatively, you might ask for help with an upcoming interview. Better yet, ask for a meeting, even if only for ten minutes or so. "I'll be in New York (or wherever) next week and would love to ask you a few questions over breakfast."

Unfortunately, many networking phone calls end up with one of two negative outcomes: (1) "the person is not available", or (2) "sorry but we are not looking." If your contact is not available, ask for voice mail and leave a voice mail introducing yourself and explaining why you are calling. This is a great opportunity to get a conversation started. If you don't hear back, keep trying. A dirty, but effective trick, is to call the office in the evening. This is when the real work gets done and you'll be often surprised to hear the person you are calling pick up the phone and be willing to talk. Now, what if your contact indicates that they are the wrong person to call or that they are not looking? This is the time to ask for help networking with other people. Instead of being pushy or hanging up, what you should do is ask for names of others that you might contact in your efforts to learn about the field and locate a position.

Face Time: Ultimately, there is no good substitute for meeting someone. One of the most helpful things you can do is to get personally acquainted with consultants at firms that interest you. This may be costly, but it's almost always worth it. If you go to school outside of a major metropolitan area, you will need to visit people at the

Page 29: Mckinsey CaseBook

28

firms that interest you. It's human nature to favor those whom we know and like in the hiring process.

2.3.4 Recommended Resources

A big part of getting started on a consulting career is to survey the profession. It is very helpful to know the histories of the various firms, the values that they hold and the practices they are in. On a more practical level, it's vital to have access to current contact information and to be able to locate the firms. Below, I have listed a number of resources that might be helpful in this regard.

Ace Your Case! The Essential Management Consulting Case Workbook. From Wet Feet Press at 1-800-926-4JOB (349 Liberty Street, San Francisco, CA 94114-2953). Also try their web site at http://www.weffeet.com.

Management Consulting: Exploring the Field, Finding the Right Job and Landing It! Convergence Multimedia and Harvard Business School Publishing, 1997. On CD-ROM. Cost $39.95.

Consultants and Consulting Organizations Directory, 1996, Gale Research, Detroit, MI, (800) 877-GALE

Consultants News, Kennedy Publications, Templeton Road, Fitzwilliam, N. H. 03447, Available from the Consultants Bookstore at 1-800-531-0007 or fax, 603 585-9555.

Directory of Management Consultants. Kennedy Publications, Templeton Road, Fitzwilliam, N. H. 03447. Available from the Consultants Bookstore at 1-800-531-0007 or 1-603-585-2200.

Harvard Business School Career Guide: Management Consulting 1997. Available from Harvard Business School Publishing. Try their web site at http.//www. hbsp. harvard. edu.

So You Want to be a Management Consultant. From Wet Feet Press at 1-800-926-4JOB or 1-415-826-1750 (349 Liberty Street, San Francisco, CA 94114-2953).

Page 30: Mckinsey CaseBook

29

3 Overview of Cases

1 A case is:

1.1 Description of a business situation

1.2 A problem

1.3 Based on a real situation

2 The purpose of a case is to judge problem-solving abilities, the basis of consulting.

3 What are consulting companies looking for in a candidate?

3.1 Problem solving skills

3.2 Personal impact

3.3 Leadership

3.4 Drive / Aspirations

4 What are consulting companies looking for in a case answer?

4.1 Ability to think through problems:

4.1.1 Clear, logical reasoning

4.1.2 Curious, probing mind ⇒ candidate to engage in solving the problem

4.1.3 Ability to synthesize

4.1.4 Basic numerical agility

4.1.5 Intuitive business sense

4.1.6 Hypothesis generation ⇒ use hypothesis to drive thinking and formulate questions

4.2 Ability to quickly build working relationships:

4.2.1 Effective communicator

4.2.2 Tolerance for ambiguity

4.3 There are two parts to a case:

4.3.1 Mechanics - how you structure the case and go through the analysis. This is akin to creating an outline before writing a paper.

4.3.2 Analysis - how you think and solve problems

4.4 Both are important.

5 A good approach:

5.1 Listen to introduction carefully

5.2 Carefully think through the problem at hand

Page 31: Mckinsey CaseBook

30

5.3 Ask one or two clarifying questions, if necessary

5.4 Structure the problem, possibly with a logic tree - enables one to organize the data presented

5.5 Pick one branch to probe, develop hypotheses, ask for relevant facts, defend/refine hypotheses, probe further

5.6 Pick the second branch ...

5.7 Put it all together: try to answer the overall question

6 During the case, you want to communicate explicitly what you want to do with the case. Remember that you are trying to lead the interviewer through your problem solving approach. Writing a paper is a good analogy to solving a case. In a paper, you first order your thoughts in an outline, creating a framework to solve the case. Go over the framework at a high level with the interviewer and then plunge into one area at a time. Use interim conclusions to sum up a section of analysis before moving on to another area.

7 Time management during the interview is the interviewee’s responsibility. Suggested time allocation for a 20-25 minute interview is as follows:

7.1 The time spent structuring the problem should average about five minutes, but will vary depending on the topic and your level of comfort with the issue.

7.1.1 Plan to spend at least three minutes thinking about the problem and framing the top-level issues/questions that will need to be answered during the course of the analysis. Do not be afraid to take your time and think about how you plan to attack the case. Successful interviewees have taken up to 10 minutes to think through the issues as hand. DO NOT BE AFRAID OF SILENCE!

7.1.2 Plan to spend two minutes discussing the issues, prioritizing them and possibly eliminating some before you dive into the rest of your analysis. Remember that time is the most precious commodity that you have during the interview. Do not squander time be focusing on issues that will not highlight your abilities or will not lead to a viable solution.

7.2 Analyze the case: 15 minutes, divide the time by the number of issues to be explored based on your perception of the importance of each section.

7.3 Summing up: 5 minutes. In many ways, this can be almost as important as the time spent up front framing the analysis. This is where you can gather all of the information gathered during your analysis and present it in a logical and persuasive fashion. If not all areas were covered during your interview, you may also chose to spend a minute or two discussing further areas that you would have liked to explore and why.

Page 32: Mckinsey CaseBook

31

7.4 Finally, always provide reasons for the conclusion you are presenting, both during the analysis and the conclusion. The interviewer is not a mind reader. You must lead him down the path that you are walking.

8 Assumptions enable one to close quickly an issue. The key to using assumptions are:

8.1 Timing. Try to state your assumptions up-front. If the assumptions are invalid, the interviewer will state that they are not correct.

8.2 Delivery. Try to state the assumptions in a non-offensive manner. Bad phraseology could lead the interviewer to think that you are arrogant.

Page 33: Mckinsey CaseBook

32

4 Different Types of Cases

1 Classic Cases

1.1 Purpose

• Assess broad functional skills

• Calibrate big picture perspective

1.2 Types

• Impact of a consolidating industry on a client company

• Should a company add capacity?

• How should a client react to a new competitor?

• Should a client enter/exit a new/old market?

2 Special Cases

2.1 Purpose

• Assess the comfort level with ambiguous problems

• Evaluate creativity

• Evaluate raw analytical horsepower

• Test poise under pressure

2.2 Types

• Why are manhole covers round?

• How many golf balls are there in this state?

• What do you think interest rates will do next year?

Page 34: Mckinsey CaseBook

33

5 Frameworks for Cases

5.1 General Models

1 Financial Frameworks - for profitability cases you should explore cost and revenues

1.1 Income Statement

Net Income

- Cost of Goods Sold (COGS)

Labor

Materials

Overhead

Delivery

Gross Margin

- Depreciation

- Sales General &Administrative (SG&A)

Operating Profit

- Interest Expense

Earnings Before Taxes (EBT)

-Taxes

Net Income

1.2 Balance Sheet

Assets

Cash

Investments

Accounts Receivables

Inventories

Property, plant & equipment

Intangibles

Liabilities

Accounts Payables

Other Short Term Debt

Long-term Debt

Other Liabilities, Reserves

Page 35: Mckinsey CaseBook

34

Shareholders Equity

- Common Stock

- Retained Earnings

2 Porter’s Five Forces

- Suppliers

- Potential Entrants

- Buyers

- Substitutes

- Industry Competition

- Complements – the forgotten force

3 Business System

3.1 R&D

- Product Development

- Innovation

- Responsiveness

3.2 Manufacturing

- Cost

- Quality

- Speed

- Supply

3.3 Marketing

- Pricing

- Product

- Place

- Promotion

3.4 Distribution

- Cost

- Channel

4 Issue Tree

4.1 Do not use this framework in an interview without practicing it a few times before hand.

4.2 Top-level identifies the highest level issues that need to be answered to solve the problem. Use MECE (Mutually Exclusive and Collectively Exhaustive) to ensure that all issues are covered.

Page 36: Mckinsey CaseBook

35

4.3 Break each level down into parts that are more manageable.

4.4 Focus on most important branches or components first.

4.5 An example of an issue tree is provided with the China Factory case.

5 Framework for a Zinger case like “How many golf balls in Albuquerque?”

5.1 Supply chain

5.1.1 Raw Materials

- Who makes the plastic needed for golf balls?

- Obtain an estimate of quantity of material supplied and work from there.

5.2 Demand side

- Who purchases golf balls?

- How many golf balls do they need each year?

5.2 Additional Models

5.2.1 The Four C’s

The four C’s stands for customer, competition, cost and capabilities. This model is intended to ask the critical questions in understanding the core business of an organization. The model is more of a back-of-the envelope sketch than a detailed analysis. Filling in these categories can be a first, cursory step in understanding a given company or industry. Although this model is unlikely to produce revolutionary insights, it may help in defining a business by breaking it down into very basic components and looking for conflicting elements. The model is difficult to use with diversified companies and interests.

5.2.2 Economics

Students should review the basics of economic theory. Many cases, especially the strategic ones, have a strong backing in basic economics so knowing the fundamentals will give the student a strong base from which to work. Some of the more relevant concepts include:

5.2.2.1 Supply & Demand

The Supply Curve -The higher the price of a product or service, the greater the quantity of the item that will be produced, all other things being equal. Supplier will be willing to make more available (i.e., supply). Conversely, the lower the price of a product or service, the smaller the quantity producers will be willing to make available. Please remember that as the supply of one product increases, the supply of another product will decrease. (We live in a world with finite resources but infinite demand.)

Page 37: Mckinsey CaseBook

36

The Demand Curve - The lower the price of a product or service, the greater that demand for the quantity consumers will be willing to purchase (i.e., demand), all other things being equal. Conversely, the higher the price of a product or service, the smaller the quantity of goods consumers will be willing to purchase.

Price

Quantity

Demand

Supply

5.2.2.2 Law of Diminishing Marginal Utility

This concept or economic "law" states that the level of demand or "satisfaction" derived from a product or service diminishes with each additional unit consumed until no further benefit is perceived, within a given time frame.

5.2.2.3 Law of Diminishing Returns

This concept suggests that although additional units of labor may contribute to increased productivity in absolute numbers, each additional unit contributes relatively less than the preceding unit to productivity.

Price

Quantity

Cost ofInputs

Value ofOutput

5.2.2.4 Comparative Advantage

Comparative advantage states that it is in the best interest of a nation to import an item from another nation when it cannot produce the item as inexpensively. The concept of comparative advantage goes a step farther, contending that it may be to a country's advantage to import goods from other nations even though they may be able to produce the goods less expensively at home. This is based upon the premise that not producing the item in favor of producing another item which offers better production efficiencies will ultimately benefit both countries (see also economies of

scale).

Page 38: Mckinsey CaseBook

37

5.2.2.5 Elasticity of Demand

The degree to which demand for a product or service can be altered by a change in price indicates the extent of the elasticity of such demand. For example, a person who seeks to purchase a particular brand and model of automobile may decide to shop competitively from dealer to dealer for the lowest price. This would characterize demand that is elastic in nature. However, there are circumstances where the level of demand is not altered by a change in price. For example, a diabetic will probably be willing to pay as much money as he or she has to buy insulin, the medication that would sustain that individual's life. In this case, the demand is inelastic.

Cross Elasticity: Percentage change in quantity demanded of one good in response to a 1 percent change in the price of a related good.

QxPy

x

y

x x

y y

y x

x y

EQP

Q QP P

P QQ P

%

%

= = =

/

/

EQxPy is positive if the two goods are substitutes

EQxPy is negative if the two goods are complements

Supplier Elasticity: Percentage change in the quantity supplied in response to a 1 percent change in price.

S

S S SEQ Q Q

P P / P= =

%

%

/∆

5.2.2.6 Economies of Scale

Economies of scale exist when the average cost (AC) declines as output increases, over a range of output. If AC declines as output increases, so must the marginal cost (MC). (Marginal cost is the cost of the last incremental unit of output.)

The relationship between AC and MC can be summarized as follows:

MC<AC = Economies of scale

MC=AC = Constant returns to scale

MC>AC = Diseconomies of scale

The shape of the cost curve is U shaped. The generally accepted explanation for this is that AC initially declines because fixed costs are being spread over increasing output and then eventually increase as variable costs increase (see law of diminishing marginal returns). The minimum efficient scale (MES) is the minimum level on the average cost curve. Economies of scale are not limited to manufacturing. Marketing, R&D, and other functions can realize economies of scale.

5.2.2.7 Economies of Scope

Economies of scope exist if the firm reduces total production costs by increasing the variety of activities it performs. Whereas economies of scale are usually defined in terms of declining average cost functions. It is more customary to define economies of scope in terms of the relative total cost of producing a variety of goods together in

Page 39: Mckinsey CaseBook

38

one firm. Economies of scope may be achieved by "leveraging core competencies" across multiple business activities. For example, it may make economic sense for a manufacturer of tape to get into the business of manufacturing note pads with adhesive backings as there are commonalties in the two businesses at many points along the value chain.

5.2.2.8 Learning Curve

The learning curve refers to cost advantages that flow from accumulated experience through lower costs, higher quality and more effective pricing and marketing. The magnitude of learning benefits is expressed in terms of a "progress ratio." The ratio is calculated as the unit cost after doubling cumulative production divided by the previous cost (C2/C1). A ratio of less than one suggests that some cost savings due to learning is taking place. The median appears to be approximately .80. This implies that for the typical firm, a doubling of cumulative output is associated with a 20% reduction in unit costs.

5.2.3 4P's

Kellogg’s Philip Kotler developed this model. It stands for product, price, placement (i.e., distribution channels), and promotion. These are the four critical dimensions in marketing any product (or service).

5.2.4 Value Disciplines

Fred Wiersema and Michael Tracy of CSC Index, Inc. have developed a set of strategic foci called the value disciplines (Harvard Business Review, January-February 1993, pp. 84-93). The disciplines are:

Operational excellence - Provide customers with reliable products or services at competitive prices and delivered with minimal difficulty or inconvenience, with the goal of leading the industry in price and convenience (e.g., Dell Computer).

Customer intimacy - Segment and target markets precisely and then tailor offerings to match exactly the demands of those niches, combining customer knowledge with operational flexibility to respond quickly to almost any need (e.g., Home Depot).

Product leadership - Offer customers leading-edge products and services that consistently enhance the customer's use or application of the product, thereby making rivals' goods obsolete (e.g., Nike).

Companies which push the boundaries of one value discipline while meeting industry standards in the other two gain an advantage that other competitors find hard to match.

5.2.5 Porter’s Five Forces

Michael Porter's Five Forces model analyzes the various competitive pressures at work in a given industry. The results indicate the overall industry attractiveness (i.e.. ease of making a profit), as well as the strength and influence that each of the

Page 40: Mckinsey CaseBook

39

competitive pressures have on the firms participating in the industry. The following is a brief discussion of the five components.

Industry Competitors (Internal Rivalry) - Often, the most powerful of the five forces is the competitive battle among rival firms which are already present in the industry. The intensity with which the competitors are jockeying for position and competitive advantages indicates the strength of the influence of this force.

Potential Entrants – This force measures the ease with which new competitors may

enter the market and disrupt the position of the other firms. The threat that outsiders will enter a market is stronger when the barriers to entry are low or when incumbents will not fight to prevent a newcomer from gaining a market foothold. In addition, when a newcomer can expect to earn an attractive profit, the barriers to entry are diminished.

Threat of Substitutes - The competitive threat posed by substitute products is strong when policies of substitutes are attractive, buyers' switching costs are low, and buyers believe substitutes have equal or better features.

Supplier Power- Suppliers to an industry are a strong competitive force whenever they have sufficient bargaining power to command a price premium for their materials or components. Suppliers also have more power whenever they can affect the competitive well being of industry rivals by the reliability of their deliveries or by the quality and performance of the items they supply.

Buyer Power - Buyers become a stronger competitive force the more they are able to exercise bargaining leverage over price, quality, service, or other terms or conditions of sale. Buyers gain strength through their sheer size and when the purchase is critical to the seller’s success.

Benefit of Complements – This is considered a sixth force that is not directly captured in Porter’s model. This force is the opposite of the Threat of Substitutes. When the economics are promising for a complementary product, there is a spillover effect on the primary product.

Industry

CompetitorsRivalry among

existing firms

Suppliers

Substitutes

Potential

Entrants

Buyers

Five ForcesComplement

Page 41: Mckinsey CaseBook

40

5.2.6 "Star" Diagram/Organizational Analysis

In doing an organizational analysis, one should consider all seven components of the organizational unit. Vision should define Strategy. Strategy determines Structure and Decision Support Systems that are required to make the organization function. The Reward Systems must reinforce what you are trying to accomplish strategically and the Human Resource Systems must select, recruit and develop the personnel the organization needs to accomplish its objectives. Corporate Culture must reinforce all seven components.

Strategy

Reward

Systems

Human

Resource

Systems

StructureDecision

Support

Systems

Vision

Organization Culture

Performance

Problems arise when these seven components do not reinforce one another. For example, managers will have trouble if they are in a decentralized structure while information and planning systems are centralized. When considering change, all seven components must be considered. If one component is changed, it is most likely that the other components will have to be changed to be consistent with each other.

5.2.7 The BCG Growth-Share Matrix

The BCG Growth-Share Matrix provides a valuable framework that enables us to identify and evaluate the company's products relative to market share and the extent to which the market, as a whole, is expanding or contracting. The model can also be utilized to analyze a portfolio of companies held by a single organization by classifying them within the matrix; each as independently held businesses.

Products or categories businesses are as follows:

Star — A product with high market share in a high-growth market; every mother's prayer.

Problem Child (also called "Question Marks") — A product with low market share in a high-growth market; mother is concerned because her child is not growing as anticipated. Another perspective is that the manager shouldn't be quite so concerned if the product has carved out a little niche that is impervious to the competition; maybe slow yet consistent growth isn't so bad.

Cash Cow — A product with high market share in a low-growth market. Since the cow is generating milk (i.e., cash), the marketer may elect to "milk the cow dry," so to speak, accelerating cash flow and, not coincidentally, the product life cycle.

Page 42: Mckinsey CaseBook

41

Dog — A product with low market share in a low-growth market. In this sense, "dog" is certainly not "man's best friend." Rather, it is analogous to a "bomb" (i.e., something that fails miserably) or to a "lemon" (i.e., something that is defective or undesirable). Therefore an astute business manager would want to drop a “dog” from the product line, unless there are some extremely important overriding issues that outweigh the products market performance.

Cash Cow

StarProblem

Child

Dog

MarketShare

Profitability

Hi

Low

LowHi

5.2.8 Value Chain

A business manager must understand the internal relatedness of the many activities involved in the production of a product or service. Every business unit is a collection of discrete activities ranging from sales to accounting that allow it to compete. Michael Porter calls these activities “value activities.” It is at this level, not the company as a whole, that the unit achieves competitive advantage.

The value activities are grouped into nine categories, as indicated in the exhibit below.

Primary activities create the product or service, deliver it to the market, create a demand for the product, and provide after-sale support. The categories of primary activities are inbound logistics, operations, outbound logistics, marketing and sales, and service.

Support activities provide the input and infrastructure that allow the primary activities to take place. The categories are company infrastructure, human resource management, information systems, and procurement.

Value chain analysis is useful in discerning possible synergies among various units of an organization (e.g., shared procurement). Value chain analysis is also helpful in determining which value activities are best outsourced and which are best developed internally. Finally, value chain analysis provides a structure that provides great insight into the flow of activities that lead to the creation and distribution of a particular product or service. (e.g., What value is added to the manufacture and sale of gasoline at each point in the value chain, and by whom?).

Page 43: Mckinsey CaseBook

42

Company Infrasructure

Procurement

Information SystemsHuman Resource Management

Inbound

LogisticsOperations

Outbound

Logistics

Marketing

&

Sales

Services

Support

Activities

Primary

Activities

Value Chain

5.2.9 Generic Strategies (Porter)

Michael Porter suggests that business strategies can be classified as pursuing cost leadership, differentiation, or focus. Each of these strategies is described as follows:

Overall Cost Leadership: Here the business works hard to achieve the lowest production and distribution costs, so that it can price its products lower than its competitors and win a large market share. Firms pursuing this strategy must be good at engineering, purchasing, manufacturing, and physical distribution of the products. Texas Instruments is an excellent implementer of this strategy. The problem with this strategy is that other firms will usually emerge with still lower costs (from the Far East, for example) and hurt the film that rested its whole future on being the lowest cost producer. The real key in this strategy is for the firm to achieve the lowest costs among those competitors adopting a similar differentiation or focus strategy, and remaining so in the long run.

Differentiation: Here the business concentrates on achieving superior performance in an important customer benefit area valued by a large part of the market. One example is if the company strives to be the service leader in its industry, the highest quality producer, the style leader, the technology leader, and so on; but it is hardly possible to be all of these things. The firm cultivates those strengths that will give it a competitive advantage in one or more benefits. Thus the firm seeking quality leadership must make or buy the best components, put them together expertly, inspect them carefully. This has been Canon's strategy in the copy-machine field.

Focus: Here the business focuses on one or more narrow market segments rather than going after a large market. The firm gets to know the needs of these segments and pursues either cost leadership or a form of differentiation within the target segment. Thus, Annstrollv Rubber has specialized in making superior tires for farm-equipment vehicles and recreational vehicles and keeps looking for new niches to serve.

According to Porter, those firms pursuing the same strategy directed to the same market or market segment constitute a strategic group. The firm that carries off that strategy best will make the most profits. Thus, the lowest-cost firm among those pursuing a low-cost strategy will do the best. Porter suggests that firms that do not pursue a clear strategy - “middle-of-the-roaders" -- do the worst.

Page 44: Mckinsey CaseBook

43

5.2.10Strategic Types (Miles & Snow)

Miles and Snow have divided strategic options into four categories (in contrast to Porter's three Generic Strategies). A firm can only pursue one of these strategies at a time, but it is common for a company to shift from one strategy to another as its situation, and the industry, changes.

Defender—Those firms that have a leadership share of the market will often concentrate on staving off the competition, moving to erect as many barriers to entry as possible. They are closely related to Porter's Low Cost Producers, leveraging their advanced position along the learning curve and their name recognition to maintain a superior market position.

Reactor— Such companies are second-movers, letting others show them the way to success. They react to changes in the market and moves of their competitors and so must maintain flexibility. While this strategy may be profitable in the short run, its long-term value is questionable.

Analyzer— Analyzers pick apart the market very carefully looking for niches and demand/supply gaps. This strategy is akin to Porter's focused companies. These firms are not necessarily innovators, but instead concentrate their efforts in very carefully and narrowly defined efforts.

Prospector— These firms are the first-movers and the innovators. This is a high-risk strategic avenue to follow, but those who are successful can change the way the game is played and create very strong competitive advantages.

5.2.11Other Key Concepts

5.2.11.1 Reengineering

Popularized as Business Process Reengineering (BPR), reengineering refers to breaking down business processes and reinventing them to work more efficiently, cutting out wasted steps and enhancing communication. Business processes are often replete with implicit rules that hamper the way in which work should truly be done. Further, processes are often viewed as discrete tasks, a habit that prevents management from making frame breaking, cohesive change. Reengineering is defined by Michael Hammer and James Champy in Reengineering the Corporation

as "the fundamental rethinking and radical redesign of business processes to achieve dramatic improvements in critical contemporary measures of performance such as cost, quality, service, and speed."

5.2.11.2 Total Quality Management (TQM)

TQM refers to the practice of placing an overriding management objective on improving quality. Whereas TQM is more of a philosophy than a specific strategy, the stated objective is often "zero defects" or “six Sigmas.” A higher level of quality is linked to increased customer satisfaction and thus leads to the ability to charge a higher price at what is often a lower cost. It is important to ensure that the added benefit from incrementally increasing quality outweighs the added cost associated

Page 45: Mckinsey CaseBook

44

with the quality improvement effort. TQM was initially limited to the manufacturing sector but has more recently been applied effectively to service businesses as well.

5.2.11.3 Key Success Factors

Essential success factors are those factors that are most critical in determining a firm's ability to survive and prosper. Attributes of essential success factors are the following:

- Management can influence them;

- They impact the overall competitive position of the firm in the industry

- They are an interaction of characteristics of an industry and each firm's strategies.

- A firm must supply what customers want and survive competition from other firms

Therefore, management should ask:

- What do customers want?

- What does the firm need to do to survive competition?

Essential success factors are those factors that lead to the answers to the above questions. For example, for wood products, essential success factors are owning large forests and maximizing the yield of those forests.

5.2.11.4 Core Competencies

A concept popularized by Professors Gary Hamel and C.K. Prahalad, core competencies provide potential access to a wide variety of markets, make a significant contribution to the perceived customer benefits, and are difficult for competitors to imitate. The classic example of a company that has effectively leveraged its core competencies is Honda, which has gained a competitive advantage in numerous product markets through its focus on leveraging its skill at making engines.

5.2.11.5 Vertical Integration

In some industries companies find it advantageous to integrate backward (towards their suppliers) or forward (towards their customers). Vertical integration makes the most sense when a company wants greater control of a channel that has major impact on its product cost or when the existing relationship involves a high level of asset specificity.

5.2.11.6 Just-in-Time (JIT)

The goal of JIT production is a zero inventory with 100% quality. In other words, the materials arrive at the customer's factory exactly when needed. JIT calls for synchronization between suppliers and customer production schedules so that inventory buffers become unnecessary. Effective implementation of JIT should result in reduced inventory and increased quality, productivity, and adaptability to changes.

Page 46: Mckinsey CaseBook

45

5.2.11.7 Fixed vs. Variable Costs

Variable Costs (VC): The costs of production that vary directly with the quantity (Q) produced: these costs generally include direct materials and direct labor cost.

Fixed Costs (VC): The costs of production that do not vary with the quantity (Q) produced: these costs generally include overhead costs.

Semi-variable Costs: The costs of production that vary with the quantity (Q) produced, but not directly. (Typically, these are discrete costs, such as the cost of adding new production capacity when Q reaches certain levels.)

Break-even Point: Break-even analysis is a managerial planning technique using fixed costs, variable costs, and the price of a product to determine the minimum units of sales necessary to break even or to pay the total costs involved. The necessary sales are called the BEQ, or break-even quantity. This technique is also useful to make go/no-go decisions regarding the purchase of new equipment. The BEQ is calculated by dividing the fixed costs (FC) by the price minus the variable cost per unit (P-VC):

BEQ = FC/(P-VC)

The price minus the variable cost per unit is called the contribution margin. The contribution margin represents the revenue left after the sale of each unit after paying the variable costs in that unit. In other words, the amount that "contributes" to paying the fixed cost of production. To determine profits, multiply the quantity sold times the contribution margin and subtract the total fixed cost.

Profit = Q x (P-VC) - FC

5.2.11.8 Net Present Value (NPV)

The NPV is a project's net contribution to wealth. Net present value is the present value (PV) of all incremental future cash flow streams minus the initial incremental investment. The present value is calculated by discounting future cash flows by an appropriate rate (r), usually called the opportunity cost of capital, or hurdle rate. Ct represents the cash flow at time t. (Ct can be negative, as in the initial investment, Co.) The NPV is calculated as follows:

NPV = Co + Cl/(l+r) + C2/(l+r)2 + ... + Ct/(l+r)t

If the net present value of the project is greater than zero, the firm should invest in the project. If the net present value is less than zero, the firm should not invest in the project.

5.2.11.9 Pareto Principle (80/20)

The Pareto Principle refers to the situation in which a large amount of the total output comes from a small amount of the total input. This phenomenon is typified by the "80/20 rule" which states that 80% of the output comes from 20% of the input. Typically, a Pareto analysis is conducted to determine the areas on which management should focus its efforts. For example, 80% of total downtime on a production line is attributed to two out of the ten manufacturing steps. Alternatively, 80% of a company's profits may be generated by 20% of its product lines.

Page 47: Mckinsey CaseBook

46

6 Dos and Don’ts

1 Helpful Hints:

1.1 The goal of the case interview is not to crack the case but to demonstrate “how you think.”

1.2 Listen carefully to everything that is said during the interview. Do not lock yourself into your answers, interviewers will drop hints regarding the case throughout the interview process. This is an area where one can demonstrate teamwork. The interviewer will look to see if you build on the information provided.

1.3 Ensure that you understand the problem at hand. Do not assume anything! Use questions to clarify issues and to gain a complete understanding of all the problems that need to be addressed.

1.4 Take your time; do not answer any question unless you have thought through your answers fully. Once the problem or case is presented to you, do not feel compelled to answer immediately. Rather, ask for a few moments and think out your approach. During that time, brainstorm to get all your thoughts on paper. Then, carefully assess them and order them into a logical format. This will serve as the structure of your analysis.

1.5 HAVE FUN WITH THE CASE! Engage yourself and get excited about the case. The interviewer has probably spent a few months working on the problem and consulting firms are looking for people who enjoy solving problems. They also want consultants who are dedicated to their client and client’s problem. You can demonstrate this by being engaged.

1.6 Do not ask for every piece of data; the more you prompt for data without assimilating what you have already been presented with, the more confused you could become. The interviewer possesses tons of data or will make it up, if need be. Better to use the bull’s eye approach to data gathering, as opposed to the shotgun approach.

1.7 Remember that some material may be extraneous.

1.8 Individual pieces of data can often be combined to draw a conclusion about part of a problem.

1.9 Think then speak. Thinking aloud leads to rambling. Practice phrasing statements clearly and succinctly. However, do not error on the side of being silent either. The interviewer is not clairvoyant. He will not understand your thought process unless you explicitly state to the interviewer.

1.10 Keep the interviewer informed of where you are going and check to see if you are on track. For example, one could say: “I intend to purse..., do you believe this is worthwhile.”

Page 48: Mckinsey CaseBook

47

2 Top Ten Things not to say during an interview:

2.1 I need a lot more data.

2.2 What are you looking for?

2.3 Is this a marketing case?

2.4 I cannot believe this is a real case. Are your clients really that stupid?

2.5 This case has no answer.

2.6 There is obviously one answer to this case.

2.7 I do not believe you; you are trying to confuse me.

2.8 I would like to concentrate totally on the political implications of this situation.

2.9 I cannot solve this case because I have not had Corporate Strategy yet.

2.10 This case sounds exactly like what we did in Managerial Accounting.

3 Don’ts

3.1 If the interviewer suddenly asks you to name the three critical drivers to the industry, DO NOT ANSWER RIGHT AWAY! Think about it and come up with five drivers, then order them from the most significant to the least significant. Then say something like this: “Well I have thought of five significant drivers, the three most important are...”

Page 49: Mckinsey CaseBook

48

7 How to Prepare for the Consulting Interviews?

Step 1 Attend Case Interviewing Workshops by leading Consulting Firms

Step 2 Research the firms and know their differences.

Helpful places to look for info:

• Attend Presentations

• Quick database search to get any recent news on the company

• Use Wet Feet Press Guides

Step 3 Practice the cases with a partner

Page 50: Mckinsey CaseBook

49

8 General Interview Questions

1. Why consulting?

2. Why do you want to work for this firm?

3. Tell us about your skills.

4. Tell us about your resume.

5. Tell us about your weaknesses.

6. Tell us about your previous industries.

7. Describe a problem you encountered in a work environment and how you handled it.

8. If you do not get into consulting what will you do?

9. Give me an example of where “you dropped the ball.”

10. Give me an example of where you did something unpopular and had to stand up for yourself at work.

11. Where do you see yourself in five years?

12. Describe a situation where you had to present orally to an important group of people.

Page 51: Mckinsey CaseBook

50

9 Sample Cases

9.1 China Products Division

9.1.1 Issue

The CEO of a large diversified building products company has asked us to help her examine the operations of her china products division. China products include tubs, toilets and urinals. Specifically, he wants to know if he should approve a $200 million capital expenditure for new manufacturing facilities.

The company is one of seven producers in the United States; the largest producer has a 20 percent share; our client is number three with a 15% market share.

• Prices for the client’s products have been flat in the recent past.

• The two largest competitors appear to earn a small return; our client is break-even.

• The largest competitor has just announced plans for a major modern plant.

What issues must the client consider?

9.1.2 Possible Solutions

Probable points of clarification:

• Will the planned investment lower operating costs? (Yes, but not substantially. The major reason for the investment is that the new process will result in a better finish.)

• Does the company rely on a limited source of raw materials? (No, materials are easy to obtain.)

“Minimum” - level answers

• Market size/growth:

What has been the industry’s growth in units?

Is the growth linked to housing starts?

• Competitive Position:

How much overcapacity exists in the industry today?

What are the competitor’s relative cost positions?

• Market segmentation:

How is the market segmented (e.g., residential vs. industrial vs. commercial)?

Are there different price points by market or within markets?

“Better answers”

Page 52: Mckinsey CaseBook

51

• Customer-buying factors:

Do customers demand a full-line supplier (e.g., with other building products)?

Is any significant portion of sales to centralized customers (e.g., Sears)?

• Barriers to entry/exit:

What is the minimum-size for a new plant?

Are most plants fully depreciated?

Generally, how expensive is entry/exit? Has there been a history of change in the industry players

• Manufacturing:

Do the plants produce other products that contribute to overhead?

Are there ways in which costs can be substantially lowered?

“Outstanding answers”

• Marketing:

How rational has pricing been in the industry?

Have competitors ever announced capacity expansions before and then not implemented them?

Are there opportunities to rationalize the product line in order to increase revenue?

Does the new finish that will result from the investment “pay for itself” with higher prices?

• Competitive Position:

How important is the product line to each competitor?

Are the products sold in combination (with each other, or with other products such as fittings)?

Would exiting the business affect the sales, profits or costs of other business units?

Are there advantages to plants being located in specific places due to high transportation costs? If the competitor’s new plant is built, will other manufacturers drop out of the market?

• External environment:

Is the regulation of the market significant?

Are there changing demographics that will affect demand?

Logic Tree Approach

Page 53: Mckinsey CaseBook

52

Level One Question: Should I invest in $200 million plant?

Level Two Questions: Is there a sizable profitable market?

Is there a better use for funds other than the current proposed investment?

Are there significant market threats?

Do I have production advantages over competitors?

Level Three Questions: Is there a sizable profitable market?

Do I have good relationships with distributors?

Does the market have enough unsatisfied demand for new capacity?

Can I sell at the right price?

Can the market be segmented?

9.2 Healthcare Company

9.2.1 Issue

A large healthcare company has decided it is interested in substantially increasing the size of its operations. Its goal is to double total sales and profits in less than two years.

As a consultant brought in to assist them, what would you do?

9.2.2 Possible Solution:

What is the current scope of operations? In what areas of healthcare does the company deal? What is its current market share in these areas?

What plans has the company already considered is currently considering?

What is the competitive nature of the industry? What would be the effect on sales and profits of reducing prices/margins?

What potential is therefor expansion by acquisition? Do they have the financial capability? Do potential targets exist?

A suitable solution will depend upon the answers to the above questions.

A business can increase profits by:

- Increasing sales

- Increasing prices

- Cutting costs

However, if the company's margins are found to be consistent with industry norms, it would seem unlikely that either increasing prices or cutting costs represent feasible

Page 54: Mckinsey CaseBook

53

methods by which to double sales & profits, particularly if the company operating in a moderately competitive environment.

This leaves only sales increases, which could be achieved by: selling more of the current products to current customers selling new products to current customers selling current products to new customers selling new products to new customers The suitability of these options will again depend on the particular environment. In the particular example of this case, it turned out that only selling new product to new customers via some form of diversification could hope to achieve the company goals.

You should then consider the potential for increasing sale by means of diversification through acquisition or joint venture. The relative benefits of each will depend on financial resources as well as the existence or otherwise of suitable targets.

9.3 Electronic Joint Venture

9.3.1 Issue

A large microelectronics manufacturing company is considering participation in a cooperative project which will receive 50% government funding. The company currently uses several hundred millions of dollars worth of microelectronics every year, the cost of which is growing steadily in many of the company's products. Although the company already has a subsidiary that does some microchip fabrication, the capacity of this subsidiary is small and its technology is relatively old. It is interested in developing application specific ICs (ASICs). The government project would more or less allow the company to define an individual project within the broad category of process technology, CAD tool and equipment development

What are the important considerations to making this decision?

What factors are important for success in the ASIC business?

Should the company get involved? If so', with whom?

9.3.2 Possible Solutions:

What is an ASIC? When are they used for and for what reasons?

(An ASIC combines the functions of many semiconductor components on one chip, decreasing size and weight and increasing speed and reliability. Assembly costs are reduced, but development costs are higher.)

What is the market outlook for ASICS?

(High growth. currently about 15% of all ICs are ASICS & this figure is expected to rise above 35% in the next 10-years. The IC market itself is expected to grow at over 10% per year too. Important demand segments are entertainment electronics and automotive applications.)

How is the industry structured? How big and how competitive is the industry? Who are the important competitors? What are their main business lines? How vertically integrated are they? Are there already joint ventures and alliances?

Page 55: Mckinsey CaseBook

54

(There are about 150 firms producing ASICs, of which only a handful are of any size, although there are powerful commodity-chip manufacturers. In contrast to the commodity-chip manufacturers, most of the top ASIC producers are in fact American. There is currently only one producer of ASICS in a similar line of business to the company under discussion in this case but this competitor has much more sophisticated semiconductor production capabilities. Strong demand has meant that price erosion is not currently an issues in the ASIC industry However, some strong alliances already exist in the industry and the possibility of converting old DRAM production facilities into ASIC facilities may well lead to great overcapacity when the next generation of memory chips takes over.)

Are there significant barriers to entry? What are the expected costs of entry?

(A state-of-the-art production facility large enough to meet the company's needs would cost around $250 million. In addition, considerable effort would be required to find the staff with suitable expertise. Also, firms whose core business was semiconductors would have significant technology and R&D advantages.)

How is the ASIC business different from that of standard ICs? Consider typical production volumes and hence the differences in fixed costs and design costs per unit produced.

(Typical ASIC production volumes are much lower, therefore need to find some way to reduce some of these costs e.g. using CAD to shorten design cycles.)

What are the client’s current purchasing habits?

(Currently purchase nearly all microelectronics through one subcontractor, who delivers high value sub-assemblies.)

Do potential joint venture partners exist?

9.4 Television Cable Company

9.4.1 Issue

Three years ago a venture capital company purchased a cable TV system that had access to 2MM households in the southwest. The VC firm was attracted by the extremely large subscriber potential (2MM households) and potential for considerable return. Despite their best efforts, they have failed to turn a profit in the past three years. You have been hired to determine if they can turn a profit or if they should sell.

9.4.2 Possible Solutions

- Analyze current revenue and cost structure

- Analyze the market potential of the area

- Analyze the competitive situation/ substitutes

- Provide recommendations

Cost Revenue

Page 56: Mckinsey CaseBook

55

Fixed costs associated with lying cable Subscribers monthly fees

Debt associated with fixed costs Subscribers special services - movie channels

Maintenance of the cable system

Information provided as soon as these cost/revenue drivers are uncovered:

The fixed costs are extremely high due to the distance between cities in the system. The debt and maintenance costs are also higher than systems in major metropolitan areas. The current systems is only at 43% capacity (# of subscribers) vs. a 63% industry average.

Assumptions:

High fixed costs are overwhelming the current revenues

The current subscriber rate is too low. Why? Moreover, can it be fixed?

Market Assumptions:

Based on the low subscriber rate, I would assume the population is less likely to watch television perhaps because of income or lifestyle issues.

A: Actually they watch more television than the average.

Does the cable system offer what they enjoy watching.

A: Yes.

Competition:

If consumers are watching television, but not our cable system, there must be a strong competitor in the market. What options do our consumer have?

A: In addition to the three network stations, there are eleven independent broadcast stations in the area.

Is the reception from these independent stations strong?

A: Yes, very.

Are the stations offered free of charge?

A: Yes.

Overall Assumptions:

The low subscriber rate (revenues) cannot overcome the high fixed costs.

The subscriber rate is low due to the high number of competitive stations available to our consumers (supply and demand problem).

Recommendations:

Determine if there are consumer needs not being met by the independents that could be provided by our system and worth paying for. If not, sell.

Page 57: Mckinsey CaseBook

56

9.5 Magazine Sunday Supplement

9.5.1 Issue

A major magazine publisher (not unlike Time Warner) is thinking about publishing a "Sunday supplement" for insertion in and distribution through metropolitan newspapers. They have hired you to determine if they should proceed or not.

Additional Information:

There are currently two major Sunday Supplements: Parade and U.S. Weekly

They are distributed in over 90% of the US’s newspapers (combined)

A newspaper can only insert and distribute one Sunday Supplement

They are offered to the newspapers free-of-charge

9.5.2 Possible Solutions

Can we turn a profit by publishing this supplement?

How? -Revenue potential, costs, competitive response

Does it fit with our current publishing strategy?

Can we turn a profit?

Revenue Potential (Assumptions):

Major sources of revenue is the advertising revenue

Question: Can we expect to gain revenues from our existing advertisers?

A: You tell me-

Can you explain the format of the supplement?

A: Typically cheap paper, low quality editorial, light reading gossip, modem day folklore

Assumption: Our current advertisers (for Time) would not be interested in this format.

Cost Assumptions:

Fixed cost of supplement set-up

Editorial, printing/paper, distribution

Internal and external sales force - (gaining ad revenues and newspaper acceptance)

Assumption: There are few publishing synergies with our current publications.

A: True

Competitive Assumptions:

The competitors are deeply entrenched - 90% penetration.

Displacing a competitive supplement would require costly incentives to the newspapers.

Page 58: Mckinsey CaseBook

57

Current newspapers use the supplements in order to publish low quality editorial without disparaging their product offering.

Key Issues:

Based on these assumptions, turning a profit would be difficult due to the large upstart costs and the strong competition for advertising revenues and newspaper acceptance.

Strategic Fit Assumptions:

The poor editorial content associated with these supplements may disparage the publisher’s current product offering.

Recommendations:

Based on this information and these assumptions, we would not recommend proceeding with the supplement until potential advertisers were committed and newspapers demonstrated an interest in accepting the supplement. (Even then, we would recommend publishing under an alternate brand name).

9.6 American Express Charge Card

9.6.1 Issue

American Express has faced strong competition from new credit cards entering the market. They are considering dropping the $50 annual fee. What are the "economics" of such a decision and should they drop the fee or not?

9.6.2 Possible Solutions

Determine how American Express makes money.

Evaluate the pros and cons of dropping the annual fee.

Make a recommendation

Revenue Drivers - Assumptions:

$50 annual fee multiplied the number of members.

No additional revenue from consumers because they pay-off monthly.

Receive 1 % of the transactions from retailers who honor the AMX card.

Key issues:

If the annual fee is dropped, AMX loses ($50 x # of members).

To overcome this loss, they have to increase the revenues from consumer purchases (1% from the retailer)

Is it likely that current cardholders will spend more per year if the annual fee is dropped?

A: Not likely. They would still have to pay off their balance every month.

Page 59: Mckinsey CaseBook

58

Therefore, the only way to increase revenues from consumer purchases is to increase the # of AMX cardholders

Assumptions:

Number of current cardholders = 4% of the U.S. population (Just a guess):

- 250MM x 4% = 10MM current cardholders

$50 x 10MM = Annual loss of $500MM by dropping the fee.

Current percentage revenue: 10NM members x $1000 annual purchase (avg.)

[10MM x (1000 x 1%)] = $100MM (Estimate of current percentage revenue)

Key Question:

Can we attract enough new members (without a fee) to offset a $500MM loss?

Each new member contributes $10 (1% of $1000 annual purchase).

(500MM/$10) = 50MM new members are needed

50MM new members is equivalent to 20% of the population (gut check)

Assessment/ Recommendation:

Based on these assumptions, increased membership equivalent to 20% of the population is probably not likely. Do not drop the fee.

May want to consider varying the fee (sensitivity vs. new members)

9.7 Television Cable Company

9.7.1 Issue

You have been hired by the CEO of a department store that has numerous locations in a major metropolitan area. She needs to increase the store's earnings over the next year and has requested your help.

Relevant Information:

20 locations in the metropolitan and surrounding suburban areas (they are present in every shopping mall).

The population growth of the city is flat

Overall, store revenue has declined slightly

They recently hired a consulting firm to streamline the back-room costs

How can you help?

9.7.2 Possible Solutions

Revenues have decreased for a reason

The streamlined costs may have caused revenue to falter

The revenue per store may differ - why?

Page 60: Mckinsey CaseBook

59

Increased competition?

Different consumer buying trends?

Start with Cost/Revenue Drivers:

Costs:

CGS

Personnel/ OH/ SG&A

Inventory holding costs, levels

Cost of debt

Other?

Revenues:

# of people shopping

Amount of purchase - $$

Frequency

Prices

Others?

You learn there is nothing drastically different (overall), so you turn to the individual store level.

Questions:

Are certain stores more profitable than others are?

A. Yes.

Do the higher performing stores have any common characteristics such as size, product mix, consumer demographics?

A: Yes, suburban stores are more profitable than urban stores. No, the product mix is the same at all stores. Yes, the demos are different by store.

Assumptions

The product mix may be more suitable and more profitable for suburban stores

The competition may be lower in the suburban areas (turns out not to be true)

The income level may be higher in the suburban areas

Product Mix:

What products are most profitable?

A: Appliances, tools, TV, Stereo, Jewelry - big-ticket items.

What products are less profitable?

A: Clothing, shoes, household items - low ticket items

Store by Store Sales/Demo's:

Do suburban Stores sell more big-ticket items?

A: Yes

What do the urban Stores sell?

Page 61: Mckinsey CaseBook

60

A: Clothing, household items, minor appliances

Are the demographics better suited for the mix in the suburbs?

A- Yes, higher income.

Assessment

Due to the identical product mix at each store and the varied profitability by item, suburban stores are outperforming urban stores. Hence, the urban stores are hindering earnings.

Potential Recommendations:

Re-configure the product mix by store (no sense holding excess inventory)

Assess the impact of the urban stores and determine the ramifications of closing them.

9.8 Credit Card Division of Bank

9.8.1 Issue

Our consulting firm has been retained by a major bank to help improve the profitability of their largest credit card offering. Their card (in the same class as a Visa or MasterCard) provides average returns in comparison to the industry, however, our client believes it can become more profitable. You need to analyze the situation and make recommendations.

9.8.2 Possible Solutions

Opportunity to decrease costs or increase revenues - analyze drivers

Opportunity to vary the annual percentage rate or the annual fee

Benchmark competition for opportunities

Analyze cost and revenue drivers:

Costs Revenue

Marketing, SG&A, Personnel – Can not change Annual fee - currently $50 (Could change)

Bad credit, theft, etc. - Cannot change Annual percentage rate = 14% (Could change)

Other costs – Can not change Merchant fee = 1.5% (Can not change)

Key Issues:

Cannot affect the cost structure, therefore have to increase revenues.

Only revenue variables available are changes to the annual fee and APR.

Competition:

Page 62: Mckinsey CaseBook

61

Interviewer tells you it is a very competitive environment - "move on".

Assumption:

Customers use the card differently, there may be different customer segments based on the balance held, how quickly balances are paid off and the "need" for the card.

Case Interviewer suggests there are three distinct categories:

1. Pay-off in full every month

2. Hold small debt for short periods of time

3. Hold heavy debt for long periods of time (basically pay-off the interest) - 80% of our revenue

He/She then asks how you would tailor card services to each of these groups:

Recommendations:

Pay-Off in Full Each Month

Hold Small Debt for Short Term

Hold Heavy Debt Long Term

Charge high monthly fee

Provide numerous services

(detailed reports, little kudos)

Increase the APR slightly

Decrease the annual fee

Waive the annual fee

Increase their credit limits

Cash back programs, points

Access to case advances, etc.

Key Issues:

These heavy debt card holders are the key to our profitability, it is imperative to get them to sign up for the card (no annual fee), use the card (cash back, point systems) and run up debt (automatic credit limit increases).

Note to Case Interviewer

As soon as the interviewee had identified the important drivers of revenue and cost, the focus of the case was shifted to customer segmentation and tailored services for each segment.

9.9 Movie Rental Business

9.9.1 Issue

Our client is a major entertainment company on the West Coast. One of their divisions is a leading home video retailer. During the late '80's and early '90's this division had a great run -opening 4,000 stores and realizing considerable profits. In the last two years, both growth and profit have declined substantially. You have been brought in by the CEO to assess the situation and provide recommendations.

Background: Our client's division is not unlike a chain of Blockbuster video stores. The majority of their business is in movie rental with a much smaller portion in sales.

Page 63: Mckinsey CaseBook

62

9.9.2 Possible Solutions

Start with a simple: (Profit = revenue - costs) structure

Analyze the competitive situation

Analyze the "substitution" factor - how else are consumers getting movies?

Costs: Revenues:

Cost of the new movies: (Actually decreased) # of rentals: (decreased, traffic down)

Overhead: (No change) Price of rental: (No change)

SG&A: (No change) Sale of rentals: (decreased)

Leases, other: (No change) Accessories: (No change)

Key Learnings:

Costs have actually decreased, but not enough to offset the decreased store traffic.

Competitive Assessment/ Substitutes: (List potential causes of decreased traffic)

New movie stores: (No real change)

New In-home sources - cable on demand: (Potential for future but no real current affect)

Sales of movies for home use and collection: (Sales have increased dramatically)

[Once the important issues have been identified, the interviewer describes the changing industry.]

I. When division was growing, it could buy excess numbers of the new releases to satisfy customer demand. Later, they would send the excess copies to the new stores as part of their "library" of existing tapes. With fewer new stores opening, this is no longer an option therefore fewer new releases have been ordered. 2. Recently, the studios have allowed new releases to be sold through warehouse stores (Wal-Mart) at the same time they are made available to the rental retailers. Thus, many of our customers are purchasing rather than renting. In addition, when customers rented a new release, they quite often rented an existing tape from the library (additional lost revenue)

Based on this industry outlook, what would you recommend for the division?

Provide a recap:

It appears as though the major issue facing the division is a reduction in store traffic for new releases. This is mainly due to the sale of these same releases through alternate channels. How can we regain store traffic or offset the rental loses?

Recommendations (these are just a few of the options considered):

Develop new, more convenient locations - kiosks, pick-up/delivery

Develop pricing/bundling formats combining new releases with existing movies

Page 64: Mckinsey CaseBook

63

Offer "rent to buy" programs - rent the first time, then have option to purchase

9.10 Auto Service Stores

9.10.1Issue

A successful chain of Canadian auto service stores (Autoland) has entered several markets in the United States in hopes of duplicating their success in America. The stores offer two services: 1. Retail sales of auto parts for customers who prefer to perform their own maintenance. 2. A service center for fixing any automobile problem, from an oil change to new transmission.

Since entering the U.S., Autoland has experienced $50MM in revenue with loses of $20MM. The owner is considering pulling out of the United States. You have been hired to determine if they can improve their performance or if they should exit the market.

9.10.2Possible Solutions

Analyze the competitive situation

Analyze the market potential/ customer segments

Competitive Situation:

What is the competitive situation in Canada?

A: We are the major player (few local stores)

Are we providing the same services in Canada as in the U.S?

A: Yes

Do we have strong competition in the U.S?

A: Yes, a national chain of stores in the exact format as Autoland exists in the U.S. They copied our Canadian format and have about 10 locations in every major city. They are very profitable in all cities including our U.S. markets.

Assumptions: Due to size, I would guess they have superior buying power over Autoland in the U.S. Is this true?

A: No, we have the same cost structure due to our presence in Canada. Assumption: The market has potential due to the competitor's performance. Key is to determine why they are out-performing Autoland.

Autoland Capabilities:

Assumption: We actually have two businesses under one roof, is one more profitable than the other?

A: In Canada - no. However, in the U.S. we are profitable in retail sales and losing heavily on the service center. Are the costs associated with each side of the business different?

A: Yes, the service center is much more expensive to operate, we have to pay mechanics and have high fixed costs

Assumption: We are profitable in retail, but losing in service. We attract the wrong consumer.

Page 65: Mckinsey CaseBook

64

Market/ Customers:

Autoland provides two services, are the customers for each service different?

A: Yes. The customers that shop for retails parts typically have lower to middle incomes and are

trying to save a few dollars by performing their own maintenance. The customers who utilize the service center have higher incomes and no interest in fixing their own car.

Assumption: We are attempting to attract two distinct customer segments. Are we doing this successfully?

A: We are not sure, how would you help us determine if we are?

Factors:

Marketing. A: We do the same as the competition

Pricing. A: Identical to competition

Location. A: Different, we located in the inner cities to save money on leases.

Where is competition located? A.- Between the inner city and the suburbs (on the border)

Assumptions/ Recommendations:

Our location is great for the retail sales business, but prohibits heavy use of the service center due the distribution of income between the inner city vs. the suburbs.

In new markets, locate between the lower and upper income areas to attract both segments.

In existing markets, move, or drop the service business and retain the profitable retail portion

9.11 Sports Franchise

9.11.1Issue

A wealthy woman, Mrs. Wentworth, who is worth millions, wants to invest in either the Cleveland Cavaliers (a basketball team) or the Cleveland Indians (a baseball team). She is indifferent between choosing between the two teams, but would like to maximize the profit from the investment. Which team should she buy?

Additional Facts

1. She wants to own a sports team: however, the interviewee must recognize that there are other investments that she should consider.

Page 66: Mckinsey CaseBook

65

2. The purchase price is the same for both franchises. Let us say that there is a $100 million price tag.

3. The financing terms available to Mrs. Wentworth are the same, regardless of the team she chooses.

4. The Balance Sheets of both teams are the same in terms of liabilities and assets.

5. Neither franchise owns the stadium. The interviewee must ask for this information to be given credit.

6. Ticket Prices on average are $25 for basketball games and $6.50 for baseball games.

7. The number of basketball and baseball home games is 41 and 81, respectively.

8. The stadium capacity is 20,000 for basketball games and 54,000 for baseball games.

9. Utilization on average is 70% for both sporting events.

10. The interviewee should explore other streams of revenue but are irrelevant (important streams are - licensing of apparel, concessions and parking) for purposes of this analysis. It is important that interviewees recognize these as possible sources of value to the investment.

11. The key is to figure out whether ticket prices can be raised. The interviewee must figure out a model to answer this question. Possible sources of data - historical sales patterns, comparable data for other franchises in other cities and overall market demand. After exploration of this issue, it turns out that they cannot be raised.

12. Examine if 100% utilization can be reached. Research shows that Cleveland residents are indifferent. Do not give this information away, make the candidate probe for it.

9.11.2Possible Solution

Everything cancels out except that baseball has more seats available. Thus, the baseball investment has a potential for greater revenue.

Equation = # of seats * # of games * stadium capacity * utilization

9.12 Durable Goods Distribution Case

9.12.1Issues

SETUP

A large durable goods manufacturer (i.e. washing machines)

• Sells products directly to customers (80% of sales)

• Has six wholly owned subsidiaries that stock and sell products (20% of sales)

Page 67: Mckinsey CaseBook

66

What changes would you make to the organization to increase its value?

FACTS TO FIND

• The market will not respond to advertising (i.e., the firm cannot create demand pull)

• Consumer purchase frequency is constant (i.e. buy one new every six years).

• Consumers buy primarily based on lowest cost product.

• Parent manufactures all equipment (parent’s manufacturing is not the scope of this case).

• Subsidiaries maintain all functions (were once bought by independent businesses, then bought by parent).

• Accounting

• Order Processing

• Sales

• Management

• Finance

9.12.2Possible Solutions

Would it be more efficient to provide central functions from parent, then diversified functional operations?

Maybe, but how would you measure that value?

Study each subsidiary's functions and determine efficiencies to be realized. For example, we could determine the number of labor hours for a transaction for the sub and parent. Plot labor/units on vertical, units on horizontal. If parent is higher, centralize the operations.

Problems with this approach?

Eliminating sales function and order processing at sub may reduce flexibility and hurt customer service?

Maybe, but how would you measure this problem?

Page 68: Mckinsey CaseBook

67

Interview clients and determine the importance of flexibility and personalized sales.

9.13 Business Forms Case

9.13.1Issue

OVERVIEW

A manufacturer of business forms wants to increase profitability by using pricing as a competitive weapon. The product line includes credit card receipts, multi-copy sales receipts, warranty papers, invoice forms, etc.

The CEO has hired you to assess and make business recommendations, if possible. You have ten minutes to interview the CEO before she leaves for a meeting. What will you ask? What do you need to know? What is important? Can you make recommendations?

HELPFUL INFORMATION (not to be offered unless asked)

• MARKET: Market share in the Business Forms business is 4%. The market is fragmented with many small local printers. The largest player has 5%.

• MANUFACTURING: The firm is a “world class” manufacturer. The company has four regional plants (Midwest, East Coast, West Coast & Texas). The firm has modern facilities and good cost controls already in place. The firm uses JIT, SPC and other cutting edge manufacturing techniques. An internal study indicates that the firm is the low cost producer in the industry.

• PROFIT CENTERS: Manufacturing and sales are profit centers. Transfer prices negotiated between manufacturing and sales are in place. The sales division sets the prices to customer.

• DISTRIBUTION: Extensive information system is in place. The system handles over 100 daily orders. The UPS delivers most orders within two days for standard items. Custom orders shipped between one to three weeks depending on complexity and quantity of the order. The company does not own delivery trucks.

• SALES FORCE: A two hundred person sales team is in place that focuses on important accounts. The sales force is paid on commissions. A salesperson receives 25% of all revenue above a standard cost. Contacts primarily via telephone, supplemented with occasional visits. All sales are handled through the sales force.

• MARKETING: The firm only uses direct mail fliers and catalogs. Client survey reveals that the company is known for the “high quality, wide product range, excellent return policy and its ability to fulfill any order.” The firm is able to deliver larger orders faster than competition when asked for rush delivery.

Page 69: Mckinsey CaseBook

68

• COST STRUCTURE: Not important. However, the materials cost is less than the shipping cost or fees charged for customized work.

Possible Solution

One possible solution is to use price as a competitive weapon, but not necessarily to increase market share by lowering price. The firm should exploit rush order customers by raising prices on less price sensitive customers. Use “Time Based Competition” to increase profitability.

Use ABC inventory analysis to get a “better” standard cost and empower the sales force to maximize revenue. The firms should constantly monitor standard costs because of volume fluctuations.

9.14 “High-End” Pots & Pans Company Case

9.14.1Issue

OVERVIEW

The CEO of a Pots & Pans manufacturer hired you to solve her problem:

• Foreign competitors are gaining market share

• Products are barely profitable at market prices

Pots & Pans is considered to be of premium quality, sold primarily in sets and is expensive. According to the CEO, a typical customer is a newly wed who places pots & pans on the registry at Hudson’s or Marshall Fields.

You have ten minutes to interview the CEO before she leaves for a meeting. What will you ask? What do you need to know? What is important? Can you make recommendations?

HELPFUL INFORMATION (not to be offered unless asked)

• MARKET: Market share in overall market is 10%. The market share in high-end pots & pans is 35%.

• MANUFACTURING: The firm has one plant in North Carolina

• DISTRIBUTION: Products are sent to local warehouses and then to six regional warehouses and than to the departmental stores. No products are shipped directly from the factory. The company and not customer pay shipping costs.

• RETAIL OUTLETS: The product is carried in high-end department stores. Each store carries one set of each product line for display and one set for inventory purposes. The reason for the low inventory levels is that the product is bulky and expensive to store. Department stores demand quick replacement upon sale.

• MARKETING: The firm uses little advertising. The firm has a three person sales staff that works with department store buyers. Department store buyers know the

Page 70: Mckinsey CaseBook

69

company and its products well. Little discussion or negotiation required. Repeat sales equal almost 100% of the business.

• COST STRUCTURE:

Materials - 30% Labor - 20% Var. O/H - 10%

Fixed O/H - 20% Shipping/Packaging - 20%

9.14.2Possible Solutions

The six regional warehouses were setup in the 1950s when long distance delivery times were slow and costs were high. With the advent of FedEx, the cost of delivering products quickly decreased. Eliminating warehouses leads to O/H savings that are far greater than the increased shipping costs. Inventory and outdated inventory would also decrease.

Distribution is a trade-off between cost and service level. The higher the service level, the lower the cost (more inventory pools, warehouses and shipments). Therefore, you need to ask where the inventory is being held. It turns out that stores, since they sell so few of these pots and pans hold no inventory and thus require next-day replenishment after a sale. The next thing you need to know is where the warehouses are located, since the closer they are to the stores the cheaper distribution costs will be. Your client has six warehouses - two in Charleston, two in Philadelphia and two in LA from which he services the whole country.

A quick way to solve this case is to realize that if stores require next day service from these six warehouses, the only way they can do this is by shipping overnight at a premium rate. You can save them a bunch of money by closing down a few warehouses and shipping everything from the plant in Charleston by UPS (after negotiating a volume rate discount). This can be confirmed by asking for the annual sales, which turns out to be 10,000 units. When you divide this into the $1 million distribution cost you discover that they are pay $100 to deliver a pan to the store. Beat this figure and you have earned your exorbitant fee.

9.15 Paper Products Manufacturer Case

9.15.1Issue

OVERVIEW

Your company makes legal pads, notebooks, and other standard paper products. Your firm uses a sixty-year-old plant in Gary, Indiana that has been producing paper for 60 years. The firm purchases 100,000 tons of white 20lb paper to produce its product lines.

Page 71: Mckinsey CaseBook

70

You are the purchasing agent responsible for buying the white 20lb paper. Due to recent developments, you have a new option to buy from Brazil, instead. Currently, you buy from a US company in Oregon. Your company values long-term supplier relationships. The Brazilian firm is offering to sell at $.35 a square yard compared with the $.38 a square yard from the current supplier. The firm will consider switching if it can gain a long-term cost advantage.

How do you determine if the Brazilian company has a long term cost advantage over the US company? What would you need to know? Why would it be important? What must be considered? How would you find out what you need to know?

HELPFUL INFORMATION (not to be offered unless asked)

Possible Items you would need to know!

• Cost Structure

• Facility age, technology used, amount invested

• Level of integration (does the firm own its own forests?)

• Control over key suppliers (wood or chemical suppliers)

• Shipping/Distribution costs

• Quality of Brazilian paper over US paper

• Sales volume

• Company and Industry capacity

• Capacity Utilization

• Political stability in Brazil

• Exchange rate fluctuation and ability to hedge

• MANY MORE!

How to find out what you need to know!

• Use Lexis/Nexis , ABI Inform

• Ask Brazilian and US company for information and study the annual reports

• Ask important suppliers for information.

• Examine Brazil through Brazilian governmental reports

• Look for analyst reports on the industry

BONUS QUESTION

A price war continued until the Brazilian company stopped lowering the price. The firm stopped lower its price once it reached $.25 a square yard. Why did the Brazilian company stop lowering its price?

Page 72: Mckinsey CaseBook

71

• Realized that they were losing the price war

• Reached variable cost level (incremental business)

Found more profitable use for capacity, maybe another customer.

9.16 Pianos

9.16.1Issue

This case is one of those “are you kidding, why would you ever want to ask that in an interview” cases, but -- it was asked, and similar cases always seem to pop up. The case question is, “How many pianos do you estimate there are in the United States?” (Similar cases involve American Express cards, gasoline stations, etc.) In this type of case, the right number is not necessarily the right answer for the interviewer. Like all cases, methodology is key.

9.16.2Possible Solutions

Along with a basic framework methodology, certain “commonly known facts” should be in your hip pocket when going into case interviews. One of these facts is the approximate population of the United States. This fact can serve as your starting point for cracking this case.

1. Split the population (~250 million) into households. Make an assumption about the “average household,” say three, and come up to about 84 million households.

2. Now, split the number of households into income quartiles.

Quartile # Households

Upper 21 million

Mid-Upper 21 million

Mid-Lower 21 million

Lower 21 million

3. Assign a percentage to each quartile to calculate the number of households with a piano (assume households usually do not have more than one piano each).

Quartile # Households % With Piano # Pianos

Page 73: Mckinsey CaseBook

72

Upper 21 million 20% 4.2 million

Mid-Upper 21 million 10% 2.1 million

Mid-Lower 21 million 5% 1 million

Lower 21 million 0% 0

Using this methodology, the “answer” to the case would be 7.3 million.

Better Answer: You have just estimated the number of pianos in homes in the United States. For a “better answer” to the question, you should state that schools, music halls, stores need to be considered as well. Be careful how you word this, the interviewer could very well say -- Well then, how would you come up with those numbers?

Now...

Now that you have been able to calculate the number of pianos, the interviewer may choose to expand on the case. (The number itself does not matter so much as the approach.) For instance, “given the number just calculated, how many piano tuners do you think there are in the United States?”

The solution to this question can be structured very similar to the one above.

1. Assign a “number of times tuned” to each of the income quartiles. Assume that the upper quartile tunes their piano once every year, the next quartile once every three years and the next quartile once every ten years. This will give you the following:

Quartile # Households % With Piano # Pianos # Tunings

Upper 21 million 20% 4.2 million 4.2 million

Mid-Upper 21 million 10% 2.1 million 0.7 million

Mid-Lower 21 million 5% 1 million 0.1 million

Lower 21 million 0% 0 0.0 million

2. This tells you that five million pianos need to be tuned each year. Assume a piano tuner can tune four pianos a day for 250 day a year, or 1000 pianos a day.

3. Using this methodology, the number of piano tuners that you come up with is about 5,000.

9.17 Coke vs. RC Value Chain

9.17.1Issue

There is very little set-up necessary for this case. The case is used simply to test the interviewee’s “assumption” skills and reasonable hypotheses. The interviewer does not really need to provide a great deal of detail for this case to be used effectively.

Page 74: Mckinsey CaseBook

73

The interviewee is given a piece of paper with the following representation of Coca-Cola’s value chain.

R&D Manufacturing Distribution Marketing

OHD Margin

Syrup Bottling “Base” DC

Local DC

$.05 $.15 $.10 $.05 $.20 $.25 $.10 $.10

Price = $1.00

Given Coca-Cola’s value chain, the interviewee is asked to formulate the value chain for a secondary manufacturer (use RC as an example).

9.17.2Possible Solutions

One approach to solve the problem would be to start at the end of the value chain with the price of RC. The interviewee may want to start here because this may be a “known” element (been shopping lately?). Now that the easy part is over, the next step is to assign percentages of the price to each portion of the value chain. For this part, the interviewee is expected to look at the percentage that Coca-Cola applies, and make reasonable inferences as to how Coca-Cola differs from RC and what effect this will have on the weighting for RC’s value chain.

Reasonable assumptions might be made about the following issues:

• Can RC afford to fund as much R&D (as a percentage of price) as Coca-Cola? Is RC a company which wants to be first in with a new product or a fast follower?

• Are in-house syrup production and bottling costs (%) really going to be different between companies?

• What type of distribution system would RC have compared to Coca-Cola? RC is a local brand so the assumption might be made that RC can deal only through the local DCs and do not have a large “base” DC.

• Does RC really do a lot of marketing by themselves or does it simply “ride the coattails” of Coca-Cola’s marketing?

• If RC has similar machinery requirements, do they have to spread their OHD over fewer products / less volume? RC’s percentage could very well be higher than Coca-Cola.

The candidate should follow this line of “assumption and inferences” until the percentages are in place. Then the interviewee can simply apply the percentages to each portion to arrive at a cost.

The interviewer can now ask if anything looks strange about the value chain or if you would suggest a different way for RC to be doing business. (The interviewer can then judge whether the assumptions are reasonable or whether the candidate sounds like he has never even seen a bottle of Coca-Cola before.)

Page 75: Mckinsey CaseBook

74

One possible answer to this (think back to Crown Cork & Seal) is for RC to start their value chain at a later stage in the process. If the syrup production and bottling costs are too large of proportion of their costs, they could consider buying syrup from someone else and use an outside bottler.

Again, this case, like most others, has no “right” answer. The interviewer will just be looking for the reasonableness of your approach and assumptions. Good luck!

9.18 Fertilizer

Issue

Your client is an agricultural chemical company. They produce fertilizer, which is a cyclical industry that is currently in a downward cycle. Lately, Russia has been selling fertilizer at a very low price. The firm’s plant in Louisiana blew up costing them $600 million in damages. The plant produced $2 billion of product revenue with $100 million in profits per year. The CEO resigned in shock and the new CEO has called you in to determine what they should do.

HELPFUL INFORMATION (not to be offered unless asked)

• Fertilizer uses a simple manufacturing process -- perfect commodity product

• Farmers are the primary customer of the product

• International markets are growing at a high rate relative to domestic demand

• The firm is the low cost producer in the industry

• They have a strong sales force in place

• Transportation costs are in line with the industry average

• Customers say it is cheaper to buy their fertilizer in LA and ship it themselves. In other words, the final price charged to the farmers is very high compare to the industry

Possible Solution

• Transfer pricing (from manufacturing/production to distribution to sales) is adding significant additional costs to the product. Therefore, the product is becoming less competitive in the market.

• Although they are the low cost producers, they are being priced out of the market by this additional cost structure.

• The sales force may not be adding the value that the company thought/needs in order to be successful.

Page 76: Mckinsey CaseBook

75

9.19 Airplane Manufacturer

9.19.1Issue

You are consulting to a CEO of an airplane manufacturer. In the last couple of years, you have gone from being number one in market share to number two. In addition, another company has announced that it will be entering the business and is presently tooling up its plant. As a consultant, what are the concerns your client might face? What additional information might you want to find out, and what recommendations would you make?

9.19.2Possible Answers

As a consultant, you are concerned with three essential items:

1. The condition of the airplane manufacturing industry,

2. Why the firm has lost market share and,

3. How to prevent the new entrant from stealing market share.

The airplane industry's demand is a function of travel among two classes: business and leisure. Business travel increases as a result of market globalization. Leisure travel increases with growth of middle and upper classes. Business travelers are primarily insensitive to price, leisure travelers are very price sensitive.

The current competitor; a comparison:

It turns out that the competitor's planes are cheaper to operate because they are more fuel-efficient. The consultant should ask as a strategic question whether the firm is interested in the manufacture of more fuel-efficient planes. The answer would depend on the future of oil prices. The consultant should also consider whether it might be better to try to compete on the basis of price, safety and service.

Prevention of a new competitor gaining share:

Key: creation of barriers to entry. Long-term contracts are pre-emptive. High concern, on the part of purchasers for a proven safety record of accomplishment.

9.20 Mysterious Audiocassette Market

9.20.1Issue

Your client is the manufacturer of audiocassettes. They have hired you to figure out why they have been experiencing an alarmingly poor sales year. They want you to determine the problem, and then provide a possible solution.

Information to be divulged gradually

• Mature market: 5-6 major players.

• Client used to have a steady 30% market share, (second largest in industry). Now the firm has a 44% share.

Page 77: Mckinsey CaseBook

76

• Your client offers a full range of audiocassettes -- from low bias to high bias/metal.

• Your client is also using the most sophisticated and quality driven cassette manufacturing techniques.

• The firm has been losing sales reps yet loyal reps claim that sales are at record high levels for the year.

• The firm historically targeted two consumer groups -- older, middle income enthusiasts and high school rock 'n roll stereophiles.

• Recently your client has been losing younger target market customers.

• The firm has traditionally managed its relationship with retailers well. However, the firm has recently lost several major accounts due to its inability to move (the firm's) products.

Possible Solutions

The combined market characteristics of sales decline and increased market share suggest that competitors are abandoning this market due to a new and better substitute technology (the compact laser disk, for example.)

Your client’s historically flat market share suggests brand loyal customers. Moreover, your older target market is loyal -- perhaps less likely to switch to the new technology in the short run. Assuming (1) that your client wants to be a provider of this new technology and (2) has the capacity to manage a primary supplier position in its traditional line of business -- short-term. The firm should target the older customers as well as other segments that are less likely to switch over to CD's, for the long-term. The firm should consider new sources and production necessary to exploit this new demand. The firm should also explore the opportunities and constraints of developing or acquiring the new technology.

9.21 Windmill

9.21.1Issue

You produce a windmill with an accompanying electric generator (generator harnesses the power produced by the windmill). The windmill costs you $10,000 to manufacture. How much are your customers willing to pay for it?

9.21.2Possible Solutions

Porter's five forces dictate that industry rivalry, potential substitutes, and supplier/buyer power need to be assessed to determine the market price. This could

Page 78: Mckinsey CaseBook

77

be an appropriate start. To narrow it down, let us assume competition and demand/supply levels are far beyond your capacity.

We must therefore examine other components. The $10m cost is irrelevant. You have no idea what this product is worth to anyone. Assessing the value of the benefits of the product is perhaps the next step. The closest substitute to the windmill is probably utility produced electricity. Therefore, inquire how the electrical utilities measure and charge for the electricity they provide. Convert the windmill's output along these terms and assert a cost/benefit estimation of how much potential customers would be willing to pay for it. Other considerations upon which to discount the value might be reliability, maintenance, etc.

9.22 Bank of Luke

9.22.1Issue

Mr. Check is the Director of Retail Lock Box Services for the Bank of Luke, a medium-sized Midwestern bank. The Retail Lock Box Department consists of 100 clerks and eight managers and Supervisors. Each year, in addition to handling of retail lock box transactions, the Department generates $1.5 million of fee revenue processing retail credit card and mortgage payments ("items") for 15 commercial accounts. The bank has many other commercial accounts that use other companies for their item processing. In fact, the Bank recently lost the item processing business of one of its largest accounts to Vader Inc., the largest item processor in the U.S.

The item processing industry has undergone dramatic changes in recent years. Types of items processed include credit card, mortgage, and utility payments (checks), airline tickets, and coupons. In the past, these items were usually processed by the issuing company (e.g., airlines would process their own tickets) or by bank item processing departments like the Bank of Luke's. At banks, the processing of payment items was done more as a service to bank customers rather than as a profit-making endeavor. Hence, it received little focus from management. Historically, verifying the correctness of incoming paperwork and manually sorting, filing, and totaling the items was performed by only the largest banks which were highly automated.

Companies specializing in item processing have emerged in the past ten years. Vader, Inc., the largest such company is a subsidiary of a small bank in Georgia. Each year Vader processes millions of airline tickets and retail payments for hundreds of companies, most of who are not customers of its parent bank. Vader uses high-speed processing equipment and is highly automated. Processing time is rapid and

Page 79: Mckinsey CaseBook

78

processing costs are low. In fact, because of this speed advantage, the parent bank is beginning to profit from the float of checks processed. Although industry-wide, a majority of the items are still processed by the issuing company or by small processors. Within five years, it is expected that most of the business will continue to migrate to Vader and other large processors. It is expected that Vader and the other large processors will dominate this market.

Vader has a significant cost advantage over smaller operations, such as the Bank of Luke, because of the great economies of scale they gain from processing such volumes of items. In addition, Vader benefits from a more constant workload by processing both airline tickets and retail lock box receipts. Airline tickets have few peaks and valleys, whereas mortgage payments always peak early in the month with very low volumes the rest of the month. Mr. Check believes that Vader quotes prices 20 cents per item to large prospective customers while the Bank of Luke processes items for 40 cents per item.

The President of the Bank, Mr. Kenobi, has asked Mr. Check to evaluate how the retail lock box service can be made profitable; the service lost $100,000 last year. Mr. Check believes that the bank must offer retail lock box services, and it must price the service to be competitive with companies such as Vader. Recognizing that outside expertise is needed, the President has given Mr. Check a budget to be used to hire a consulting firm. Mr. Check has asked you to visit his office to discuss the proposed engagement. While walking to his office, you observe that the Bank's retail lock box operations remains primarily a manual system, with limited use of modern, high-speed equipment and methods. Once in Mr. Check's officer, you note a picture showing the Department's staff in 1965; Mr. Check was a supervising clerk at that time. After reviewing some background information with you, Mr. Check asks you the following questions:

Question One

What do you see as your (the consultant's) role at the Bank of Luke?

Question Two

What steps would you take and what information would you gather to diagnose the problems facing the Retail Lock Box Department and to develop solutions to those problems?

Question Three

Page 80: Mckinsey CaseBook

79

From what you now know, what are the problems facing the item processing service and what recommendations would have the greatest impact on the performance of the Bank of Luke and the item processing service?

9.22.2Possible Solutions

In this case, we want to test the candidate's ability to handle a case in which the events appear hopeless until the end of the interview when an apparently easy solution (automation) is made available. The candidates should challenge the general premise of the case, and not simply believe that the business is necessary just because Mr. Check says so. We also want to test creativity with this case. We purposely leave the case rather vague by not suggesting any particular actions and by not offering much data. The candidate should be given time to think about this case and propose solutions which are not readily apparent

• Why not sell the business of these customers?

• Why not offer increased services to justify higher fees?

• What is the strategic plan for the bank and how does this unit fit into those plans?

• What does Mr. Check feel his unit should be generating?

(After all, $15,000 per employee is pretty low!)

• Has he considered acquiring other bank's customers to increase the economies of scale in his own operation?

This case can also be used to discuss cost cutting. Again, creativity and sensitivity to the real issues should be the goals of your probe; cutting 25 percent of the staff is too obvious and too easy.

9.23 Candy Company

9.23.1Issue

Your company is a rather successful producer of candy. It originally started as a single product line. The production process consists of two basic activities: manufacturing and packaging. The firm has also expanded its sales through

Page 81: Mckinsey CaseBook

80

product line extensions. Management is concerned that sales are growing but profits are not increasing at the same rate. What can you recommend?

9.23.2Possible Solution

This is a revenue vs. cost exercise. Margins are shrinking.

Find the critical components of cost: raw material, labor and fixed cost. Raw materials are commodities with cyclical prices that have fallen in recent years but are expected to swing up (as you have guessed, makes the problem worse). Labor and fixed capital has increased per unit over-proportionally compared with ten years ago. Upon further examination, you will find out that the company's controlling system is still focusing on the manufacturing part of production and the cost explosion occurs in packaging. (Candy is candy and the product line extension is primarily an issue of different packaging.) Controlling schedules manufacturing which is rather efficient already but not packaging, thus causing slack in labor and fixed capital (small batch sizes. high setup times).

The firm should consider reducing the number of product lines and introducing controlling/scheduling measures for packaging. However, we still need to consider whether the company's customers (i.e., retailers) are willing to accept the reduced product line.

Revenue killers: Concentration of retailers, trade brands, retailers demanding large introductory discounts for new products and high failure rate of new products.

The firm should streamline product lines, reduce low margin trade brand production, emphasize pull marketing, and reduce introduction rate for new products.

9.24 Skyscraper

9.24.1Issue

Your client is going to build a skyscraper, but is not sure how tall it should be. How should he decide how tall to make the building?

Page 82: Mckinsey CaseBook

81

9.24.2Possible Solution

This is an economic supply/demand mind teaser. Clearly, you do not want to lose money on the deal. The building will house tenants, who will pay to reside there. The costs of building and maintaining the structure (both fixed and incremental by story) needs to compared to the revenue generating capacity of the project. When marginal revenue equals marginal cost, the firm should stop adding stories to the building.

9.25 Consulting Firm (I)

9.25.1Issue

Your are the managing director in a large international consulting firm. The traditional strengths of your firm have been solving strategy and organizational issues. Recently, you have noticed an increasing number of your firm's proposals are rejected because of a lack of information technology expertise in your firm. So far, your firm's growth has been strong enough that proposals lost have not hurt annual earnings. Nonetheless, you are becoming increasingly concerned about the need to develop the firm’s capabilities in information technology.

Assuming your concern is valid, what reasons will you provide to the other partners about the need to acquire information technology skills?

Assuming you are able to convince other partners of the importance of IT expertise, what steps would you take to rapidly build IT capacity in this area?

What are the major risks in executing an IT capacity-expansion?

9.25.2Possible Answers

Good answers focus on the value of IT to clients. Discussion topics should include the increasing importance of information in business, strategic value of information and information loss, the importance of information systems for implementing new organizational structures and management control systems.

Page 83: Mckinsey CaseBook

82

Better answers focus on the costs of losing clients to competitors. Discussion topics should included the incremental costs of having clients talking with competitors about IT problems and the risk of losing new clients by not being able to solve a problem.

Good answers will focus on various methods to build expertise. The methods may include buying expertise by acquiring another firm, raiding IT practices of other firms for a few key consultants, building capacity through recruitment of IT experts and training them to be consultants, building capacity by training current consultants in IT practice skills and establishing a strategic alliance with a IT boutique firm.

Candidates should discuss the pros and cons of each method. Discussion points should address the impact on firm's current culture, the cost to the firm and the time needed to build expertise.

Better answers will realize the importance of stimulating client demand as capacity builds through seminars, articles, strategic studies in IT areas...

Good answers depend on the expansion methods discussed, but an important issue is the loss of the firm’s focus on just strategy and organizational issues.

Better answers will focus on the difficulty of implementation in IT; rapid technological changes in the IT industry require significant ongoing training and development costs; new practice cultures may be significantly different from current culture. These issues could be intensified if "external experts" are brought into the organization.

9.26 Cosmetic Company in Europe

9.26.1Issue

Eurocos Inc. produces and sells various cosmetics products in several European countries. The company's different brands are well established in the markets. The various products are quite similar in terms of raw material and production.

Page 84: Mckinsey CaseBook

83

The company has been doing very well in the past, however profits have been shrinking in recent years. The CEO of Eurocos Inc. thinks he should change his strategy. He asks you if this is a good idea and what he should do next.

Additional information:

* The industry has many small to medium size companies.

* A few big companies own several brands.

* Many small to medium size brands.

* Eurocos produces all products in all countries

* Transportation costs are small (see operational part).

Possible Solutions

What is the structure of the industry? -- Highly fragmented industry with the following characteristics:

* Low entry barriers (small setup costs...).

* High product differentiation (many ways of differentiation).

* Diverse markets, customer needs (language, complexions).

* High barriers, tariffs, customs exist between geographical markets

How can fragmentation be overcome and is the strategy feasible for Eurocos?

Learning curves present -- Yes

Standardize market needs -- No

Separate the commodity aspect of the product from fragmenting aspect -- Yes

Changing environment: reduced tariffs

The firm should consider consolidating production while keeping the marketing and branding nationally decentralized.

Pros:

• Lower costs in production (better sourcing, longer runs, quality)

• Optimizes locations (interest rates, wages, labor)

• Learning curve of running a more complex plant and logistics (see also Cons)

Page 85: Mckinsey CaseBook

84

• Keep "fragmented" marketing required in the market

• Total inventory decreases (safety stock at original plant locations can be pooled centrally)

Cons:

• More complex central operation

• Increased logistic complexity

• Transportation costs increase

9.27 Semiconductors

9.27.1Issue

The domestic semiconductor industry is beleaguered - brutal price competition from the Japanese, accusations of "dumping” against the Japanese, etc. Domestic semiconductor manufacturers are clamoring for protection from Washington, and some of the public policy solutions being proposed are research consortium sponsored by the government, trade restraints, etc. You are a consultant at a major firm. You are concerned that the public policy debate ignores basic issues regarding industry economics and whether the solutions being proposed will solve the problems faced by your clients. You know that each generation of memory chips lasts only four to five years. What are some of the factors you will consider while looking at the economics of the industry and how might they impact the idea of shared research by U.S. manufacturers?

9.27.2Possible Solutions

What are the cost drivers in the industry? (e.g., The split between fixed and variable costs involved.) The basic issue to be determined is that it costs huge amounts of money to be a player - roughly 250m in research and 600m for each plant. This increases exponentially for each succeeding generation of memory chip. Therefore, fixed costs are high.

Negligible variable costs. Cut-rate volume-oriented pricing - marginal cost of an additional chip is minimal. Semiconductor firms need access to huge amounts of capital on a continuous basis to survive for the long term.

Page 86: Mckinsey CaseBook

85

Raise pros/cons/issues of government participation in this issue. Is government involvement even feasible? What will be the priorities for the scarce government resources? Will the relaxation of anti-trust laws help? The candidate will also need to consider foreigner's access to cheaper capital.

Finally, what will shared research accomplish?

9.28 Airline Industry

9.28.1Issue

Historically low returns and stiff competition characterize the airline industry. In the early years after deregulation, discount carriers like People Express sprang up. Years later, the discounters have gone out of business. In a price-competitive industry, why is it that the higher-cost carriers were able to survive and the low-cost ones were not?

9.28.2Possible Solutions

Characteristics of discounters:

• Low fares

• Limited service.

Characteristics of major carriers:

• Higher fares but better coverage and service

• Hub systems

• Full service capabilities with larger volume base.

Competitive moves by major airlines included the innovative use of information technology for yield management and differential pricing. The larger airlines priced every seat individually based on continuously monitoring of demand/supply. They wooed leisure customers with fares lower than discounters and charged more from business travelers (indifferent to price but sensitive to service and frequency). The larger airlines stole the discounters' market and forced them out of business.

Page 87: Mckinsey CaseBook

86

9.29 Oil Tanker

9.29.1Issue

Your rich uncle has just passed away and left you with three small oil tankers in the Persian Gulf. How do you determine how much they are worth?

9.29.2Possible Solutions

This problem involves the interplay of supply and demand forces to determine the value of the tankers. The nature of tanker supply will be revealed by defining the different tanker types (in layman’s terms: small. medium, and large) in the industry and the cost-related prices associated with employing each type. In effect, a step-function supply curve rsults for the industry with each step representing a different tanker type. Demand for the services of tankers is assumed inelastic due to refinery economics dominating the purchase decision. It will turn out (by carefully drafting the supply/demand curves) that at the given level of demand, only large and medium tankers are put into service. This renders your late uncle's small tankers suitable only for scrap at the present time.

9.30 Fertilizer

9.30.1Issue

You have been hired by a fertilizer manufacturer to help them out of a difficult situation. Their market share and profits are declining and they cannot figure out what is happening. What are you going to do?

Possible Solutions

These are some of the basic issues to be flushed out:

Fertilizer is a commodity and consequently the basis for competition in the industry is on a cost basis. Who are the major players? What is their cost position vis-a-vis yours? It turns out that your client is the high-cost producer

Why is your client the high-cost producer? Examine the inputs to the process and analyze each one vis-a-vis your competitors (a long drawn out process). Are there economies of scale and where do you stack up on that dimension? It turns out that you are comparable on all dimensions except for a critical raw material (phosphate). You also do not have any scale advantages. Again, you will have to flush this out with your questions and approach.

Page 88: Mckinsey CaseBook

87

Examine critical issues relating to your disadvantage in raw material supplies? Why is it that you are at a disadvantage? It turns out that you probably cannot overcome this disadvantage. What are your alternatives? (If you got this far, you are probably doing fine!). Looks like you would try to explore the possibility of competing on a scale basis. What do you look at to analyze the issue?

9.31 Retail Advertising Pricing

9.31.1Issue

You are the new retail-advertising manager of a large daily newspaper. This morning you received a call from the advertising director (your boss). He sounded extremely worried about the retail advertising division's performance. (Naturally, he does not explain why, assuming that a hotshot like you would by now be totally familiar with the status quo!) He has to attend a meeting of senior executives convened by the publisher where he will have to defend the advertising department's performance. He also wants to make a big splash by presenting a new "strategic pricing methodology" aimed at achieving "value-based differentiated pricing."

9.31.2Possible Solutions

The interviewee must first find out the corporate profitability objectives. Assess gap between actual departmental performance and assigned targets. Examine both revenue and cost issues. (The candidate will discover that revenues have gone up steadily over the past few years. Further, costs have not risen significantly. So, why worry?) Apparently, corporate pressure to improve bottom-line results has led to steep advertising price increases. A classic demand-curve scenario has led to greatly decreased cumulative ad volume with potentially serious long-term consequences.

Examine competitor pricing and customer price sensitivity. Discuss heterogeneity in advertising customers based on business size, breadth of product line, price-point, etc. The candidate must understand advertising attributes of importance to different segments (e.g., color, size, frequency, discounting, etc.). Use difference in needs of customers to implement prices based on appropriate advertising service needed.

9.32 Automobile Industry

Page 89: Mckinsey CaseBook

88

9.32.1Issue

Your client, one of the big three automakers in Australia, has over the last few years under-performed relative to its competitors as measured by profitability. All three company’s current car models are "badged" Japanese designed cars (i.e., they are products of joint ventures with one of the smaller Japanese automobile manufactures). The Japanese market is much bigger than the Australian market. These cars are then sold in both Japan and Australia, the only difference being the place of manufacture and the model names (i.e., badges). You have been asked to establish why your client has performed poorly relative to the competition.

9.32.2Possible Solutions

Explore possible reasons for the under-performance - dissimilar products or production leading to the under-performance? - Different market segments? - Poor sales/ distribution? - Interior product? - High general expenses (admin, marketing, etc.)? - High cost of production?

Given that the reason for under-performance is the high cost of production, the candidate should establish the sources of high costs relative to the (other auto makers. using:

- Management accounts.

- Published financial accounts.

- Data from your American holding company.

- Reverse engineering.

NONE OF THE ABOVE HELPS! Don't panic, you know the solution of the problem has something to do with cost so... determine what makes up the costs and their relative importance?

- Labor costs.

- Raw materials.

- Manufacturing overhead.

- Design.

Given that design costs are by far the most important component of costs, explore the relevance of the Japanese connection?

Page 90: Mckinsey CaseBook

89

- Are the terms of out joint venture different from our competitors? - It turns out that the terms are all similar?

- What are the terms of the joint venture?

- Share of design costs pro-rated between the parties based on number of cars sold respectively?

- Does our car cost more to design than our competitors?

Although the answer to the last set of questions are negative, the solution is at hand. To recap, your client sells a similar product, in similar amounts and to similar markets in Australia. Your Japanese partner incurred similar design costs (in absolute costs). The key lies in your discovery that design costs are pro-rated. A line in the description of the problem that mentioned that your client's partner is one of' the smaller auto manufacturers in the huge Japanese market. Thus the design costs defrayed by the Japanese partner's sales in Japan are relatively small and your client’s share is significantly larger than the Japanese counterpart.

9.33 Scientific Industry

9.33.1Issue

A manufacturer of scientific instruments- is experiencing declining sales in its major product line. Why?

9.33.2Possible Solutions

Here are some questions that may help isolate the important issues:

l. Describe the instrument and what it does. (Goal: gather background information on the product).

Response: The instrument, call it Y, is able to perform elemental mapping; that is, it is able to determine the specific composition of material placed in the chamber for observation. Y is an accessory for larger and much more expensive instrument that functions almost exactly like a microscope, which we will call X.

Page 91: Mckinsey CaseBook

90

2. What other products does our client manufacture? (Goal: gather background information on the client).

Response: They recently began manufacturing X, and they have begun producing an unrelated product.

1. Can these instruments be used separately and are they ever sold separately? (Goal: understand the sales process and the potentially interactive role of the X and Y sales forces).

Response: X can be used by itself, but Y is essentially dependent on X for its operation. Consequently, except for replacement sales, Y is rarely sold individually. In fact, X's sales force will frequently recommend that a buyer purchase a certain Y while buying an X. Two years ago, over 30 percent of our clients sales were generated by a manufacturer of X.

2. What is the current percentage? (Goal: determine whether this could be a cause of the sales decline).

Response: It is currently around 5%.

3. Does our product X compete with other manufacturers of X and particularly the manufacturer that was selling our Y? (Goal: understand reasons for our friendly X manufacturer stopping promotion of our product).

Response: Yes, our X does compete directly with other Ys and our client introduced the product about one and a half years ago. (You have discovered a significant portion of the sales decline).

4. How does our product compare to other Y's? (Goal: determine whether others are beating us in technological or other product features).

Response: Our client's product is regarded as one of the best in the market.

5. Is the market for X and Y growing, shrinking or flat? (Goal: a shrinking market could be a good explanation for declining company sales).

Response: Both markets are flat.

Page 92: Mckinsey CaseBook

91

8. Who uses X and Y? (Goal: determine market segments).

Response: There are two basic user groups: industry - primarily semiconductor manufacturers, and academia, in research labs. What we have noticed lately is that the specific users in each of these groups, who also happen to be the primary buyers have become relatively less sophisticated. In other words, they are hired just to run the instruments and therefore know less about their technical qualities. These buyers have become even more dependent on the sales forces. What has happened is that our client alienated itself from other manufacturers of X at a time when a strong relationship was becoming even more important than it used to be. The buyers are relying more and more on the X sales force who is typically called well in advance of' the Y sales force. (The interviewer will not likely give you all of this information at once -questions about the buying process and changing decision makers would have brought it out.)

This is the second part of the main reason for our clients declining sales. In addition to ruining our relationship with a manufacturer of X by producing our own, we happened to do so at a time when relationships became even more important.

9.34 Aluminum Industry

9.34.1Issue

Your client is a leading manufacturer in the aluminum industry. Because aluminum is a commodity, relative cost position is the primary source of competitive advantage. As part of a strategic review, you have been asked to construct an industry cost curve (cost/kg of aluminum produces vs. industry supply), for various plant-to-market combinations. There are five major players in the industry, supplying six major geographic market segments. Your model should be flexible enough to enable various future scenarios to be run.

9.34.2 Possible Solutions

How to estimate competitors cost management?

- Financial accounts.

- Direct estimates by client management.

Page 93: Mckinsey CaseBook

92

- Indirect estimates by client management.

How to simulate the market mechanism?

- Determine what kind of market structure exists - perhaps an oligopoly.

- Perfect competition.

Given perfect competition, how do you simulate demand?

- Back of the envelope approach. (There are many combinations!)

- Linear programming approach.

-

The use of linear programming allows considerable flexibility as well as provides insight into questions such as:

- Is the industry currently efficiently configured?

- If a new plant is added to the industry, which market segment is most likely to be affected?

- What will the equilibrium price be in the future?

9.35 Meat Packing Industry

9.35.1Issue

Your client, a US firm, owns a meat packing plant in Spain. Over the last few periods, profits have steadily declined despite the fact that sales are growing. You have been hired to figure out why.

9.35.2Possible Solutions

Porter's five forces is a useful starting model in this case. By looking at the suppliers, you will know that they are independent farmers with little power against your client. Therefore, the costs of your raw material cannot be the issue. In analyzing the internal rivalry, you will discover the market is regional. Hence, transportation costs and competition have not changed dramatically. In addition, your production costs have remained stable. You will also discover that there has been no introduction of a substitute product. Since there are stable costs and strong sales, the only other alternative is the price of your product. Investigate this avenue, and you will discover

Page 94: Mckinsey CaseBook

93

the buyer link. Your margins are being squeezed due to the increasing concentration and buying power of your customers.

9.36 Piano Tuners

9.36.1Issue

How many piano tuners are there in Chicago?

Possible Solutions

This is a brain teaser case. Its purpose is to test your logical and quick mathematical thinking. There is no right answer. The test is to see if you can come up with an answer based on information that you provide during the case using estimates.

You need to start by asking questions about the critical factors. One way to solve it is to estimate the number of households in the Chicago area. The interviewer gave this piece of information at 2,000,000 households. Next, you can break the income of the households into four quarters (500,000 each). You can then make an estimate of 20% of highest income quarter have pianos, 10% of second quarter, 5% of third, and 0% of fourth.

Thus:

Income quarter Population % w/Pianos #of Pianos

1st 500,000 20 100,000

2nd 500,000 10 50,000

3rd 500,000 5 25,000

4th 500,000 0 0

Page 95: Mckinsey CaseBook

94

With 175,000 pianos to tune, you can estimate how often these pianos are tuned. You can estimate that the top income cluster tunes their pianos once a year, the second quarter once every three years and third quarter once every 10 years. This gives you (100,000 + 50,000/3 + 25,000/10) = 119,167 or approximately 120,000.

We can estimate that a piano tuner can do four pianos a day, 250 days a year, therefore:

120000/250=480 pianos a day to tune 4 = 120 pianos tuners needed.

How could you check this? Look in the yellow pages. Would all the piano turners be in there? You could guess that at least half would be. By the way, there are 46 piano tuners listed in the Chicago Yellow pages.

Page 96: Mckinsey CaseBook

95

9.37 Consulting Firm Strategy

9.37.1Issue

You are the newest member on the management committee of a well-known top-tier strategy-consulting firm. Eager to be accepted by your more senior peers, you volunteer to study the industry and propose a firm strategy for next millenium, which you will present to the committee at its next meeting. As you leave the meeting, you begin to realize the enormous task to which you have committed yourself.

How do you evaluate the consulting environment and determine likely future scenarios?

What information do you use in this process? How will this information be obtained?

What (to you believe is most likely to happen in the consulting industry given your present knowledge? How did you arrive at this conclusion?

What strategy do you propose to the management committee?

Possible Solutions

This is one of the most difficult types of cases because the answers are completely unknown and will vary substantially depending upon the interviewee's knowledge of the industry. This is also an interesting case since the salience is likely to be high. As an interviewer, you should feel free to add information on an as-needed basis. When information is not available, ask the interviewee to develop his or her own hypotheses. What matters here is the thinking process, not necessarily the answer.

A good place to begin is to evaluate the industry from a competitive analysis perspective. such as Porter’s five forces. The following is an abbreviated analysis.

Rivalry (low to moderate): The management consulting industry is fragmented, with many players each holding relatively small concentration of total market. Firms act as competitive monopolists and differentiate themselves by specialty, type of customer (Fortune 100 versus Fortune 1000 companies), reputation (McKinsey versus accounting firms), and the resources they employ (no MBAs versus all MBAs). Many companies are relationship-driven with their customers, which limits competition and keeps prices high. Top tier firms in particular are able to have high price points.

Page 97: Mckinsey CaseBook

96

Potential Entry (moderate): There are no great barriers to entry into the consulting industry; however, it is primarily the established firms that compete in the top tier of the industry. Its possible new firms would enter if the industry were earning positive economic profits.

Substitutes (moderate): Companies can move the consulting process in-house by hiring ex-consultants and bright MBAs.

Buyer Bargaining (moderate-high): In the last decade, the consulting market supply generally following demand, which lowers buyer power. However, it is appropriate to question the effect a recession might have on industry. Its possible demand may decrease as companies quit expanding, which would reduce demand, give buyers more bargaining power, and push prices lower.

Supplier Bargaining (low-moderate): Major suppliers are the intellectual capital employed by firm (e.g., experienced consultants who bring in sales and new consultants who provide analytic). Firms must pay market price or risk losing suppliers.

Other interesting points might explore the critical success factors in the consulting industry. What sets top tier firms apart from middle ones? Do any firms have specific sustainable competitive advantages? How does the marketing mix differ among firms? Does your firm have any specific core competencies or advantages that set it apart from other companies?

Determining likely future scenarios is more ambiguous. There are several important points to consider:

What affect will a recession have on consulting firms?

Will top tier firms suffer differently from others?

How will the mix of products demanded change over time? (e.g. cost-cutting studies rather than market expansion studies.)

Will the consulting market continue to expand or suffer a cutback?

Will certain geographical areas expand (Pacific Rim or Eastern Europe) faster than others will?

Again, the thought process is more important here than actual answers.

Page 98: Mckinsey CaseBook

97

Information gathering is a critical reason companies use consultants. An interviewee should have an understanding of business information sources and how information is gathered.

Information can be broken into two groups: secondary and primarily. Usually one begins with secondary material. Specifically, a complete review of published literature (a "lit search") pertaining to the study (e.g. journal and newspaper articles, investment bank research, specialized studies, books, etc.). This often points towards other good sources (e.g. industry experts, associations, major competitors, government sources, etc.). Hypotheses are often created from the secondary information. Primary research is then used to focus in on the critical issues. This research includes telephone interviews, in-person interviews, mailed questionnaires, focus groups, laboratory experiments, etc.

Most answers will depend upon the material covered in the first two sections of the interview. The interviewer should ask the following questions of the candidate:

What are the likely trends?

What is a positive scenario? A negative one?

If you had any information at your disposal, how could you get a better handle on this issue?

There is no right answer here, so the interviewee may balk. However, you can provide some structure y using the following questions:

What are the key success factors to succeeding in the industry?

Is there any way to achieve a sustainable advantage that cannot be duplicated by your competitors?

Can you use non-traditional methods to achieve competitive advantage, such as leveraging through technology?

Given your firm's competitive strengths and core competencies, what is the best strategic route?

9.38 Corn Feed Company

9.38.1Issue

Page 99: Mckinsey CaseBook

98

A corn feed company has eight manufacturing plants located in the Midwest. These plants service the entire United States. Their plant in Ohio is in need of refurbishing. The company has four possible options:

1. Refurbish the existing plant

2. Build a larger plant at the current location

3. Build a similar size plant at a new location

4. Build a larger plant at a new location

Which is the best option for this plant?

9.38.2Possible Solutions

There are two issues to this decision. The plant size and the plant location should he considered separate.

Size of Plant

First, consider the demand for the product. Corn feed is a commodity product. Pricing on the product is dependent on current corn prices as opposed to the manufacturing process. There are four main competitors in the industry - our company is the second largest. All four competitors have similar manufacturing processes and similar cost structure. The proposed largest plant will not have economies of scales that are not already present in the existing plant. The capacity utilization is 65%, which is industry standard. The current customers buy from all four manufacturers in order to guarantee supply. Currently demand is being met and there are no alternative uses for corn fed.

9.38.2.1.1.1.1 Location of Plant

Transportation cost and perishability are the main issues with location. The transportation cost per ton of corn stock (raw material) is much higher than the cost of transporting the actual feed. The corn is grown in the Ohio area and the feed is sold to the East Coast. The raw material is perishable where as the corn feed can be stored for any length of time and is easier to transport. Cost analysis of the transportation cost of feed versus raw material should be completed. Included in this analysis could be the rate of spoilage through longer transportation of corn stock.

Page 100: Mckinsey CaseBook

99

Conclusion

The current plant is located close to the cornfields and this is the best location for the plant from the cost/benefit analysis.

9.39 Selective Binding Case

9.39.1°Issue

Your client is a major fashion magazine that has been offered by its printer a proprietary new process called selective binding which enables publishers to customize the pages included in readers magazines based on demographic data known about the reader. For example, an ad in Better Homes & Gardens for lawn chemical services could be placed in only in those issues going to subscribers who live in houses and not to those living in condominiums or apartments. In this way, advertisers can focus their communications on the demographic segment they are targeting. Would you advise your client to take advantage of this new process and offer selective binding to its advertisers?

9.39.2Possible Solutions

This is a straightforward cost/benefit analysis. The magazine would want to offer the service to its advertisers if it would be able to enhance its earnings by being able to charge its advertisers a premium for being able to target more exactly the demographic segment. Of course, the increased revenue from any premium must be able to offset revenue lost as advertisers stopped using mass advertising The interviewee could start the analysis by obtaining the following information from the interviewer.

Q. What demographic breakdowns are possible to make in the magazine's database?

A. The only breakdown possible on your database is between subscribers who make under $50,000 and those who make over $50,000.

Page 101: Mckinsey CaseBook

100

Q. What is the total readership, the proportion of readers who are subscribers (as opposed to newsstand buyers), and the proportion of subscribers in each demographic category?

A. There are 1 million readers, 80% of who are subscribers. Twenty-five percent of subscribers make under $50,000 and 75% make over $50,000. The same mix applies to the newsstand buyers according to readership audits.

Q. What proportion of the client's advertisers target each demographic category of readers'?

A. Most advertisers are selling high-end fashion products, so 75% of them are targeting the high-income group.

Q. What is the cost of the selective binding service and what does the magazine charge for its ads?

A: The service is being offered to your client for free for three years since the printer wants to promote the services use by getting a major magazine to start using it. The client charges $50 per thousand per full-page ad (selective binding can only be offered on full-page ads). Therefore, revenue associated with a single inserted page (front and back) in an issue is 100 per thousand.

Q. What does the client's closest direct competitor charge for ads and what is their readership like?

A. The client's closest direct competitor has 500,000 readers, 100,000 of who are subscribers. Effectively, all of their readers make over $50,000. They charge $70 per thousand for their full one-page ads.

Since the printing cost to the client of selective binding is zero, the client simply needs to evaluate cost on the basis of revenue per thousand gained or lost as their advertiser base uses the service to better target. Presumably, instead of 100% of advertisers paying the full $50/thousand per page, the 25% of advertisers targeting the lower income segment will choose to target only that 25% of subscribers. The 75% of the advertisers targeting the high-income segment will advertise only to the high-income subscribers (75% of subscribers). Assume that all advertisers continue to advertise in 100% of the newsstand copies. The revenue effect of this change can be calculated by looking at the impact the change would have on average ad rate per thousand on subscription readership:

New add revenue per page = Old ad revenue per page X [(% low income subscribers X % low income target advertisers) + (% high income subscribers X % high income advertisers)]

Page 102: Mckinsey CaseBook

101

Thus,

New ad revenue per page = $50 X [(25% X 25%) + (75% X 75%)] at old rate = $31.25 < $50

The next question is can ad rates per thousand on the selective binding portion of ads sold be increased sufficiently to increase average revenue per thousand over what it is today. To answer this question, our client's ad rates must be looked at from the perspective of their advertisers. If you consider the advertisers targeting the high-income group, their alternative to advertising in your client's magazine is to put their ad dollars toward the 100% high-income readership competitor. The cost per thousand high-income readers with the competitor magazine is:

(Page rate X total readership)/ (portion of readers who are high income) = ($70 X 500,000)/500,000 = $70

Thus, $70 is the maximum price per thousand the client can charge its advertisers for selectively targeted ads. Any higher cost and the advertisers would switch to their competitor. Note that currently the client is a cheaper option for these high-income advertisers although they are paying to reach readers they do not want:

($50 X 1 million)/750,000 = $66.67

If the client charged $70/thousand for selectively bound ads, the average revenue per thousand to the client would he:

$70 X [(255 X 25%) + (75% X 75%)] = $43.75

Since $43.75 is less than the $50 that advertisers are currently paying, the magazine should not often advertisers the selective binding service.

Of course, there are other issues which interviewees might want to mention such as the possibility of price discriminating between high - and low-income advertisers. For example, the potential for and cost of expanding the advertising base using selective binding as a selling tool. However, it is important at the end of the interview to have reached a recommendation regarding the initial question posed by the interviewer. To mention these other possibilities and areas for further investigation is

Page 103: Mckinsey CaseBook

102

certainly wise, but it is also important not to get too far off track or to complicate the issue so much that a final recommendation is never reached.

9.40 Video Games

9.40.1Issue

The CEO of a large, diversified entertainment corporation has asked a consulting team to examine the operations of a subsidiary of his corporation that manufactures video games. Specifically, he needs to know if he should approve a $200 million capital request for tripling the division's capacity.

You are a member of the consulting team assigned to this project. Assume you and I are at the first team meeting. What are the critical issues we should plan to examine to determine if the industry is an attractive one for the CEO to continue to invest and why'?

9.40.2Possible Solutions

The following information may be given if requested by the candidates though you should focus on having the candidate identifying issues and not simply obtain more information.

Market Share: The division is the third largest manufacturer of hardware in the industry with 10 percent market share. The top two producers have 30 and 25 percent market share, respectively. The remainder is divided among small producers. The division sells to great range of consumers.

Sales: The division sales have increased rapidly over last year from a relatively small base. Current estimated annual sales of 500,000 units. The current estimate of industry hardware sales is 5.000,000 units annually. Industry growth has been strong though over last few months sales growth has slowed. The division’s current sales price for the basic unit is $45 per unit. The division’s sales remain less than 20 percent of parent company sales. The top two competitors also develop, manufacture and sell software/games though our division sells only licensed software. Industry growth of software continues to increase.

Page 104: Mckinsey CaseBook

103

Costs: The division estimates current cost is $30 fully loaded. The requested expansion should reduce the cost by 5 to 7 percent and triple production capacity of the hardware units. Our competitors are estimated to have a 10 to 15 percent cost advantage currently. The main costs are assembly components and labor.

Customers: The division estimates that much of the initial target market (young

families) have now purchased video game hardware. No other large segments

have been identified, yet.

Distribution: The primarily outlets of distribution are top retailers and electronics stores.

Profitability: The division currently exceeds corporate return requirements; however, margins have recently been falling.

Product: the industry leaders have established hardware standards. Product features are constantly being developed (e.g.. new remote joystick) to appeal to market segments.

The primary issue of the case is to determine if the industry is attractive and especially, if our client's position in that industry is sustainable. The candidate should identify issues that are necessary for assessing both the industry and our client's position, but should not be expected to solve the problem, per se.

It the candidate begins to discuss too deeply a specific issue before having covered the key issues overall, bring them back to discuss the industry more broadly by asking "what other issues must be examined'?"

If the candidate is discussing issues that seem irrelevant to the attractiveness of the industry, ask, "how will that analysis help to assess the attractiveness of the industry or our client's position?" Then ask the candidate to identity other issues that must be examined.

The following issues would need to be covered for the candidate to have done an acceptable job:

1. What is the future market potential? The candidate needs to question the continuation of overall industry growth. She/he might ask about the saturation of markets, competitive products (home computers), and declining "per capita" usage.

Page 105: Mckinsey CaseBook

104

2. What is the competitive outlook? The candidate should at least recognize the need to examine the competitive dynamics. Issue areas might include: concentration of market shares; control of retail channels and R&D capabilities (rate of new product introductions, etc.).

3. What will be the price/volume relationship in the future? Issues of prices need to be considered.

There are no bounds on creativity, but better answers would address:

Market Potential

Recognize that there is a relationship between market penetration and growth in new users which, when combined, yields an industry volume estimate.

Address the shitting mix of product purchases. In this case, from hardware (player units) to software (videocassettes).

Seek to look at buyer behavior in critical buyer segments. (i.e., "fad" potential of product.)

Software

Recognize industry leaders set technology standards. In this situation, the division as a secondary player and will have to follow these standards.

Recognize that different distribution needs may exist for different products (in this case. hardware versus software).

Company ability to Compete

Should ask what the capacity expansion is designed to do.

Explore the cost position of the client division relative to that of other competitors.

-Seek to understand the reason for poor profit performance of division.

Page 106: Mckinsey CaseBook

105

9.41 Steam Boiler Hoses

9.41.1Issue

The firm was asked by a diversified manufacturing client to help turn around the steam boiler hose division. This steam hose division provides boiler hoses for both external customers and the client's boiler division. Background information on the client and industry includes:

Boiler hoses are sold both with original equipment and as replacements. There has been increasing price pressure in the industry. The client is third of eight industry participants.

How would you structure an analysis aimed at restoring profitability? Where do you expect to be able to save in costs?

9.41.2Possible Solutions

The following information is also available in response to questions asked by the candidate:

Last year's P&L showed (as a percent of sales):

Raw Material 70%

Labor 20%

Distributed overhead 10%

SG&A 15%

Profit (15%)

The raw material is a commodity petrochemical. At least two of the other companies in the industry are making moderate profits.

MINIMUM REOUIREMENTS

Page 107: Mckinsey CaseBook

106

The candidate should avoid being bogged down in the following areas:

1. Drop the product line (apparently not possible because hoses are necessary for boiler sales).

2. Raw material prices (they are the same as everyone else's)

3. Allocation of overhead (no cash savings and provides little savings potential)

4. SG&A (standard industry fee paid for independent installers).

BETTER ANSWERS

Better answers will move beyond the previous answers to consider:

1. Scale economies (client is big enough to achieve scale production).

2. Production technology (client has a modern plant)

3. Labor costs (wages rates and productivity are average for the industry)

4. Raw material purchasing practices (material are purchased through long term contracts with prices based on the spot market minus a discount).

OUTSTANDING ANSWERS

The best answers follow a logical progression and should not stumble upon the actual answer:

The product has been over-designed, requiring excess raw material.

The answer should address the following organizational implication:

1. How is our product engineering operation wired into the marketplace? (There is little contact between the engineering and marketing/sales organizations.)

2. What kind of comments are we receiving form our sales force? (Customers are delighted with our hoses but require all the product features.)

3. Are there other areas in the company where similar problems exist?

9.42 Merger Candidate in Chemical Industry

Page 108: Mckinsey CaseBook

107

9.42.1Issue

One major chemical producer has retained the consulting firm to evaluate another major participant in the industry. Both companies are bulk commodity chemical producers. We have been asked to begin our work by analyzing the future prospects of the target company's major product line, a bulk chemical used in the production of plastics. Essential facts included:

* Production of this chemical has slowly declined over the last five years.

* Prices have declined rapidly.

* There are 7 to 8 major producers: the largest producer has a 30 percent share, number two has 20 percent, our target company has 15 percent and the rest is divided among the other competitors.

* The two largest competitors earn a small return. The target company is probably at break-even and the rest are operating at break-even or loss.

* The largest competitor has just announced construction plans for a major new plant.

How would you structure an analysis of the target company's future prospects in this product line?

9.42.2Possible Solutions

MINIMUM REOUIRENENTS

1. What markets use this chemical and what has been the nature of the growth in these markets? (The end users of this product are largely automotive-related.)

2. How much overall capacity exists now? (Far too much.)

3. What has been the relative capacity utilization of competitors in the industry? (60 to 70 percent for last three years).

4. What are relative cost positions of competitors? (Related to size/efficiency, age of plant. The target company has reasonably "good" position.)

BETTER ANSWERS

1. How rational is pricing in the market? (The industry is prone to self-destructive cuts to gain temporary share points.)

2. Are there niche or value-added uses for chemical? (Not really.)

Page 109: Mckinsey CaseBook

108

3. Does the chemical have a major by-product or is it a by-product? (Not of significance in this case.)

4. How often have companies entered/exited, and how expensive is entry/exit'' (Entrance is expensive; exit cheap mostly because older plants are fully depreciated.)

5. How important is this product line to each of the competitors? (Most producers are fully diversified.)

OUTSTANDING ANSWERS

1. Reasons for announced capacity expansion. (It is a bluff to try to encourage smaller competitors to shut down.)

2. Is regulation important? (Yes: all competitors have installed pollution control equipment.)

3. What is nature of operational improvements that target company could make? (Lots)

4. How is the product sold and distributed? (Economies of scale in marketing and transportation are critical.)

5. Is there synergy between our client and target? (Not really.)

9.43 Machine-Loading Case

TYPE Macroeconomic

PURPOSE To determine whether the candidate can dissect a general economic problem

9.43.1Issue

A client produces a range of synthetic materials in varying widths and lengths. Each material is used for packaging but differs in physical properties in terms of costs, weight, flexibility, and general performance. Each material can he coated with any one of four or five types of chemicals which make the materials more or less impervious to heat, light, water, vapor, etc.

All of the machines on which these materials are made are housed in one enormous factory location. Each machine is capable of running any one of the various materials and/or coating combinations. The client does not wish to invest in additional equipment at this time.

The client has asked us what combination of products he should ran to increase the profitability of the plant. How would you go about determining the optimal mix of potential products?

Page 110: Mckinsey CaseBook

109

9.43.2Possible Solutions

Market Share: The industry is highly fragmented. A variety of' small manufacturers supply similar products to a wide range of customers. Our client estimates he has less than 1 percent of' the total market. No competitor has more than three percent of the total market.

Cost: Each product has a different cost to manufacture dependent on materials used and the manufacturing process.

Price: Each product has a different price dependent on both the client's cost to manufacture as well as the market for the product.

Products: Our client's machinery can produce hundreds of different products. Some

are unique to meet specific customer requirements while others are used by a variety of customers.

Customers: Our client's customers are primarily consumers or industrial product

manufacturers who use the synthetic materials in packaging their own products.

Suppliers: Our client uses primarily commodity products in the manufacturing process. All products can be obtained from a number of sources.

NOTE TO THE INTERVIEWER

The primary issue of the case is to determine that the profits of the plant will be maximized when the most profitable product mix is produced and sold. The candidate could cover differences for each product in the fixed and variable manufacturing, selling cost and prices. These areas must be determined to understand the profitability of each product. The interviewee should also address the market demand for each product (to ensure what is produced can be sold at an acceptable price).

MINIMUM REQUIREMENTS

1. Are there market limitations to the potential production of any one material'?

2. Is there competition for these products?

Page 111: Mckinsey CaseBook

110

3. Are there differences in costs in the manufacturing of these materials? For example, do some coatings cost more than others do? Do some materials have inherent cost differences?

4. Is there flexibility in pricing of' these products?

BETTER AN'SWERS

1. Are there differences in setup time and cost for various materials or coatings?

2. Do these materials move at different speeds through the machines?

3. Are the machines truly interchangeable or are some better suited to one product or another?

4. Is there unlimited market demand for these products?

5. Are there technological displacement or replacement products on the horizon?

OUTSTANDIN'G ANSWERS

The best candidates will formulate a profit maximization algorithm. The best algorithm is to maximize the profit contribution per machine hour.

1. Profit contribution is (unit volume) times (unit price minus variable cost).

2. Machine-hour capacity is a surrogate for fixed costs per unit of volume. Fixed costs take into account depreciation and standby costs as well as those costs that are independent of the variable costs per pound or ton produced.

3. An outstanding answer must include recognition of the asset costs and capital implied in that as well as income or profit contribution. In addition, the potential substantial differences in volume produced per product-hour and/or the price obtainable in the market demand and competitive actions.

9.44 Oil Refining Industry

9.44.1Issue

Your company has 25% world-wide market share of the oil industry. You generate $4M annually in revenues through the machinery division of the company, which supplies machinery to refineries (not owned by your company) around the world. How do you asses the current operating status of this division?

Page 112: Mckinsey CaseBook

111

9.44.2Possible Solutions

Define "assess...operating status" - most likely in comparison two dissimilar pieces of information: 25% market share and $4M (but no idea what % of the market this represents). The guide is to request what % of the market $4M represents. Assume this is unknown. An estimate of the market size is therefore needs to be done. The way to do this is to ask how many oil refineries there are, how much does each cost to build, how long they last (actual life, not dependent life) and what the machinery replacement costs are. From this, one can estimate what the industry spends per year on machinery can. Divide the above mentioned $4M into this and the refining division's market share can be assessed. This % can then be compared to the 25% share of the parent.

9.45 Agricultural Equipment Manufacturer

9.45.1 Issue

Your client is a large agricultural equipment manufacturer. Their primary product line, farming tractors, is losing money. What questions would you ask of your client to help them solve their profitability problem?

9.45.2 Possible Solutions

It is unlikely that there are too many players in this market. You might want to start off by asking how many competitors there are. Suppose the answer is that there are two direct competitors.

What is your client's market share relative to their competitors (your client has 40% of the market, competitor #1: 30%, competitor #2: 15%, with the remaining 15% belonging to many small manufacturers.)

What-are the market share trends in the industry? (Five years ago, your client had 60% of the market, competitor #l, 15%, and competitor #2, 10%. Obviously, your client has lost significant market share to its two competitors over the last few years.)

Do all three competitors sell to the same customers? (Yes)

How is your product priced relative to your competitors? (Your client’s product is priced higher than the others.)

Has this always been the case? (Yes)

Page 113: Mckinsey CaseBook

112

Are the products the same? (Essentially yes, they all have the same basic features. Of course, tractors are not commodity items and a few differences do exist.)

What are the differences that allow you to charge a premium for your product? (Your client has a strong reputation/image of quality in the market and the market has always been willing to pay a premium for that reputation because it meant they would last longer and need less maintenance. This can be critical for some farmers because they cannot afford to have a piece of equipment break down at a critical time.)

Are sales revenues down? Are sales quantities down? (Yes)

Is the price down? All costs the same? (No, in fact both the price and costs are up.)

Have fixed costs increased? (`No, material costs, (variable costs,) have gone up out of sight, and the client has no answer as to why material prices have gone up so staggeringly.)

Do you manufacture your tractor or just assemble it? (Primarily an assembly operation.) Finished part prices have gone up? (Yes)

Raw material prices for your suppliers? (I don't believe so)

Have labor costs Increased for your supplier? (No)

Have you changed suppliers? (No)

Why are your suppliers charging you higher prices for the same products? (Well, they're not, the prices have increased as a result of our product improvement efforts. We've tightened tolerances and improved the durability of our component parts.)

Why do you make these improvements? (Because we strive to continue to sell the best tractors

in the world.)

Are your customers willing to pay for these product improvements? (What do you mean.)

Are your customers willing to pay a marginal price which will cover your cost of implementing these improvements? (I don't know, I guess we assume that they will...)

Page 114: Mckinsey CaseBook

113

It turns out that prices have been raised to cover the costs of these improvements, but customers do not value these improvements unless they are essentially free --so sales are down. The client needs to incorporate a cost/benefit analysis procedure into its product improvement process. Don't forget though, that you must consider the long-term effects of these decisions.

9.46 Insurance Company

9.46.1 Issue

An insurance company pays its sales people a base salary of monthly wages and commission of 25% of new policy sales (2% of renewal). Which is the right way to pay the sales agents?

9.46.2Possible Solutions

This, in case you have not already surmised, is an organizational behavior scenario. Again, you must define what the "right way is". Assume some generic definition like "the manner by which agents are both motivated and equipped to accomplish there tasks in the interests of the organization..." is applicable. Having set up by definition, the results achieved by the above mentioned composed system are examined. The only factor determining how much the agents paid is their sales $. In essence, they are motivated to issue a policy to anyone at as high a price as possible. They are not motivated to give consideration to the riskiness of the insured party. The absence of such a consideration (for example) would be detrimental to the company in the long run. A more efficient compensation structure might pay the agent on a sliding scale, depending on how risky (costly) an insured party proves to be.

9.47 Consulting Firm (2)

9.47.1 Issue

Your client is the treasurer in a significantly privately held corporation. She is in charge of managing a portfolio of investments in addition to her treasury responsibilities. Recently, she has asked your advice about the purchase of a large position in company 456, whose stock is listed on the NYSE.

Company 456 is currently selling for $22 per share. The treasurer's investment analyst predicts that the stock will pay a dividend of $1.25 for the foreseeable future. Short-term treasury bills are yielding 7 percent, and long-term t-bills are yielding 8 percent. The treasurer is contemplating the purchase of 5000 shares of company 456 and wants your help in determining a fair market price.

How would you go about determining a fair price for company 456?

Page 115: Mckinsey CaseBook

114

9.47.2 Solution

$22 per share

9.48 Pots & Pans (2)

9.48.1 Issue

A manufacturing company based in Charleston, SC makes high quality pots and pans which are sold throughout the U.S. in specialty and department stores. You are called in because they feel that the $ l million that they spent on distribution last year was way too high. How can you show your client money that he can save money.

9.48.2 Possible Solutions

Distribution is basically a trade-off between cost and service level. The higher the service level, the higher the cost (more inventory pools, warehouses and shipments). So you need to ask where the inventory is being held. It turns out that stores, since they sell so few of these pots and pans, hold no inventory and thus require next-day replenishment after a sale. The next thing you need to know is where the warehouses are located, since the closer they are to the stores the cheaper the distribution costs. Your client has three warehouses - one in Charleston, one in Philadelphia and one in LA - from which they cover the whole country.

A quick way to solve this case is to realize that if stores require next day service from these three warehouses, the only way they can do this is by shipping overnight at a premium rate (UPS - no wonder they're spending so much). You can save them a bunch of money by closing down Philadelphia and LA and shipping everything from the plant In Charleston by UPS (negotiate a volume rate). This can be confirmed by asking for the annual sales which turns out to be 10,000 units. When you divide this into the $1 million distribution cost you discover that they are pay $100 to deliver a pan to the store. Beat this figure and you've earned your exorbitant fee.

9.49 Diapers

9.49.1 Issue

You have been retained jointly by Pampers and a federal commission on waste management to estimate the volume percentage of disposable diapers in the total US household garbage.

9.49.2 Possible Solution

Wet your pants/skirts. (No, wait until after the interview for that). This is strictly a mathematical, number crunching exercise. You need a numerator (diapers) and a denominator (total US household garbage). Let's assume this will be done in pounds. For diapers you could take the total $ sales of disposable diapers and divide by the average price per total unit (box: etc.). Multiply this number by the average weight per unit, yielding the estimate of total diaper weight (numerator).

Page 116: Mckinsey CaseBook

115

Figures on garbage tonnage (denominator) are probably available in some obscure federal report.

9.50 Cable Television Company (2)

9.50.1 Issue

Q: Your client is a small holding company that owns three cable television companies in the Northeast: Rochester, NY, Philadelphia and Stamford, CT. Each of these three companies is profitable, and each has been experiencing steadily growing sales over the past few years. However, the management feels that th e Northeast is not the fastest growing area of the country, and, therefore, acquired another cable television company in Tucson, Arizona a little over a year ago. Despite every effort of management, the Tucson company’s sales have been stagnant, and the company has been losing money. How would you analyze this situation, and what could be the cause of the poor performance of the Tucson cable company? To be divulged gradually: The Tucson area is smaller than Philadelphia, but larger than Rochester and Stamford. Tucson is also growing at 12% per year on average. Per capita income is higher than in Philadelphia and the same as in Rochester and in Stamford.

Operating costs in Tucson are essentially the same as in the other markets. The cost of programming is based on number of subscribers and is equal across the nation. Operating costs are composed of variable items: sales staff, maitenence, administration and marketing. Only maintenance is higher that in the other markets, due to the larger land area serviced. Fixed costs relate to the cable lines, which is a function of physical area covered.

The Tucson company has attempted marketing efforts in the past, such as free Disney programming for one month, free HBO for one month, free hookup, etc. These programs have been modeled after the other three markets.

Cable penetration rates in the three Northeastern markets average 45%. The penetration rate in Tucson is 20%. These rates have been steady over the past three years in the Northeast. The penetration rate in Tucson has only rised by 2% in the past three years in Tucson.

There is only one real substitute good for cable television: satellite dishes. However, many communities are enacting legislation that limits their usage in Tucson. They are also prohibitively expensive for most people.

9.50.2 Solution

The real error of management results from their failure to recognize another “substitute” good: no cable television at all; television reception is far better in the desert Southwest than in Northeastern cities. The lower penetration rate is most likely a result of different climate conditions and lower interference in Arizona.

Page 117: Mckinsey CaseBook

116

9.51 Chilled Beverages

You are consulting for the manager of a division of a large consumer products company. Her division produces fruit juices in three forms, all marketed under the same name: chilled (found in the milk section of the supermarket, usually), juice boxes, and frozen concentrate. This division has sales of $600 million per year. The entire company has sales of over $20 billion. The chilled segment represents $120 million in sales per year. While juice boxes and frozen concentrate are profitable, chilled juices are only breaking even in good quarters and losing money in bad quarters. She has received a proposal from upper management to sell the chilled juices business. What would you advise that she do? To be divulged gradually: Chilled beverages is a $5 billion dollar industry nationwide. There are two large players that have 40% and 25% of the market, respectively. Your client’s market share, 12%, makes her third in the industry. The best available information indicates that the two market leaders are profitable. The two market leaders are able to fund more advertising and more promotion, trade and couponing that your client.

The market leaders produce pure orange juice and blends that are based on citrus juices. Your product uses more elaborate blends of juices, usually with a base of pear or peach juice (95% of the inputs) and flavored with cranberries, bananas, mangoes, etc. (the other 5% of the inputs). Pear and peach juice are about the same price as orange juice, but the other flavorings cost about twice as much.

The market for chilled juices is essentially mothers with school age children. This is a highly price sensitive market that loves coupons, promotions, etc.

Brand name is important in this market, as in juice boxes and frozen concentrate, as mothers tend to prefer highly reliable products for their children. However, the brand premium must be in line with other branded products. Therefore, all branded juices tend to sell in the same price range.

One plant in California produces all of the product, chilled, juice boxes and frozen. It would be difficult to find another use for the plant without a major conversion.

9.51.1 Solution:

Page 118: Mckinsey CaseBook

117

There are three choices:

Sell the chilled juice business. This would, however, affect the juice bix and frozen concentrate businesses, as there are both advertising and manufacturing synergies.

Sell all of the juice business. This may be more feasible, as the buyer could capture the synergies, but would not be too likely to turn the business around. The selling price is likely to be low.

Keep the chilled juice business and rework the ingredients and costs. This turns out to be the most feasible option, as evidenced by the success of the competitors.

9.52 Distilled Spirits

9.52.1 Issue

You are consulting for a major United States producer of distilled spirits. Their primary products are a line of mid-priced vodkas and two brands of mid-range rum. Over the past few years, the business has become less and less profitable. What could be causing this:

Other information:

The split of product sold has consistently been 60% vodka / 40% run over the past few years. The selling prices of the two lines are essentially the same. Overall sales are growing at about 3 to 5% per year, the same as the idustry average for these product lines.

An analysis of the costs reveals the following: Production Costs have remained constant

Advertising Costs have remained constant on average

Distribution Costs have increased significantly

The products are sold throughout the country. In 27 states, where alcohol is sold in privately managed supermarkets and liquor stores, “open” states, shelf space is extremely expensive and trade promotions are critical. Such stores are alsom becoming less and less willing to hold inventory, which is increasing distribution costs by requiring more frequent deliveries. In the other 23 states, liquor is only sold through state regulated liquor stores. Distribution costs in these states is much lower, as there are far fewer outlets to service and central warehouses for the state-run stores. Advertising of alcohol is much more tightly regulated, and therefore, advertising spending is lower.

Page 119: Mckinsey CaseBook

118

9.52.2 Solution:

A greater and greater share of the volume is being sold in the “open” states, with sales in these states increasing at about 10% per year. Sales in the regulated states are actually decreasing. Because the regulated states are less expensive to serve, and therefore, more profitable, the fact that they represent a shrinking portion of the total has caused total profits to decline.

9.53 Chewing Gum Market

9.53.1 Issue

How would you estimate the size of the annual U.S. chewing gum market? Check your answer for reasonableness.

9.53.2A typical approach:

Estimate the number of people who chew gum: of the 300 million population, 15% are between the ages of 10 and 20, the heaviest users, for a total of 45 million. Estimate that these people chew two packs per week, for annual sales of 4,500 million packs. For the other users over age 20, (70% of the 300 million population, or 210 million) estimate a usage rate of one half pack per week, for a total of 5,250 packs per year. Total packs per year is 9,750.

To check for reasonableness, figure the dollar sales that these packs represent: at 25 cents per pack, annual sales would be $2.4 billion, a reasonable figure.

9.54 French Pizza Market

Pizza Hut has recently entered the home pizza delivery business in Paris. The market for home delivery is currently dominated by Spizza Pizza. Pizza Hut has asked your consulting firm to help it analyze issues that will determine its likelihood of success in the Parisian Pizza market. First, what information would you need and second, how would you analyze the pizza delivery market?

9.54.1Possible Information Needs:

An estimate of the size of the Parisian home pizza delivery market. This could be obtained by knowing the population of Paris (6 million) and making some educated guesses about factors that determine pizza market size.

You may also want to know the size of Spizza, the current competitor, including sales, number of stores, and proportion of Paris that is currently served by Spizza.

Other useful information: market segments targeted and served by Spizza; market segments that are neglected by Spizza; what type of product do they offer; what do they charge for thier product; what is the cost structure of their business and what products are most profitable.

Page 120: Mckinsey CaseBook

119

9.54.2Method of analysis:

The best method of analysis would start by determining if any part of the market is not well served currently by Spizza. Determine what are the needs of any neglected market, and understand if your client could profitably serve this market.

Also, try to understand the likely competitive response of Spizza to your client’s entry. How will you defend your position if Spizza decides to fight for market share?

9.55 Golfball Market Entry

9.55.1 Issue

You are visiting a client who sells golfballs in the United States. Having had no time to do background research, you sit on the plane wondering what is the annual market size for golfballs inthe U.S. and what factors drive demand. Your plane lands in fifteen minutes. How do you go about answering these questions?

9.55.2 Typical solution:

Golfball sales are driven by end-users. The number of end users: take the population of 300 million; assume that people between 20 and 70 play golf (about 2/3 of the population, or 200 million) and estimate what proportion of these people ever learn to play golf (guess 1/4) which reduces the pool to 50 million. Now, estimate the frequentcy of purchase. If the average golfer plays twenty times per year, and requires two balls per time, that’s forty balls per person. Multipy that times the 50 million, resulting in a 2 billion ball market.

9.56 Overseas Construction

9.56.1 Issue

An overseas construction firm wants to expand by estamblishing a presence in a growing U.S. regional market. How should it go about doing this? What factors are critical for its success?

9.56.2 Suggested framework

What are the diversifying firm’s distinct competitive advantages?

What is its capacity for funding an acquisition?

What is the competitive environment like in the proposed region?

How does this environment differ from the current markets of the diversifying firm?

9.56.3 Possible Solution

Diversification could be effected through joint ventures or through acquisition. Which of these two strategies would prove the most suitable would depend on the availability of funds and uponthe nature of the companies operation in the region.

Page 121: Mckinsey CaseBook

120

However, the success of the venture would depend notonly upon the means of entry. Other critical factors would include:

The existence of a distinct sustainable competitive advantage. For example:

Non-unionized labor might help support a low cost production strategy (but for how long?)

Proprietary technology not available to other compaies in the region

Special expertise in a growth area (such as, for example, hazardous waste)

Access to distribution channels

9.57 Packaging Material Manufacturer

Your client is the largest North American producer of a certain kind of bubble-pack packaging material. Currently, the company has 80% of the market, and has asked your firm to assess the strategic outlook for this company. How would you begin to assess the future for this client, and what type of recommendations could you make?

Information to be divulged gradually:

Costs for the product are broken down as follows: 20% for polyethylene, a plastic chemical. 35% conversion costs, including allocated fixed costs, labor and energy costs 10% distribution and storage, 15% marketing and overhead. Profit margins are 20%. Poyethylene is a commodity chemical. The factory is thriry years old, and the technology used is the same as when the factory opened.

The client had 100% of the market until two years ago. Since that time, a localized upstart company has appeared in the Philadelphia / New Jersey market and has captured nearly all of that market. This factory has purchased technology from a German company. Your client does not have much information about this competitor, but it appears that their factory is extremely efficient. They have also been undercutting your client on price.

Solution:

The competitor has used their new technology to produce a lower price product. As evidenced in the Philadelphia / New Jersey market, nearly all customers prefer this product to your client’s. Therefore, the future is extremely bleak for your client, and they should be advised to respond to the competitive threat, perhaps by updating their own technology.

9.58 Airline Expansion

Page 122: Mckinsey CaseBook

121

A major airline is considering acquiring an existing route from Tokyo to New York. How can it determine if the route is a good idea? Suggested frameworks: Profitability analysis looks like the best approach. Simply determine if revenue less costs equals a positive profit. Then, analyze the factors that go into revenue and the factors that comprise cost to come to a conclusion. Interviewer Notes: Revenues will be determined by occupancy rates and expected prices. Both of these will be determined by expected demand, the competitive invironment and the extent to which our client could win over passengers from competitor routes. Operating costs will depend on expected fuel costs, incremental costs for landing rights, etc. It is also very important to estimate the cost of cannibalization on existing Tokyo-LA, LA-New York routes. And, last but not least, it is important to note that losing passengers to cannibalization is better than losing them to competitors.

9.59 Health Care Costs

Bill Clinton has just fired Hillary Clinton as Chief of Health Reforms and has appointed you to fill the position. while in his office, you discover that kidney dialysis is a major portion of public health care expenditures. What analytical techniques do you use to determine if this cost can be reduced?

Suggested frameworks:

You can start this case by looking at the cost half of profitability analysis (Costs - Fixed + Variable). Since this is a procedure, rather than a shole industry, it is mostly a variable costs, the sum of which is measured by cost per unit x # of units. Thus, one could look at this problem by analyzing (1) how much it costs per kidney dialysis and (2) how many kidney dialyses occur in the U.S. also, Don’t forget the external factors, such as corruption or government regulation, that may play role.

Interviewer Notes:

Analyze the proportion of public versus private health expenditures that are applied to kidney treatment to determine if this expensive treatment is being pushed onto the public leath budget by unscrupulous practitioners.

Page 123: Mckinsey CaseBook

122

Compare the indicence of kidney disorder in the country with other countries. is ours higher? If so, can public policy ofr efforts to increase awareness help reduce it?

If incidence is indeed higher for the U.S, build a model (regression, perhaps) that will somehow determine the factors that are most related to kidney treatment. Perhaps those who are typically covered by public funds (the poor, the elderly) have a higher incidence of kidney problems. Is there room for any type of preventative program for these groups?

9.60 Local Banking Demand

How would you determine whether a location in New York City holds enough banking demand to warrant opening a branch?

Suggested framework:

Because this is a demand-oriented question, one should consider a marketing framework, such as the 4 P’s.

Interviewer Notes:

The demographics of the area surrounding the prospective branch should be examined. Population, business concentration, income levels, etc. should be compared with those of historically successful branches.

Competitor reactions could easily make this benture unprofitable, so it is essential to anticipate them. These will depend on the importance of the area to competitiors (in terms of profit, share, etc.)

The client will have to match competitors’ incentives to customers and should estimate the cost of doing so.

The client must examine if the new branch would complement their existing competence and strategy (retail or commercial, high growth or high profitability, etc.) and what purpose it would serve. If the need focuses on deposits and withdrawls only, maybe a cash machine would suffice.

Page 124: Mckinsey CaseBook

123

9.61 Frozen Desserts

You are consulting for a small, regional maker of high quality premium priced frozen desserts. (Ice cream and similar products). Though sales have been increasing, the business is barely making a profit and the management is unsure that they will able to pay their usual dividend this year. They have asked you to help them identify the problem. Additional information: The client sells a complete line of product (ice cream and frozen yogurt) in major supermarket chains in the Northeast. In recent years, as Americans jump on the fitness bandwagon, frozen yogurt has begun to outsell ice cream, and currently represents 55% of product sold. The selling price per pint is the same for frozen yogurt and ice cream. The ingredients are different, however. Ice cream uses locally available milk and cream, and flavorings such as chocolate, pecans, vanilla and coffee. The premium frozen yogurts use more exotic flavorings such as mangoes, kiwis, pineapple and raspberries. All other costs are equal for the two lines.

Solution:

Margins on frozen yogurt products must be lower than for ice cream, or possibly even negative, due to the higher ingredient costs. Therefore, the shift of sales from ice cream into frozen yogurt is causing the company as a whole to be less profitable.

9.62 Direct Mail Retailer

You are consulting for a direct mail retailer that sells ladies clothing. Your client’s catalog printing and postage costs have just increased to thirty-two cents per catalog. How can your client decide if the new price is acceptable?

Information to be divulged gradually:

The average response rate for catalogs mailed is 2%. In other words, each 100 catalogs mailed results in 2.5 orders place. The average order size is $80. In addition, 25% of customers who order product can be expected to reorder within six months. The fully allocated profit margin (excluding mailing costs) on catalog orders is 15%.

Solution:

Page 125: Mckinsey CaseBook

124

For each 100 catalogs mailed, printing and postage costs are $32. (100 x 32 cents).

Each 100 catalogs will result in 2 orders, plus 2 x 25%, or .5 additional reorders, for a total of 2.5 orders placed per 100 catalogs mailed.

2.5 orders will result in 2.5 x 80, or $200 in sales. At a profit margin of fifteen percent, these sales will return a total profit of $30.

The $30 profit is not sufficient to cover the printing and mailing costs of $32. Therefore, the client should reject the printing arrangement at 32 cents per copy.

9.63 Chemical Sweetener Manufacturer

Your client manufactures a chemical sweetener used in beverages and other food products. The chemical will come off patent in one year. You have been asked to predict what might happen to the profitability of this product when the product comes off patent.

Information to be divulged gradually:

This is the only product of its kind, in terms of taste and safety (lack of harmful health effects) as proven in lab tests. The brand name of the product has slowly become a common household word.

The largest two customers (75% of your sales) are two worldwide beverage companies. The companies feature the brand name of your client’s chemical on their product, and consider it a sign of quality. In addition, the cost of the chemical sweetener represents 1.5% of their total costs.

The costs to manufacture the product are extremely low (about 20% of the price of the product). Currently, the margins on this chemical are almost 40%.

Solution:

This is a classic customer analysis problem. While most products that come off patent quickly drop in price (e.g. pharmaceuticals), this product will be able to retain some of its premium due to the strong brand name. Because the major two customers feature the chemical name on their product, and because the chemical represents such a small portion of their total costs, they can be expected to be willing

Page 126: Mckinsey CaseBook

125

to continue to pay the premium into the future. Therefore, the outlook for the product is good even after the patent expires.

9.64 Telecommunications Diversification

A Baby Bell company is interested in diversifying into other areas besides telecommunications. They are considering entering the market for electronic home security systems. Would you recommend that they do so?

Suggested frameworks:

Use an industry attractiveness framework, such as Porter’s Five Forces, to determine whether this is a business you want to be in, or at least to determine what kind of returns you can expect to achieve. then, use the value chain to look at where value is added in the home security business. finally, once you feel you understand the market, determine if the core competencies of the Baby Bell are likely to match the demands of the home security markets.

Interviewer Notes:

The company is a holding company. They have previously made unsuccessful forays into software and into real estate.

The home security business is highly fragmented. The top five players in the industry generate less than 4% of the total industry revenues. This implies that the industry largely consists of small, regional companies.

10% of all residences currently own an electronic security systems.

This is is some sense a razor and razor blade sort of business. The economics are:

Item Retail Price Cost / Margin

Equipment and Installation $500 - $1,500 0-10% margin

Monthly Service $20 / month $5 / month

What strengths / competencies of the Baby Bell company are useful in this market? Consider: Installation expertise, operator services, transmission system (phone lines)

Page 127: Mckinsey CaseBook

126

It turns out that the “expensive home” segment of this market is saturated. Growth has been slow in recent years.

Price sensitivity is unknown in “moderate-priced home” segment.

The conclusion is that this business is a reasonably good fit for the company, but that more market research needs to be done to assess the growth and profit potential of each segment of the market.

9.65 Aluminium Can Manufacturer

An aluminum can manufacturer has discovered a way to improve its manufacturing process. As a result, its manufacturing cost has been reduced from $0.89 to $0.79 cents. How can the manufacturer best exploit this cost advantage?

Suggested frameworks:

Remember basic economics. The firm can either use a penetration strategy or price skimming strategy. Consider the impact of either strategy on the company and its competitors. Also, don’t forget to think about any substitutes for aluminum cans.

Interviewer Notes:

Clearly, the client should either drop price or reap additional profits.

It turns out that the client is the leader in its market with a 40% share and supplies directly to major beverage manufacturers. The number two player in the market has about 30% of the market and the rest is shared by many small competitors.

Aluminum cans have a lower priced substitute, steel cans, which have inferior printing and stamping characteristics. Steel cans are used by customers who do not want to pay the premium for aluminum cans.

If the client drops prices, other competitors will have to follow since this is a commodity market and not following would mean a quick demise. The lowering of prices might increase the client’s market share marginally, but some smaller competitors will have to start exiting the industry and larger competitors will have to start investing to discover the client’s cost advantage.

Page 128: Mckinsey CaseBook

127

At the same time, steel can users sill start switching to aluminum cans, thus hurting manufacturers in that market. The resulting growth in the aluminum can market will attract steel can manufacturers to enter it. Since some steel can manufacturers have deep pockets and a strong backing, these new entrants could pose a future threat to our client. In conclusion, it is best to retain prices and generate extra profits for now. The cost advantage may help another day during a price war.

9.66 Film Processing

The CEO of the largest domestic manufacturer of photo film want to enter the film developing business. He needs your advice on how to go about evaluation this idea. What would your approach be?

Suggested frameworks:

This is and industry entry question; look at industry attractiveness with Porter’s five forces analysis. Then, think about what part of the marketing mix (4 P’s) would be best for film developing. Finally, analyze competitive response.

Interviewer Notes:

Distribution chanels are the key factor in this business. Major discout stores sell the service.

This is a scale economy business in the back-office, so profits are easier with high volume. This makes the business tough to enter.

This company ended up establishing a “store within a store” concept with Wal-Mart.

9.67 Concrete Manufacturer

Your client, a concrete manufacturer is considering acquiring a small local firm. What factors should be considered? After considering these factors, would you recommend the acquisition?

Additional Information to be divulged gradually:

The target firm is currently profitable, with margins of 5%. Your client’s margin is 15%. Your client attributes its higher profit margin to economies of scale in trucking and mixing, and a stable labor force.

Page 129: Mckinsey CaseBook

128

Both companies compete in the geographical market, the Southeastern U.S. Your client’s customers are large construction firms and contractors generally in the office and commercial building construction business. The smaller firm sells mainly to other small businesses and contractors. (Swimming pool installation firms, patio builders, etc.)

Additional research shows that the smaller customers for concrete are growing, while the major office building construction market is stagnant. The smaller firm has strong contacts with many local customers, and is often the preferred supplier due to their customer responsiveness.

Your client is not able to fund the acquisition internally, but could obtain bank financing at a rate of 10%. Similar acquisitions generally are made for two to three times current sales of the target firm.

Solution:

From a financial point of view, the acquisition is not attractive if there are no synergies between the firms. With profit margins of only 5%, the income generated by the smaller firm will not cover the capital charges (interest due to the bank) on the acquisition price. (Acquisition price = 3 x sales. Interest on this amount will be 10% x 3 x sales, or 30% of annual sales. Profits are only 5% of sales. This analysis, of course, ignores the tax shields.)

However, if your client were able to use some of its competitive advantages to improve the financial outlook of the target firm, the acquisition would be advisable. It is reasonable to expect that synergies would arise from economies of scale in trucking and mixing, which could raise the profit level of the target firm, and make the acquisition more attractive.

9.68 Shipping Container Manufacturer

Your client is a manufacturer of large steel shipping containers that are designed to hold up to several tons of material for shipping on ocean liners. The container consists of a steel frame, a steel shell and an insulation and waterproofing material that uses a hazardous chemical. The containers are leased by the company to worldwide shipping companies. Shippers can lease the containers one-way or round-trip. The client has asked you to do an assessment of their strategy. What issues might you examine?

Suggessted Issues:

Page 130: Mckinsey CaseBook

129

Sales and cost issues: The growth of the shipping container market; your client’s share in that market; trends in the leasing terms in the industry; customer power; steel prices; manufacturing costs.

Market issues: changes in the worldwide shipping market (e.g. does the growth of an area like Southeast Asia imply many more one-way contracts than round-trip?); growth of the largest customer industries; new technology in shipping containers; customs and trade agreement trends.

Environmental Issues: Production and disposal of the insulation chemicals; costs of handling the chemicals.

9.69 Healthcare Company Growth

A large healthcare company has decided it is interested in substantially increasing the size of its operations. Its goal is to double total sales and profits in less than two years. As a consultant brought in to assis them, what would you do? What issues would you consider? What are some likely alternatives for the company?

Possible issues to consider:

What is the current scope of operations? In what areas of healthcare does the company deal? What is its current market share in these areas?

What plans has the company already considered?

What is the competitive nature of the industry? What would be the effect on sales and profits of reducing prices and margins?

What potential is there for expansion by acquisition? Do they have the financial capability? do potential acquisition targets exist? Will the market for acquisitions be competitive?

Possible recommendations:

Naturally, a suitable solution will depend upon the answers to the above questions.

A business can increase profits by:

Increasing sales

Page 131: Mckinsey CaseBook

130

Increasing prices

Decreasing costs

However, if the company’s margins are found to be consistent with industry norms, it would seem unlikely that either increasing prices or cutting costs represent feasible methods by which to double sales & profits, particularly if the company is operating in a moderately competitive environment.

This leaves only sales increases, which could be achieved by:

Selling more of the current products to current customers

Selling new products to current customers

Selling current products to new customers

Selling new products to new customers

The suitability of these options will again depend on the particular environment. In the particular example of this case, it turned out that only selling new products to new customers via some form of diversification could hope to achieve the company goals.

You should then consider the potential for increasing sales by means of diversification through acquisition or joint venture. The relative benefits of each will depend on financial resources as well as the existence of, and competition for suitable targets.

9.70 Regional Grocery Store Chain

A regional chain of grocery stores currently receives its stock on a decentralized basis, i.e. each store deals directly with the vairous suppliers. The president of the chain is wondering whether it would be better if they established a centralized warehouse through which all supplies would be delivered and then disbursed by company trucks. What are the key consideration to making this decision?

Issues to consider:

Would the savings from bulk purhcasing more than compensate for the cost of:

Building and maintaining the warehouse

Employing additional personnel and trucks

Opportunity cost of capital tied up in inventory for additional periods

Page 132: Mckinsey CaseBook

131

Do the stores buy similar products? (i.e. do purchasing synergies actually exist?)

Will delivery frequency to the stores by better or worse? Consider the costs of stockout and the need for fresh produce.

Will the stores prefer delivery direct from the supplier or from the warehouse? Consider the time tied up in order processing, the flexibility of delivery times and quantities.

Possible solution:

The proposed solution would depend upon your interpretation of the trade-offs both financially and organizationally for the two methods of delivery. For you to propose going with the new method, you need to establish not only that it will cost less, but also that all the affected players can be persuaded to buy into it.

9.71 Magazine Distribution

A magazine publisher is trying to decide how many magazines she should deliver to each individual distribution outlet in order to maximize profits. She has massive amounts of historical data for sales volumes through these outlets and a well constructed internal accounting system. How should she go about computing an appropriate number?

Possible solution:

The best way to tackle this one (without going into a huge Economic Order Quantity qunatitative analysis) is not so much to start asking questions as to set out and outline analysis and fill in as you go.

It should be observed immediately that to maximize profits, marginal revenues whould be set equal to marginal costs. The marginal revenue for a magazine would be its cover price times the probability that it will be sold. The probability of sale, with an appropriate confidence interval, could be established in some manner from the historical data. The marginal costs could be obtained from the internal accounting data.

A detailed discussion of the application of these concepts from basic microeconomics and statistics may be necessary.

Page 133: Mckinsey CaseBook

132

9.72 Knitting Machine Demand

How would you asses the world demand for knitting machines?

Possible Solution:

The worl demand for knitting machines basically depends on the world demand for cloth.

In order to evaluate the world demand for cloth, we need to know how much cloth (measured in square meters, for instance) is being purchased per unit time per inhabitant of the world. In order to refine our appraisal, we may segment the inhabitants of our planet per level of personal wealth. Note that this may not be a linear relationship.

Furthermore, you may need to consider other factors:

The current level of the ratio: amount of cloth manufactured per working year / number of machines

The expected usable life of an average machine

The existence of substitues for knitting machines and the consequences of this on our expected demand

9.73 Cement Manufacturer Capacity Addition

You are consulting for the number-one producer of cement in Portugal. This company currently has 45% of the market, and feel it could have more, but is running at 100% capacity of their one plant, located near Lisbon, in Southern Portugal. The CEO has asked you to help him decide if they should build another plant or expand the current plant.

Additional information to be divulged gradually:

The cost structure for cement production is as follows:

Raw materials 28%

Labor and allocated fixed costs 16%

Distribution 26%

Sales and overhead 18%

Page 134: Mckinsey CaseBook

133

Pre-tax profit 12%

The company’s selling prices are set by prevailing market prices in Portugal. Land is available to expand the current factory; there is also a suitable site near Porto, about 200 miles to the north. Approximately 80% of the customers are within 100 miles of the current plant.

Raw materials are purchased from a government-owned company, and prices are set by a yearly contract with the government. The plant is unionized, and extra shifts are not possible. The trucks are owned by the company, and transport all product directly to the customers throughout the country. Customers pay for trucking by the mile. The fixed cost of plant additions is roughly the same as the cost of a new plant of the same capacity.

Solution:

As distribution is the second-largest cost item, it makes sense to minimize distribution costs in choosing the site of the next facility. From the data, it is safe to assume customers that are further away are less inclined to buy due to the increased trucking costs. Therefore, location of the plant in the north may increase sales in the north by reducing delivery costs to these customers.

Page 135: Mckinsey CaseBook

134

9.74 Snack Food Company

A large salted snack food company has steadily been losing market share over that past two years, from a high of 20% to the current level of 18%. Profits as a percent of sales, however, have been growing. What could be causing this?

Additional Information to be divulged gradually:

The size of the total salted snack food market has grown from $15 billion to $17 billion during these two years; the interviewee’s conclusion should be that the client’s total dollar sales have actually grown, but not kept pace with the market. The product line of the client has not changed over this period.

The costs for the client have changed over this period: ( % of selling price)

Current Two years ago

Raw Ingredients: 28% 26%

Conversion costs: 24% 24%

Distribution: 8% 9%

Marketing: 16% 18%

Sales force: 7% 9%

Pre-tax profit: 17% 14%

The total sales force was cut to reduce costs, though the same number of outlets are still covered by this sales force. The changes in the marketing budget come from reduced trade promotions.

The products are mostly sold through large grocery store chains and convenience stores. The sales force generally visits each customer at least once per quarter. Promotions usually occur at the end of each quarter. Grocery stores and convenience stores require some type of promotion to grant valuable end of aisle displays or advertising space.

The largest competitors are two multinational consumer products companies that feature complete lines of snack foods. Their sales forces are regarded as the best in the industry. Together, these two companies have 55% of the market.

Page 136: Mckinsey CaseBook

135

Solution:

The data show that the greatest change is in the sales force numbers. It turns out that the company went on a cost-cutting spree over the past two years. The sales force was drastically cut and the commission scheme was reworked. The marketing expenditure was also decreased. Most of the reduction came from trade promotions. The product is sold through the same channels as previously: large grocery chains and convenience stores. These channels are traditionally driven by periodic trade promotions. The reduction in trade promotions brought about a loss of shelf space, which has directly led to the decrease in market share. Also, the product line has not changed in the past two years in a product category where new products and line extensions are routine. In addition, the market has been growing, indicating a missed opportunity for new products in the market. Lastly, the increase in profitability has resulted from the lower costs, but may not be sustainable.

9.75 Beverage Company Cost Structure

RC Cola and Coca Cola both compete in the same industry. Their cost structures are vastly different, however. Using Coca Cola as a benchmark, estimate the likely cost structure for RC Cola. In other words, for which costs would RC Cola be higher, for which would they be lower, and why?

Possible solution:

This is a twist on the standard price/cost case that also questions the interviewee’s understanding of the cost items. A possible analysis, line item by line item:

Cost of goods sold: RC Cola would be higher due to their lesser power in negotiating price breaks from suppliers.

Distribution: would be higher for RC Cola for two reasons. RC is not distributed in as many outlets as Coca Cola. Therefore, the average truck driver will be driving more miles and spending more time to deliver a truckload of RC that the Coca Cola driver, who will have several stops within an immediate area. Also, the typical order size for RC Cola would be smaller, meaning that more stops would have to be made. In the case of Coca Cola, it is conceivable that one truckload may be deliver to just one customer.

Sales Costs: could be lower for RC, as there are fewer, but more loyal customers.

Page 137: Mckinsey CaseBook

136

Marketing: is lower for RC Cola as they are not a frequent advertiser like Coca Cola.

Administration / Overhead: lower for RC Cola as they are more of a “one-product” company than is Coca Cola.

9.76 Permanent Light Bulbs

A small R&D lab in the Swiss Alps has developed a super-durable filament for light bulbs; with this filament, the light bulb will never burn out. The lab is ready to licence this product to a light bulb manufacturer. What will be the effect on the light bulb industry?

Additional Information:

The light bulb industry is dominated by two multinational producers. The two companies sell their products side by side for essentially the same price in similar outlets internationally. There are a several small local players in various regions of the world who produce local brands and some private store brand light bulbs. There have been no technological innovations in light bulbs for many years.

Possible solutions:

One outcome is that one of the two major players purchases the technology. If the technology is patented and exclusively licenced, this player may enjoy an advantage for a limited time. If the producer makes enough bulbs at a low enough cost, all customers will eventually switch over to the permanent light bulb, thereby drying up the industry, putting the competitor out of business and greatly reducing their own business.

Another solution is that all of the players obtain some version of this technology. If that were to happen, the price for this product would decline to the normal industry profit level, and customers would shift to the permanent light bulb. Over time, all bulbs would be permanent and the industry volume would greatly decrease, making the industry more competitive and wiping out industry profits.

9.77 Super Regional Bank

Page 138: Mckinsey CaseBook

137

You have a have recently been assigned to a project with one of the nation’s super regional banks. The bank is one of the top 10 largest retail banks in the country. Like most banks in its class it has branches in 8 geographically contiguous states.

Your client has recently concluded that the old “local branch” way of business is no longer viable. Typically, this bank has canvassed its territory with small free-standing branches; however, the new age of electronic banking and commerce is changing all of that.

They are considering replacing many branches with Calling Centers. Calling Centers offer both live and phone automated services that may be accessed by phone. The new Centers would offer virtually all of the services currently offered through local branches plus some additional things.

The question to you is: how would you go about setting up the engagement to determine the viability of this new concept? Specifically, what kinds of things would you investigate? and what hypothesis would you form?

Possible Solution:

This is a very open broad-brushed case. There certainly is no right answer; however this type of case occurs frequently. The following is a guideline of some things you should probably consider:

Market analysis: What kinds of customers would be attracted to this no service? What kinds of customers would be turned off? (Hypothesis: younger people would be heavier users and more attracted than older) Of the people attracted to this new service, how profitable are they? How profitable are the people who are turned off by this service? (Hypothesis: older people have more money and thus are more profitable)

Revenue: What types of new services could be added to increase revenues? Automatic bill payment, Fund transfer, etc.

Cost Savings: How much would it cost to establish a Calling Center and what are the risks involved? Do we have the expertise in-house to do this? How many branches could we close? Can we cut down on traffic to existing branches - thus requiring less tellers?

Page 139: Mckinsey CaseBook

138

Summary: It probably is best setup as a cost benefit analysis. The number of new customers times the expected revenue from them plus the additional revenue generated by potential new services plus the cost savings must outweigh the forgone revenue generated by the customers you end up driving away.

9.78 Cigar Bar

I was sitting in one of Chicago’s new specialty “Cigar Bars” around the end of August with a friend. It was a Saturday night and the weather was fair. While enjoying one of the bar’s finest stogies and sipping a cognac, I asked my friend how much he thought the bar was worth.

On the back of an envelope, how would you go about determining the value of this bar?

Issues to consider

We arrived at the bar around 8:30pm. There appeared to be 30 customers already there. By 11pm the place had at least 70 customers. I would estimate the maximum capacity to be close to 100.

The bar sells two things: liquor and cigars. The average cost of a cigar is $8 and the average cost of a drink is $7.

There was one bar tender, a waiter and a waitresses. All three were there the entire evening.

The bar is located on one of Chicago’s trendier streets with a lot of foot traffic.

The bar is open Tuesday thru Sunday from 5 pm until 2 am.

Possible Solution:

This is a straight forward valuation. To perform a valuation you must estimate the cash flows from the business and discount them back using an appropriate weighted average cost of capital (WACC).

Page 140: Mckinsey CaseBook

139

Revenues: One way to project revenues is to estimate the number of customers per day or per week and multiply that by the average expenditure of each customer. Keep in mind that Friday’s and Saturday’s are typically busier than other days and that people tend to be out more during the Summer than in the Winter.

Costs: There are two components to costs: fixed costs and variable costs. Under fixed costs you might consider: rent, general maintenance, management, insurance, liquor license, and possibly employees. The only real variable cost is the cost of goods sold.

Valuation: Subtract the costs from the revenues and adjust for taxes. You now have the annual cash flows generated from the bar. How long do you anticipate this bar being around? Cigar bars are a trend. In any case pick some number for the expected life (4-5 years). The discount rate should be a rate representative of WACC’s of similar businesses with the same risk. Perhaps 20%. This gives you a value of:

Value = CF1/1.2 + CF2/(1.2)2 + ... + Cfn/(1.2)n

9.79 New Magazine

Your client is the CEO of a publishing company that produces a line of educational magazines as well as a line of women’s magazines. Both businesses are profitable but are not growing quickly. He want’s to start a third monthly magazine in the US targeted at 30-50 year old men (eg. GQ Magazine) His stated goal is to generate circulation revenues of $10 million in the first year. He has hired you to figure out whether this is possible.

Possible Solution:

This is an estimation case. The key here is to clearly define your assumptions, the specific anwser is not important as long as you are making reasonable assumptions. For example

Target Customers

The total US population is approximately 240 million. Based on a normal distribution with the average life span of 80 years, approximately 2/3 of the population falls between 30-50 or about 160 million people. Approximately 1/2 are male or 80 million.

Page 141: Mckinsey CaseBook

140

Of the 80 million 30-50 year old men in the country, assume that at least 1/2 would read a magazine or 40 million. Given the wide range of magazines on the market assume that only 10% of magazine readers would want to read a men’s journal or 4 million target customers.

Share

As a new magazine assume that you can generate a 5% share of the men’s magazine market in year one or 240,000 customers.

Revenues

Based on what other magazines sell for ($2.50-$5.00) assume a cover price. Lets say $3/magazine at the news stand and $2/magazine for a subscription. Now make some assumptions on how many customers will buy on the news stand versus subscription, lets say 50% subsrcibe (120,000) and 50% buy at the news stand (120,000). This comes out to $360,000 + $240,000 or $600,000. Finally, this is a monthly magazine. For simplicity assume that all target customers buy a magazine every month. This would generate total revenues of $600,000 X 12 or $7.2 million.

In this case given the CEO’s stated goal of $10 million in circulation revenues, it would not make sense to launch the magazine.

9.80 Castor Manufacturer

Q: Your client manufactures castors (the wheels found on the bottom of office chairs) out of a plant in West Germany and One in East Germany. Over the past two years the company’s profits have declined by 20% while revenues have been relatively flat. You have been asked to find out what is happening and suggest a course of action to reverse these trends.

Information to be divulged slowly:

The company operates in three divisions: 50% of sales are to hospital bed manufacturers, 25% are to mop bucket manufacturers, and 25% are to chair manufacturers. The hospital bed and mop bucket divisions are located in the West German manufacturing operation, the hospital bed division is located in the East German manufacturing operation.

Page 142: Mckinsey CaseBook

141

Breaking out each division as a separate profit center shows that revenues are up 10% for both mop bucket and chair divisions but down 10% for the hospital bed division. Similarly, profits are down 10% for both the mop bucket and chair divisions but are down 30% for the hospital bed division.

Further investigation shows that labor is the major component of cost in manufacturing castors. In the past two years, wages in the formerly state regulated East Germany have skyrocketed. This is what is driving most of the increased costs. Similarly, the demand for hospital beds (and thus castors) in East Germany has declined as they have become more efficient at managing their health care system.

A:

This is a typical revenue/cost case. We have already been told that revenues are flat which should be a clue to explore the cost side of the income statement. In this case it helps to work logically through both the fixed and variable costs to see if there are any major items. Sometimes the interviewer will provide you with an income statement that will break out the major cost components by percentage.

Case Type: Industry Analysis; Profitability Analysis

9.81 Logging Company

Background:

You are hired by a Canadian logging company to analyze its current operations and provide advice on future operations. The logging industry in Canada is regulated by the government. Land is leased to individual companies by the government. The company is making a lot of money and is unsure why. You have been asked to determine: (1) Why they are making money? (2) Is it sustainable? (3) Is it replicable?

Additional Details:

• Products: The company produces lumber boards of two sizes 2”x4” and 2”x8”. Lumber is a commodity product and as such the company is a price-taker in the market.

Leases: The government leases tracts of land at a annual price that is set to allow for a 12% profit margin for the entire logging industry. Thus, all tracts of land have the same lease price per acre. The leases last for 99 years and the original lessee has the right of first renewal on the lease.

• Profit Structure: The profit equation for the lumber industry can be written as: Profit per ft^3 = Revenue per ft^3 - Non-land cost per ft^3 - Lease Cost per ft^3

Page 143: Mckinsey CaseBook

142

• Revenues: There is a revenue advantage for the company due to its product mix. Margins are higher on 2”x8” boards than on 2”x4” boards. The company’s product mix is made up of a greater percentage of 2”x8” boards than the “typical” logging company percentage.

• Non-land Costs: The company has a 5% cost advantage in its ”tree-to-dock” production process. There is no significant difference between the distribution costs among the industry firms.

• Production Process: The cost advantage is not generated by a better logging process (i.e. better equipment, more skilled laborers) but instead exists because of the exceptional quality of the trees on the particular piece of land that the company leases. The mineral content of the land leads to faster growth of healthier trees which improves both yield and turnover. Healthier trees are straighter and easier to cut, thus reducing costs in each phase of the logging process. These healthier, taller, straighter trees yield more 2”x8” board-feet than is typical and leads to the advantaged product mix. There are no significant economies of scale to the process.

Key Points

• The company leases land with a significantly higher quality of trees. This leads to a revenue advantage because more 2”x8” board-feet can be produced per acre of land. Additionally, there is a cost advantage because the higher quality inputs make the logging process easier and increase yields and turnover.

• Since the leases are for 99 years and renewable, the current situation seems sustainable.

• Since it is unlikely that another piece of land similar to this one exists or that another firm will give up advantaged land, the situation is not replicable.

9.82 Information Services Company

Background:

You are hired by a library information services company that provides a computerized article search product on CD-ROM. The product allows users in a library to locate articles by keyword search. The company currently has a weak market share of only 10% of all installed units. The company wants to understand (1) why they have so small a market share, (2) what could be done to improve the situation, and (3) where it should focus its resources.

Additional Details:

• Competition: There is a single major competitor which has 50% market share. The client and two other competitors each have 10%; and the remainder is divided among many competitors.

Page 144: Mckinsey CaseBook

143

• Market Segmentation: The following table outlines many of the details of the market segmentation and client product data.

Type of Library

Number of

Libraries

Client Market Share

Major Competitor

Market Share

Competitive Features

Academic 5000 20% 60%

• Research 500 80% 10% Search Quality, Content

• Other 4500 13% 66% Content, Ease of Use

Public 10000 10% 40% Content, Ease of Use

Secondary Schools

20000 ~0% 10% Price, Ease of Use

• Product: The client sells a CD-ROM based product which is used on a dedicated PC in a library. The product has different versions that are upgraded each year. Each version is marketed to a specific library segment. Libraries are interested in matching the article search to hardboard volumes available within the library. The client’s product is considered to have the highest quality of article search.

• Pricing: The client sells its product at a 25% discount to the major competitor and has the lowest prices in the industry. The pricing and profit schedule for each version are shown below.

Library Client Price Client Profit per Unit

Major Competitor Price

Academic $2000 >$500 $2667

Public $1500 $500 $2000

Secondary School $1000 $100 $1333

• Competitive Features: Competition within the industry focuses on four dimensions: (1) Search Quality, (2) Content, (3) Ease of Use, and (4) Price. The table above indicates the relative preference for these features for each market segment. There is a trade-off between ease of use and search quality. A better search requires a more skilled approach to keyword usage and often makes the search more difficult. The client’s product is considered to have the highest quality search among the competitors.

• Production: the product is created by programmers who seek to match the product to library volumes. Since the principal input is labor, the type of CD-ROM created can be altered relatively easily.

Key Points

Page 145: Mckinsey CaseBook

144

• The client’s product does not match the needs of the large segments of the market (i.e. the client’s high quality of search only appeals to a small segment of the total market) ==> weak market share

• The client should reallocate its resources to create products in the larger market segments -- products that emphasize content and ease of use over search quality.

The most profitable segment can be identified by using current client prices which should allow it to gain market share (due to the 25% discount to the major competitor) and calculating the maximum market profit. Academic = 5000 x 500= $2.5M; Public = 10000 x 500 = $5.0M; Secondary = 20000 x 100 = $2.0M. Therefore, if we realign our product to emphasize ease of use and content, the potential profit is 4500 x 500 + 10000 x 500 = 7.25M ( minimum since profit in academic segment is > $500 per unit).

9.83 Pipeline Company

Case Type: Industry Analysis

Background:

You are hired by a large pipeline company to evaluate the current and future potential of the pipeline industry. The pipeline industry sprang up as transportation costs for mineral extraction companies began to escalate. There is currently 20,000 miles of pipeline throughout the U.S. What information would you want to know about the pipeline industry that could help you plot a strategy for a pipeline company?

Additional Details:

• Industry Structure: There are many pipeline competitors. Pipeline can be characterized as either common carrier pipelines (~70% of all pipeline miles) which are regulated by the government and proprietary pipelines (~30% of all pipeline miles) which are wholly located on the private property of a firm (e.g. a pipeline from a port station to a near-shore refinery). There are many suppliers of common carrier pipelines. The second group (proprietary) is not regulated by the government.

• Products: The pipelines carry liquid and gaseous materials -- crude oil, natural gas, methane gas, liquid nitrogen, refined oil products (gasoline), and chemicals.

• Cost Structure: There are exceptionally high fixed costs involved in a pipeline. The variable costs are primarily the electricity to power pumping stations along the pipeline. There are different cost structures depending on the type of product being moved. Pumping crude oil along the pipeline can cost as much as $2M/month in electricity for a station. Gaseous products require considerably less energy to move.

• Market Conditions: U.S. proven reserves are diminishing and foreign imports are increasing. It is expected that for the next 5-10 years demand will be steady.

Page 146: Mckinsey CaseBook

145

Key Points: (classic Porter analysis could be used -- This is rarely the case!!!)

• Threat of Entry is low because ...

- there are high fixed costs (high initial investment)

- pipeline services are essentially a commodity product (commodity markets are slow growth and unattractive)

• Industry Rivalry is strong because ...

- there are many competitors and switching costs are low

- industry growth is expected to be slow (i.e. market share is important)

- many competitors use pipeline for in-house uses and only carry other products if capacity is underutilized

- there are very high exit barriers (i.e. there is a strategic relationship between refining and piping)

• Substitute Products are many as witnessed ...

- by proliferation of tanker cars and tractor trailer rigs for liquid and gaseous materials

• Power of Suppliers is not a significant factor.

Power of Buyers is not a significant factor because many pipelines are regulated and there are many buyers

• Other considerations:

- Product Mix: The margins on gaseous products is higher than heavy unrefined products.

- Government Regulation: Margins are greatly affected by common carrier status. Any future environmental regulations will cut even deeper into margins.

- Pipeline as a storage medium: For many firms the product in a pipeline can be a significant portion of its inventory and the volume in line must be considered in production. The classic question: Is it better to make product and sell it now at low prices or wait for prices to increase (e.g. crude oil prices)? A large pipeline could be a temporary storage facility.

- Operations: Maximizing profit means understanding the parameters of pumping -- costs of pumping at less than full capacity; layout of pipeline and pumping stations; products which can share the same pipeline; construction of parallel pipelines.

� Market Differences: The market for crude oil is very different than the market for specialty chemicals or natural gas. the pipeline manager must aware of these rapidly changing commodity markets to maximize his profit.

Page 147: Mckinsey CaseBook

146

9.84 Auto Manufacturer

Background:

Your team is hired by a large U.S. automobile manufacturer (GM). They are interested in your evaluation of their $10B after-market parts business. This business can be segmented into two sets of buyers: dealers authorized to sell GM parts ($8B) and non-dealer merchandisers ($2B). This second group can be subdivided into mass merchandisers and “service” providers. Mass merchandisers are of two types -- those which specialize in auto parts (e.g. Auto Zone) and those which sell diverse products including auto parts (e.g. Sears). “Service” providers include Goodyear or Western Auto. GM would like for you to answer two questions: (1) Is there an opportunity to expand this part of the business? (2) How would they go about doing it if they chose to expand?

Additional Details:

• Company Economics: There are tremendous fixed costs in the auto business (including labor). All of GM’s parts manufacturing facilities are fully depreciated and they currently have excess capacity.

• Competitors: While Ford and Chrysler make parts for their own cars, they are not nearly as integrated as GM and tend to focus in specific parts categories. There are hundreds of small parts manufacturers which tend to focus on commodity-like auto parts (e.g. oil filters).

• Products: GM produces a full spectrum of parts classified as either platform-

specific or universal.

Platform-specific Parts Universal Parts

Types of Parts Body panels, brakes, transmissions, engines

Spark plugs, filters, hoses, batteries

Market Characteristics

Sold through dealers under warranty; high margins/low volume

Sold through many outlets; high turnover; strong

competition; slim margins/very high volume

GM Sales $8B $2B

• Growth Rates: The table below provides the basic facts about each market segment’s growth rate.

Market Segment Overall Market Growth Rate

Total Market Size

Dealer-authorized -35% per annum $40B

Non-dealer

• Mass +65% per annum $70B

Page 148: Mckinsey CaseBook

147

merchandisers

• Service providers

+15% per annum $30B

Key Points: (Porter Five Forces analysis)

• Threat of Entry is minimal for a broad category because the fixed costs are very high. However, a manufacturer could go after a niche play if it were to develop an advantaged cost structure or superior product. Switching costs among consumers is very low.

• Industry Rivalry is important for the mass merchandiser category because margins are slim (meaning price wars are more prevalent). Brand names (e.g. Fram, AC Delco, AutoLite) are important to many consumers.

• Substitute Products are relevant only in the sense that there are many competing products and future technologies such as electric cars could eliminate the need for many types of parts.

• Power of Suppliers is not a significant factor because inputs are commodity raw metal and rubber.

• Power of Buyers is important since there are few mass merchandisers such as Sears or Kmart and they demand full range of products and tremendous volume discounts.

GM’s Position: GM may have a cost advantage due to its fully depreciated plants and excess capacity in a fixed-cost environment. Thus its variable costs must be below sales revenue. Also, its brand names are respected and are valuable to merchandisers in maintaining margins. GM’s ability to produce a full-range of products is also an advantage. These advantages combined with the high growth rates for the non-dealer merchandisers should motivate GM to expand it business in this segment.. GM should use its cost advantage, brand names, and full range of products to go after the most lucrative market -- the mass merchandisers.

9.85 Deli Meat Producer

Background:

You have been hired by a producer of deli meats to investigate the cause of its recent decline in market share. The client would like an action plan for resolving the cause of this decrease.

Details:

• The Company:

Page 149: Mckinsey CaseBook

148

- Product: The firm produces plastic-wrapped packages of sliced deli meats at all price points (generic, midrange, and premium). The market share loss is primarily in the premium category. The deli meats carry a well-known brand label.

- Price: Products in the premium category carry a higher price and have slightly higher margins. Although price decreases will garner market share, the competitors have maintained prices during the recent loss in market share.

-Place (Distribution): The product is sold in grocery stores and delis. Company investigation has shown that grocers have maintained the same amount of shelf facings and space for your product (so the decrease in share was not caused by changes in display or incentives provided to the grocers by competitors).

- Promotion: Advertising and marketing efforts have been steady during this period of decline and there has been no noticeable change in the competition’s efforts.

• The Competition: There are three other competitors in the deli meat industry. Each of these competitors has about 20% of the market share; the client has 40% of the market share. Overall the market (generic, midrange and premium) is growing. The competition uses the same channels to sell its products.

• The Customer: Although the customer buying premium deli meats has not changed, a survey of the customers indicated a variability in the quality of the product produced by the client. Sometimes the product was better than the competition; sometimes not. This was causing customers to change to the competition.

Solution:

• Production Process: The client receives chunk meat in bins which meet a certain average quality measurement. Meat is rated on a scale of 1 to 100 (100 being best). The client is in a long-term contract with a supplier for bins at three quality ratings: 40, 70, and 90. Individual chunks within a bin may vary from this average. The premium deli meats are made from a mix of the three bins with the majority coming from the 90-rated bin. Meat in the 90-rated bin ranges from 80-95 while meat in the 70-rated bin ranges from 55-80. The variability in the quality of the premium product is being driven by the variability within a 90-rated bin.

• To reduce the variability, the client could (1) negotiate with the supplier to narrow the range within a bin or (2) sort the meat within the 90-rated bin at his own facility. The impact of the first proposal will depend on the relationship with the supplier. That is, is the client a major buyer; how much longer is the contract set to run.? The second option will add cost to the production process and reduce margins.